130
Polos Olímpicos de Treinamento Curso de Álgebra - Nível 3 Prof. Antonio Caminha Aula 1 Desigualdades 1 Nesta aula, aprenderemos e exercitaremos a desigualdade entre as m´ edias aritm´ etica e geom´ etrica e a desigualdade de Cauchy, bem como alguns corol´ arios seus. Para saber mais sobre o conte´ udo desta aula, sugerimos as se¸ oes 7.1a7.3 de [1], 2.3e2.4 de [3] e a se¸ ao 5.5 de [4]. A observa¸ ao b´ asica para o estudo sistem´atico de desigualdades ´ e o fato do quadrado de todo n´ umero real ser n˜ ao negativo, sendo igual a zero se e s´ o se o n´ umero em quest˜ ao for tamb´ em igual a zero. Portanto, para x, y R temos (|x|-|y|) 2 0, ocorrendo a igualdade se e s´ o se |x| = |y|. Desenvolvendo a express˜ ao entre parˆ enteses, conclu´ ımos que x 2 + y 2 2 ≥|xy|, (1) ocorrendo a igualdade se e s´ o se |x| = |y|. Assim, partindo de dois n´ umeros reais positivos a e b e fazendo x = a 0e y = b 0, segue da desigualdade acima que a + b 2 ab, (2) ocorrendo a igualdade se e s´ o se a = b, i.e., se e s´ o se a = b. Exemplo 1. Para x,y,z reais positivos, temos x 2 + y 2 + z 2 xy + xz + yz, (3) ocorrendo a igualdade se, e somente se, x = y = z. Prova. Para obter a desigualdade do enunciado, basta somar membro a membro as desi- gualdades parciais (obtidas a partir de (2)) x 2 + y 2 2 xy, x 2 + z 2 2 xz, y 2 + z 2 2 yz. Se x = y = z, ent˜ ao a desigualdade do enunciado ´ e claramente uma igualdade. Reci- procamente, se ao menos uma das desigualdades acima for estrita, digamos x 2 +y 2 2 > xy, ent˜ ao, ap´ os somarmos as mesmas membro a membro, obteremos x 2 + y 2 + z 2 > xy + xz + yz.

Álgebra nível 3

Embed Size (px)

Citation preview

Page 1: Álgebra nível 3

Polos Olímpicos de TreinamentoCurso de Álgebra - Nível 3Prof. Antonio Caminha

Aula 1

Desigualdades 1

Nesta aula, aprenderemos e exercitaremos a desigualdade entre as medias aritmetica egeometrica e a desigualdade de Cauchy, bem como alguns corolarios seus. Para saber maissobre o conteudo desta aula, sugerimos as secoes 7.1 a 7.3 de [1], 2.3 e 2.4 de [3] e a secao5.5 de [4].

A observacao basica para o estudo sistematico de desigualdades e o fato do quadradode todo numero real ser nao negativo, sendo igual a zero se e so se o numero em questao fortambem igual a zero. Portanto, para x, y ∈ R temos (|x|− |y|)2 ≥ 0, ocorrendo a igualdadese e so se |x| = |y|. Desenvolvendo a expressao entre parenteses, concluımos que

x2 + y2

2≥ |xy|, (1)

ocorrendo a igualdade se e so se |x| = |y|. Assim, partindo de dois numeros reais positivosa e b e fazendo x =

√a ≥ 0 e y =

√b ≥ 0, segue da desigualdade acima que

a+ b

2≥

√ab, (2)

ocorrendo a igualdade se e so se√a =

√b, i.e., se e so se a = b.

Exemplo 1. Para x, y, z reais positivos, temos

x2 + y2 + z2 ≥ xy + xz + yz, (3)

ocorrendo a igualdade se, e somente se, x = y = z.

Prova. Para obter a desigualdade do enunciado, basta somar membro a membro as desi-gualdades parciais (obtidas a partir de (2))

x2 + y2

2≥ xy,

x2 + z2

2≥ xz,

y2 + z2

2≥ yz.

Se x = y = z, entao a desigualdade do enunciado e claramente uma igualdade. Reci-

procamente, se ao menos uma das desigualdades acima for estrita, digamos x2+y2

2 > xy,entao, apos somarmos as mesmas membro a membro, obteremos

x2 + y2 + z2 > xy + xz + yz.

Page 2: Álgebra nível 3

POT 2012 - Algebra - Nıvel 3 - Aula 01 - Prof. Antonio Caminha

A desigualdade (2) e um caso particular da desigualdade entre as medias aritmetica

e geometrica. A fim de enunciar e provar tal generalizacao precisamos, inicialmente, daseguinte

Definicao 2. Para n > 1 numeros reais positivos a1, a2, . . . , an, definimos sua:

(a) Media aritmetica como o numero a1+a2+···+ann

.

(b) Media geometrica como o numero n√a1a2 . . . an.

No contexto da definicao acima, o que fizemos em (2) foi mostrar que a media aritmeticade dois reais positivos e sempre maior ou igual que sua media geometrica, ocorrendo aigualdade somente se os dois numeros forem iguais. Estabelecemos o caso geral no resultadoa seguir, sendo (4) conhecida como a desigualdade entre as medias.

Teorema 3. Dados n > 1 reais positivos a1, a2, . . . , an, temos

a1 + a2 + · · ·+ an

n≥ n

√a1a2 · · · an, (4)

ocorrendo a igualdade se e so se a1 = a2 = · · · = an.

Para entender a dinamica da prova do teorema acima, analisemos separadamente oscasos n = 3 e n = 4, comecando com o caso n = 4. Para tanto, dados reais positivosa, b, c, d, ja sabemos que a+b

2 ≥√ab e c+d

2 ≥√cd. Daı,

a+ b+ c+ d

4=

a+b2 + c+d

2

2≥

√ab+

√cd

2≥

√√ab√cd =

4√abcd.

Mostramos, acima, que a+b+c+d4 ≥ 4

√abcd. Escrevendo tal desigualdade com 3

√abc no

lugar de d, obtemos

a+ b+ c+ 3√abc

4≥ 4

abc3√abc =

4√d3d = d =

3√abc.

Segue, daı, a desigualdade a+ b+ c+ 3√abc ≥ 4 3

√abc, ou, o que e o mesmo, a+b+c

3 ≥ 3√abc.

Conforme veremos a seguir, a prova da versao geral da desigualdade entre as medias euma adaptacao dos argumentos utilizados para os dois casos acima.

Prova do Teorema 3. Inicialmente, provemos por inducao que a desigualdade desejadae verdadeira sempre que n for uma potencia de 2, ocorrendo a igualdade se e so se a1 =a2 = · · · = an. Para tanto, temos de verificar o caso inicial n = 2 (o que ja foi feito ao longoda discussao que estabeleceu (2)), formular a hipotese de inducao (para n = 2j , digamos)e executar o passo de inducao (deduzir o caso n = 2j+1 a partir do caso n = 2j). Masdesde que 2j+1 = 2 · 2j , basta supormos que a desigualdade seja verdadeira para quaisquerk reais positivos, com igualdade se e so se os k numeros forem todos iguais, e deduzir apartir daı que ela tambem sera verdadeira para quaisquer 2k reais positivos, com igualdade

2

Page 3: Álgebra nível 3

POT 2012 - Algebra - Nıvel 3 - Aula 01 - Prof. Antonio Caminha

novamente se e so se todos os numeros forem iguais. Para estabelecer esse fato, considereos 2k reais positivos a1, a2, . . . , a2k. Entao:

1

2k

2k∑

j=1

aj =1

2

1

k

k∑

j=1

aj +1

k

k∑

j=1

ak+j

≥ 1

2

(

k√a1 . . . ak + k

√ak+1 . . . a2k

)

≥√

k√a1 . . . ak k

√ak+1 . . . a2k = 2k

√a1 . . . akak+1 . . . a2k.

Para haver igualdade, devemos ter igualdade em todas as passagens. Entao, deve ser

a1 + · · · + ak

k= k

√a1 . . . ak,

ak+1 + · · · + a2k

k= k

√ak+1 . . . a2k

ek√a1 . . . ak + k

√ak+1 . . . a2k

2=

k√a1 . . . ak k

√ak+1 . . . a2k.

Para as duas primeiras igualdades, devemos ter por hipotese que a1 = · · · = ak e ak+1 =· · · = a2k. Por fim, a ultima igualdade ocorre se e so se k

√a1 . . . ak = k

√ak+1 . . . a2k, e esta

condicao, juntamente com as duas anteriores, implica que devemos ter a1 = · · · = ak =ak+1 = · · · = a2k. E tambem evidente que, se os numeros forem todos iguais, entao aigualdade ocorre (verifique!). Logo, por inducao temos (4) verdadeira, com a condicao paraa igualdade dada no enunciado, sempre que n for uma potencia de 2.

Provemos agora, por inducao forte, que a desigualdade e verdadeira em geral, ocorrendoa igualdade se e so se os numeros forem todos iguais. Para tanto, seja n > 1 natural ea1, a2, . . . , an reais positivos dados. Tome k ∈ N tal que 2k > n. Aplicando a desigualdadeentre as medias aos n numeros a1, a2, . . . , an, juntamente com 2k − n copias do numeroa = n

√a1a2 . . . an (totalizando n+ (2k − n) = 2k numeros), obtemos

a1 + · · ·+ an + a+ · · ·+ a

2k≥ 2

k√

a1 . . . an · a2k−n =2k√

ana2k−n =

2k√

a2k = a.

A partir daı, obtemos a1 + a2 + · · ·+ an + (2k − n)a ≥ 2ka ou, ainda,

a1 + a2 + · · ·+ an

n≥ a = n

√a1a2 . . . an.

Para haver igualdade, segue da primeira parte que a1 = a2 = · · · = an = a = · · · = a.Em particular, todos os numeros a1, a2, . . . , an devem ser iguais. Finalmente, e facil verque se esses numeros forem todos iguais, entao havera igualdade.

Exemplo 4. Para n > 1 reais positivos a1, a2, . . . , an, temos

(a1 + a2 + · · ·+ an)

(

1

a1+

1

a2+ · · · + 1

an

)

≥ n2, (5)

ocorrendo a igualdade se e so se a1 = a2 = · · · = an.

3

Page 4: Álgebra nível 3

POT 2012 - Algebra - Nıvel 3 - Aula 01 - Prof. Antonio Caminha

Prova. Aplicando duas vezes a desigualdade entre as medias, temos

(a1 + a2 + · · ·+ an)

(

1

a1+

1

a2+ · · ·+ 1

an

)

≥ (n n√a1a2 · · · an)

(

n n

1

a1· 1

a2· · · 1

an

)

= n2.

Para haver a igualdade, devemos ter a1 + a2 + · · · + an = n n√a1a2 · · · an, donde a1 = a2 =

· · · = an. Reciprocamente, e imediato verificar que se todos os numeros forem iguais, entaoteremos igualdade em (5).

Exemplo 5 (Asia-Pacıfico). Se a, b e c sao reais positivos, prove que

(

1 +a

b

)

(

1 +b

c

)

(

1 +c

a

)

≥ 2

(

1 +a+ b+ c

3√abc

)

.

Prova. Desenvolvendo o primeiro membro, obtemos

(

1 +a

b

)

(

1 +b

c

)

(

1 +c

a

)

= 2 +a+ c

b+

b+ c

a+

a+ b

c,

donde basta mostrarmos que

a+ c

b+

b+ c

a+

a+ b

c≥ 2(a+ b+ c)

3√abc

.

Denotando por S o primeiro membro da expressao acima, segue da desigualdade entreas medias e de (5) que

S = (a+ b+ c)

(

1

a+

1

b+

1

c

)

− 3

=2

3(a+ b+ c)

(

1

a+

1

b+

1

c

)

+1

3(a+ b+ c)

(

1

a+

1

b+

1

c

)

− 3

≥ 2

3(a+ b+ c)

(

33√abc

)

+1

3· 9− 3

=2(a+ b+ c)

3√abc

.

Voltando a (1), suponha dados numeros reais a1, a2, a3 e b1, b2, b3, tais que a21+a22+a23 =

1 e b21 + b22 + b23 = 1. Temos

a21 + b21 ≥ |a1b1|, a22 + b22 ≥ |a2b2|, a23 + b23 ≥ |a3b3|, (6)

ocorrendo a igualdade se e so se |a1| = |b1|, |a2| = |b2|, |a3| = |b3|. Somando membro amembro as desigualdades acima, obtemos

(a21 + a22 + a23) + (b21 + b22 + b23) = (a21 + b21) + (a22 + b22) + (a23 + b23)

≥ 2(|a1b1|+ |a2b2|+ |a3b3|)≥ 2|a1b1 + a2b2 + a3b3|,

4

Page 5: Álgebra nível 3

POT 2012 - Algebra - Nıvel 3 - Aula 01 - Prof. Antonio Caminha

onde, na ultima desigualdade, aplicamos a desigualdade triangular para tres numeros (cf.Problema 1).

Portanto, provamos acima que, se a21 + a22 + a23 = 1 e b21 + b22 + b23 = 1, entao

|a1b1 + a2b2 + a3b3| ≤ 1. (7)

A igualdade ocorre se e so se tivermos igualdade tanto nas desigualdades em (6) quantona desigualdade triangular utilizada, i.e., se e so se |a1| = |b1|, |a2| = |b2| e |a3| = |b3| e,alem disso, a1b1, a2b2, a3b3 ≥ 0 ou a1b1, a2b2, a3b3 ≤ 0. Mas e imediato verificar que taiscondicoes sao equivalentes a a1 = b1, a2 = b2 e a3 = b3.

Considere, agora, numeros reais a1, a2, a3 e b1, b2, b3 quaisquer, exceto pelo fato de quepelo menos um dos numeros a1, a2, a3 e pelo menos um dos numeros b1, b2, b3 sao nao nulos.Fazendo c =

a21 + a22 + a23 e d =√

b21 + b22 + b23, temos c, d > 0; portanto, se xi =aic

e

yi =bid, para 1 ≤ i ≤ 3, temos x21+x22+x23 =

a21+a2

2+a2

3

c2= 1 e, analogamente, y21+y22+y23 = 1.

Segue, pois, de (7) que|x1y1 + x2y2 + x3y3| ≤ 1,

com igualdade se e so se xi = yi para 1 ≤ i ≤ 3.Substituindo as definicoes de xi e yi na desigualdade acima, concluımos ser ela equiva-

lente a desigualdade

|a1b1 + a2b2 + a3b3| ≤ cd =√

a21 + a22 + a23

b21 + b22 + b23.

Ademais, ha igualdade se e so se ai =cd· bi para 1 ≤ i ≤ 3.

A discussao acima estabeleceu, para n = 3, a desigualdade do teorema a seguir, conhe-cida como a desigualdade de Cauchy.

Teorema 6 (Cauchy). Sejam n > 1 inteiro e a1, a2, . . . , an, b1, b2, . . . , bn numeros reais

dados. Entao∣

n∑

j=1

ajbj

n∑

j=1

a2j ·

n∑

j=1

b2j , (8)

ocorrendo a igualdade se e so se os ai e os bi forem respectivamente proporcionais, i.e., see so se existir um real nao nulo λ tal que a1 = λb1, a2 = λb2, an = λbn.

Prova. Se todos os ai ou todos os bi forem iguais a zero, nada ha a fazer. Senao, a fim deestabelecer (8), basta seguir os passos do caso particular n = 3 discutido acima, tomandoo cuidado de, no momento oportuno, utilizar o caso geral da desigualdade triangular.

Os dois exemplos a seguir ilustram a utilizacao da desigualdade de Cauchy.

Exemplo 7 (Romenia). Sejam x1, x2, . . ., xn+1 reais positivos tais que x1+x2+ · · ·+xn =xn+1. Prove que

x1(xn+1 − x1) + · · ·+√

xn(xn+1 − xn) ≤√

xn+1(xn+1 − x1) + · · · + xn+1(xn+1 − xn).

5

Page 6: Álgebra nível 3

POT 2012 - Algebra - Nıvel 3 - Aula 01 - Prof. Antonio Caminha

Prova. Para 1 ≤ j ≤ n seja yj = xn+1 − xj . Pela desigualdade de Cauchy, temos

√x1y1 + · · ·+√

xnyn ≤√x1 + · · · + xn

√y1 + · · · + yn

=√xn+1

(xn+1 − x1) + · · ·+ (xn+1 − xn).

Exemplo 8. Dados numeros reais a1, . . . , an e b1, . . . , bn, temos√

n∑

j=1

(aj + bj)2 ≤

n∑

j=1

a2j +

n∑

j=1

b2j , (9)

ocorrendo a igualdade se e so se a1, . . . , an e b1, . . . , bn forem positivamente proporcionais,i.e., se e so se existir um real positivo λ, tal que ai = λbi para 1 ≤ i ≤ n.

Prova. Facamos a prova para n = 3, sendo o caso geral inteiramente analogo. Uma vezque ambos os membros de (9) sao reais nao negativos, basta mostrar que o quadrado doprimeiro membro e menor ou igual que o quadrado do segundo membro, i.e., que

(a1 + b1)2 + (a2 + b2)

2 + (a3 + b3)2 ≤

(

a21 + a22 + a23 +√

b21 + b22 + b23

)2

.

Desenvolvendo todos os quadrados (ai+bi)2, segue que o quadrado do primeiro membro

e igual a(a21 + 2a1b1 + b21) + (a22 + 2a2b2 + b22) + (a23 + 2a3b3 + b23).

Analogamente, o quadrado do segundo membro e igual a

(a21 + a22 + a23) + 2√

a21 + a22 + a23

b21 + b22 + b23 + (b21 + b22 + b23).

Mas, como em ambas as expressoes temos a parcela (a21 + a22 + a23) + (b21 + b22 + b23), adesigualdade do enunciado e equivalente a

2(a1b1 + a2b2 + a3b3) ≤ 2√

a21 + a22 + a23

b21 + b22 + b23,

a qual e, precisamente, a desigualdade de Cauchy.A deducao das condicoes para a igualdade fica a cargo do leitor.

Problemas

1. * Dados numeros reais nao nulos x1, x2, . . .xn, prove a desigualdade triangular:

|x1 + x2 + · · ·+ xn| ≤ |x1|+ |x2|+ · · ·+ |xn|,

ocorrendo a igualdade se e so se x1, x2, . . . , xn tiverem um mesmo sinal.

6

Page 7: Álgebra nível 3

POT 2012 - Algebra - Nıvel 3 - Aula 01 - Prof. Antonio Caminha

2. (OBM). Sejam a, b, c reais positivos dados. Prove que

(a+ b)(a+ c) ≥ 2√

abc(a+ b+ c).

3. Dispomos de uma folha de cartolina de 2m por 3m e queremos construir com a mesmauma caixa aberta com o maior volume possıvel. Quais devem ser as dimensoes dacaixa? Justifique sua resposta.

4. (Estados Unidos). Prove que, para todos a, b, c reais positivos, tem-se

1

a3 + b3 + abc+

1

b3 + c3 + abc+

1

c3 + a3 + abc≤ 1

abc.

Para os dois problemas a seguir precisamos de um pouco de geometria Euclidianaplana. Mais precisamente (cf. Figura 1), sendo a = BC, b = AC e c = AB oscomprimentos dos lados de um triangulo ABC, existem x, y, z > 0 tais que a = y+z,b = x + z e c = x + y: basta tomar x, y e z como sendo iguais aos comprimentosdos segmentos determinados sobre os lados de ABC pelos pontos de tangencia comos mesmos do cırculo inscrito em ABC (para uma prova de tais afirmacoes, veja oCapıtulo 3 de [2]). No contexto de desigualdades envolvendo os lados a, b e c deum triangulo, a substituicao dos mesmos respectivamente por y + z, x+ z e x+ y econhecida como a transformacao de Ravi.

A

B

C

I

x

x

y y

z

z

Figura 1: a transformacao de Ravi.

5. (IMO). Se a, b, c sao os comprimentos dos lados de um triangulo, prove que

abc ≥ (a+ b− c)(b+ c− a)(c+ a− b).

6. Sejam a, b, c os comprimentos dos lados de um triangulo. Prove que

a

b+ c− a+

b

c+ a− b+

c

a+ b− c≥ 3.

7. (Baltic Way). Sejam a, b, c, d reais positivos dados. Prove que

a+ c

a+ b+

b+ d

b+ c+

c+ a

c+ d+

d+ b

d+ a≥ 4.

7

Page 8: Álgebra nível 3

POT 2012 - Algebra - Nıvel 3 - Aula 01 - Prof. Antonio Caminha

8. (Uniao Sovietica). Sejam a, b, c reais positivos. Prove que

(ab+ ac+ bc)2 ≥ 3abc(a + b+ c).

9. Sejam a, b e c reais positivos dados. Prove que 9(a3 + b3 + c3) ≥ (a+ b+ c)3.

10. Dados a, b, c reais positivos, prove que

a4(1 + b4) + b4(1 + c4) + c4(1 + a4) ≥ 6a2b2c2,

com igualdade se e so se |a| = |b| = |c| = 1.

11. Sejam a1, a2, . . . , an reais positivos. Prove que

a1

a2+

a2

a3+

a3

a4+ · · · + an−1

an+

an

a1≥ n.

12. O proposito deste problema e apresentar uma segunda demonstracao da desigualdade(5), a qual nao faz uso da desigualdade (4). Para tanto, faca os dois itens a seguir:

(a) Mostre que

(a1 + a2 + · · · + an)

(

1

a1+

1

a2+ · · ·+ 1

an

)

= n+∑

i<j

(

ai

aj+

aj

ai

)

.

(b) Aplique a desigualdade entre as medias para cada uma das parcelas aiaj

+ajai

do

somatorio acima e obtenha (5).

13. (Romenia). Sejam n > 1 inteiro e 0 < a1 < a2 < · · · < an reais dados. Prove que

12

a1+

22

a2+ · · ·+ n2

an≤ n

a1+

n− 1

a2 − a1+

n− 2

a3 − a2+ · · ·+ 1

an − an−1.

Sob que condicoes a igualdade ocorre?

14. (China). Para a, b e c reais positivos, prove que

a+√ab+ 3

√abc

3≤ 3

a

(

a+ b

2

)(

a+ b+ c

3

)

.

15. Dados reais positivos a1, a2, . . . , an, definimos sua media quadratica como o numeroreal

a21 + a22 + · · ·+ a2nn

.

Prove a desigualdade entre as medias quadratica e aritmetica:√

a21 + a22 + · · · + a2nn

≥ a1 + a2 + · · ·+ an

n, (10)

com igualdade se e so se a1 = a2 = · · · = an.

8

Page 9: Álgebra nível 3

POT 2012 - Algebra - Nıvel 3 - Aula 01 - Prof. Antonio Caminha

16. Sejam a1, a2, a3, a4 reais positivos. Prove que

1≤i<j<k≤4

a2i + a2j + a2k

ai + aj + ak≥ a1 + a2 + a3 + a4,

ocorrendo a igualdade se e so se a1 = a2 = a3 = a4.

17. (Leningrado). Dados reais positivos a, b, c e d, prove que

1

a+

1

b+

4

c+

16

d≥ 64

a+ b+ c+ d.

18. (Uniao Sovietica). Se x, y, z > 0, prove que x2

y2+ y2

z2+ z2

x2 ≥ yx+ z

y+ x

z.

19. (Torneio das Cidades). Sejam a1, a2, . . . , an reais positivos dados. Prove que

(

1 +a21a2

)(

1 +a22a3

)

. . .

(

1 +a2na1

)

≥ (1 + a1)(1 + a2) · · · (1 + an).

20. (IMO). Sejam a, b e c reais positivos tais que abc = 1. Prove que

1

a3(b+ c)+

1

b3(a+ c)+

1

c3(a+ b)≥ 3

2.

Bibliografia

1. A. Caminha. Topicos de Matematica Elementar, Volume 1: Numeros Reais. Socie-dade Brasileira de Matematica, Rio de Janeiro, 2012.

2. A. Caminha. Topicos de Matematica Elementar, Volume 2: Geometria Euclidiana

Plana. Sociedade Brasileira de Matematica, Rio de Janeiro, 2012.

3. E. Lozansky e C. Rousseau. Winning Solutions. Springer-Verlag, Nova Iorque, 1996.

4. P. Zeitz. The Art and Craft of Problem Solving. John Wiley & Sons, Nova Iorque,1999.

9

Page 10: Álgebra nível 3

POT 2012 - Algebra - Nıvel 3 - Aula 01 - Prof. Antonio Caminha

Dicas e Solucoes

1. Faca inducao sobre n ≥ 2. Para o caso inicial, eleve ambos os membros da desigual-dade em questao ao quadrado e utilize (1), juntamente com o fato de que |a|2 = a2,para todo a ∈ R.

2. Desenvolva o primeiro membro e, em seguida, aplique a desigualdade (2) adequada-mente.

3. Sendo x o comprimento do lado do quadrado que deve ser recortado de cada cantoda folha, ficaremos com uma caixa de dimensoes 2− 2x, 3− 2x e x. Escolha numerosreais positivos a, b e c tais que a(2−2x)+b(3−2x)+cx independa de x e a(2−2x) =b(3−2x) = cx; em seguida aplique a desigualdade entre as medias a fim de maximizaro volume da caixa.

4. Mostre inicialmente que a3 + b3 ≥ (a + b)ab; em seguida, deduza a partir daı que1

a3+b3+abc≤ c

abc(a+b+c) , obtendo desigualdades analogas para as outras duas parcelasdo primeiro membro.

5. Aplique a transformacao de Ravi e, em seguida, utilize (2) tres vezes.

6. Aplique a transformacao de Ravi para escrever o primeiro membro como

1

2

(

y

x+

z

x+

x

y+

z

y+

y

z+

x

z

)

;

em seguida, utilize a desigualdade entre as medias.

7. Agrupe adequadamente as quatro parcelas em pares e utilize duas vezes a desigual-dade (5), para n = 2.

8. Inicialmente, mostre que e suficiente provar que (ab)2+(bc)2+(ca)2 ≥ abc(a+ b+ c);para o que falta, faca x = ab, y = bc, z = ca e aplique a desigualdade (3).

9. Inicialmente, mostre a identidade algebrica

(a+ b+ c)3 = a3 + b3 + c3 + 3(a+ b)(a+ c)(b+ c);

em seguida, apos efetuar as simplificacoes obvias, mostre que

8(a3 + b3 + c3) ≥ (a+ b)3 + (a+ c)3 + (b+ c)3

e utilize a desigualdade entre as medias para tres numeros.

10. Aplique a desigualdade entre as medias.

11. Aplique a desigualdade entre as medias.

10

Page 11: Álgebra nível 3

POT 2012 - Algebra - Nıvel 3 - Aula 01 - Prof. Antonio Caminha

13. Faca a0 = 0 e aplique a desigualdade (5) para obter

1

aj − aj−1+ · · ·+ 1

a1 − a0≥ j2

aj.

Em seguida, some membro a membro as desigualdades acima para 1 ≤ j ≤ n eagrupe os termos iguais para obter a desigualdade procurada. Por fim, conclua queha igualdade se e so se a sequencia (ak)k≥1 for uma PA.

14. Substituindo a, b e c na desigualdade desejada respectivamente por 6x6, 6y6 e 6z6,mostre que basta provarmos a desigualdade

7x12 + 12x6y6 + 7y6z6 + 9y12 + 9x6z6 ≥

≥ 2x3y9 + 6x9y3 + 6x2y8z2 + 12x5y5z2 + 6x4y4z4 + 6xy7z4 + 6x8y2z2.

Para tanto, escreva a expressao do primeiro membro como a soma de sete outrasexpressoes tais que, aplicando a desigualdade entre as medias a cada uma delas,obtenhamos as sete parcelas do segundo membro; por exemplo,

2x6z6 + 2x6z6 + 2x12 + 2y12 + 2y12 + 2x6y6 ≥ 12x5y5z2.

15. Aplique a desigualdade de Cauchy com b1 = b2 = · · · = bn = 1.

16. Aplique a desigualdade do problema anterior ao numerador de cada parcela do so-matorio acima.

17. Multiplique ambos os membros por a + b + c + d e use a desigualdade de Cauchy;alternativamente, tente aplicar a desigualdade (5).

18. Use a desigualdade de Cauchy.

19. Faca a prova por inducao sobre n ≥ 2. Para o passo de inducao, aplique a desigualdadede Cauchy.

20. Faca x = 1a, y = 1

be z = 1

ce, em seguida, aplique a desigualdade de Cauchy para

obter

((y + z) + (x+ z) + (x+ y))

(

x2

y + z+

y2

x+ z+

z2

x+ y

)

≥ (x+ y + z)2.

Por fim, aplique a desigualdade entre as medias.

11

Page 12: Álgebra nível 3

Polos Olímpicos de TreinamentoCurso de Álgebra - Nível 3Prof. Antonio Caminha

Aula 2

Desigualdades 2

Esta aula e devotada ao estudo de outras desigualdades elementares importantes. Parasaber mais sobre o material aqui discutido, remetemos o leitor ao Capıtulo 2 de [1], a Secao7.4 de [2], ao Capıtulo 7 de [3] ou, por fim, a Secao 2.4 de [4].

A primeira desigualdade que apresentamos remonta os irmaos Bernoulli (Jacob e JohannBernoulli, matematicos suıcos do seculo XVIII), sendo conhecida como a desigualdade

de Bernoulli. Apesar de sua aparente simplicidade, veremos que ela se revela bastanteutil em aplicacoes.

Proposicao 1 (Bernoulli). Dados n natural e x > −1 real, temos (1 + x)n ≥ 1 + nx,

ocorrendo a igualdade para n > 1 se e so se x = 0.

Prova. Facamos inducao sobre n, sendo o caso n = 1 imediato. Suponha, por hipotese deinducao, que (1 + x)k ≥ 1 + kx; como 1 + x > 0, temos

(1 + x)k+1 = (1 + x)(1 + x)k ≥ (1 + x)(1 + kx) = 1 + (k + 1)x+ kx2 ≥ 1 + (k + 1)x,

ocorrendo a igualdade se e so se (1 + x)k = 1 + kx e kx2 = 0, i.e., se e so se x = 0.

Exemplo 2. Dados n natural e a e b reais positivos, mostre que

(

1 +a

b

)n

+

(

1 +b

a

)n

≥ 2n+1,

ocorrendo a igualdade se e so se a = b.

Prova. Dividindo ambos os membros da desigualdade do enunciado por 2n, vemos quebasta provar que

(

1− 1

2+

a

2b

)n

+

(

1− 1

2+

b

2a

)n

≥ 2.

Como −12 +

a2b > −1 e −1

2 +b2a > −1, aplicando a desigualdade de Bernoulli a cada parcela

do primeiro membro acima e somando os resultados, obtemos

(

1− 1

2+

a

2b

)n

+

(

1− 1

2+

b

2a

)n

≥ 2 + n

(

a

2b+

b

2a− 1

)

.

Page 13: Álgebra nível 3

POT 2012 - Algebra - Nıvel 3 - Aula 02 - Prof. Antonio Caminha

Basta, agora, aplicar a desigualdade entre as medias para obter

a

2b+

b

2a− 1 ≥ 2

a

2b· b

2a− 1 = 0,

com igualdade se e so se a2b = b

2a , i.e., se e so se a = b.

A proxima desigualdade que apresentamos e conhecida na literatura como a desigual-

dade de Chebyshev, assim nomeada apos Pafnuty Chebyshev, matematico russo doseculo XIX.

Teorema 3 (Chebyshev). Se a1, a2, . . . , an e b1, b2, . . . , bn sao numeros reais tais que

a1 ≤ a2 ≤ · · · ≤ an e b1 ≤ b2 ≤ · · · ≤ bn,

entao(

1

n

n∑

i=1

ai

)(

1

n

n∑

i=1

bi

)

≤ 1

n

n∑

i=1

aibi,

ocorrendo a igualdade se e so se a1 = a2 = · · · = an ou b1 = b2 = · · · = bn.

Prova. Temos de mostrar que

n

n∑

i=1

aibi −(

n∑

i=1

ai

)(

n∑

i=1

bi

)

≥ 0,

para o que basta observar que a expressao do primeiro membro e igual a

n∑

i,j=1

(ai − aj)(bi − bj).

Mas, como os ai’s e bi’s sao igualmente ordenados, concluımos que a expressao acima e,realmente, nao negativa.

Note agora que, se a1 = a2 = · · · = an ou b1 = b2 = · · · = bn, entao havera igualdadena desigualdade de Chebyshev. Reciprocamente, suponha que temos igualdade em taldesigualdade. Como (ai − aj)(bi − bj) ≥ 0 para todos os ındices i, j, para haver igualdadedevemos ter (ai − aj)(bi − bj) = 0 para todos i, j = 1, . . . , n. Se existir 1 ≤ k ≤ n tal quebk < bk+1, entao b1 ≤ · · · ≤ bk < bk+1 ≤ · · · ≤ bn e a condicao (ai − ak+1)(bi − bk+1) = 0para todo i garante que ai = ak+1 para i ≤ k. Portanto, temos a1 = a2 = · · · = ak = ak+1.Por outro lado, a partir de (ai−ak)(bi−bk) = 0 para i > k, concluımos que ak+1 = · · · = an.Logo, todos os ai’s devem ser iguais.

Exemplo 4. Se k e um natural e a1, a2, . . . , an sao reais positivos, entao

ak1 + ak2 + · · ·+ aknn

≥(

a1 + a2 + · · ·+ an

n

)k

, (1)

com igualdade se e so se todos os ai’s forem iguais.

2

Page 14: Álgebra nível 3

POT 2012 - Algebra - Nıvel 3 - Aula 02 - Prof. Antonio Caminha

Prova. Facamos inducao sobre k ≥ 1, sendo (1) trivialmente verdadeira para k = 1 etodos os a1, a2, . . . , an reais positivos. Seja agora l > 1 um natural tal que (1) valha parak = l − 1 e todos a1, a2, . . . , an reais positivos. Dados reais positivos a1, a2, . . . , an, comoambos os membros da desigualdade que queremos provar sao invariantes por permutacoesdos ındices 1, 2, . . . , n podemos supor, sem perda de generalidade, que a1 ≤ a2 ≤ · · · ≤ an.Daı, temos al−1

1 ≤ al−12 ≤ · · · ≤ al−1

n , e segue da desigualdade de Chebyshev que

1

n

n∑

i=1

ali ≥(

1

n

n∑

i=1

ai

)(

1

n

n∑

i=1

al−1i

)

.

Por outro lado, a hipotese de inducao fornece

1

n

n∑

i=1

al−1i ≥

(

1

n

n∑

i=1

ai

)l−1

,

e combinando essas duas desigualdades obtemos

1

n

n∑

i=1

ali ≥(

1

n

n∑

i=1

ai

)l

,

conforme desejado. Por fim, a condicao de igualdade e obvia a partir condicao de igualdadena desigualdade de Chebyshev.

Exemplo 5 (Polonia). Sejam a1, a2, . . . , an reais positivos com soma s. Prove que

a1

s− a1+

a2

s− a2+ · · ·+ an

s− an≥ n

n− 1.

Prova. Suponhamos, sem perda de generalidade, que a1 ≤ a2 ≤ · · · ≤ an. Entao s− a1 ≥s− a2 ≥ · · · ≥ s − an. Como s− ai > 0 para todo i, segue que 1

s−a1≤ 1

s−a2≤ · · · ≤ 1

s−an.

Portanto, pela desigualdade de Chebyshev temos

n∑

i=1

ai

s− ai=

n∑

i=1

(

ai ·1

s− ai

)

≥ 1

n

(

n∑

i=1

ai

)(

n∑

i=1

1

s− ai

)

=s

n

(

n∑

i=1

1

s− ai

)

.

Mas, pelo Exemplo 4 da Aula 1, temos

n∑

i=1

1

s− ai≥ n2

(

n∑

i=1

(s− ai)

)−1

= n2(ns− s)−1 =n2

(n− 1)s.

Por fim, basta combinar as duas desigualdades acima.

A segunda igualdade que apresentamos e conhecida como a desigualdade do rear-

ranjo. Para o enunciado da mesma, recorde que uma permutacao de uma sequencia(a1, a2, . . . , an), e uma sequencia (x1, x2, . . . , xn) tal que xi = aσ(i), para alguma bijecaoσ : {1, 2, . . . , n} → {1, 2, . . . , n}.

3

Page 15: Álgebra nível 3

POT 2012 - Algebra - Nıvel 3 - Aula 02 - Prof. Antonio Caminha

Proposicao 6. Sejam a1 < a2 < · · · < an reais positivos dados. Se (x1, x2, . . . , xn) e uma

permutacao qualquer de (a1, a2, . . . , an), entao

n−1∑

i=1

aian−i ≤n−1∑

i=1

aixi ≤n−1∑

i=1

a2i ,

ocorrendo a igualdade na primeira (resp. segunda) desigualdade acima se e so se xi = an−i

(resp. xi = ai) para 1 ≤ i ≤ n.

Prova. Mostremos como maximizar a soma a1x1 + a2x2 + · · · + anxn, sendo o raciocıniopara minimiza-la totalmente analogo.

Como o numero de permutacoes (x1, x2, . . . , xn) dos ai’s e finito, ha pelo menos umadelas que maximiza a soma a1x1+a2x2+· · ·+anxn. Se (b1, b2, . . . , bn) e uma tal permutacao,queremos mostrar que bi = ai para 1 ≤ i ≤ n, e para tanto basta mostrarmos que deve serb1 < b2 < · · · < bn. Suponha o contrario, i.e., que existam ındices i < j tais que bi > bj .Defina a permutacao (b′1, b

′2, . . . , b

′n) dos ai’s pondo

b′k =

bk, se k 6= i, j

bi, se k = j

bj, se k = i

.

Entao

n∑

i=1

aib′i −

n∑

i=1

aibi = (aib′i + ajb

′j)− (aibi + ajbj)

= (aibj + ajbi)− (aibi + ajbj)

= (ai − aj)(bj − bi) > 0.

Isso e o mesmo que

a1b′1 + a2b

′2 + · · ·+ anb

′n > a1b1 + a2b2 + · · ·+ anbn,

o que por sua vez contraria o fato de que a permutacao (b1, b2, . . . , bn) dos ai’s maximiza asoma a1x1 + a2x2 + · · ·+ anxn maximo. Logo, b1 < b2 < · · · < bn.

Exemplo 7. Dados reais positivos a, b e c, mostre que a3 + b3 + c3 ≥ a2b+ b2c+ c2a.

Prova. Suponha, sem perda de generalidade, que a ≤ b ≤ c. Uma aplicacao direta dadesigualdade do rearranjo nos da

a3 + b3 + c3 = a2 · a+ b2 · b+ c2 · c ≥ a2 · b+ b2 · c+ c2 · a.

Conforme mostrado pelo proximo exemplo, a ideia da prova da desigualdade do rear-ranjo e, por vezes, tao util quanto a desigualdade em si.

4

Page 16: Álgebra nível 3

POT 2012 - Algebra - Nıvel 3 - Aula 02 - Prof. Antonio Caminha

Exemplo 8 (Eslovenia). Dados 2n reais positivos a1, a2, . . . , a2n, como devemos arranja-losem pares de modo que a soma dos n produtos dos numeros de cada par seja maxima?

Solucao. Suponha, sem perda de generalidade, que a1 ≤ a2 ≤ · · · ≤ a2n e arranje taisnumeros em n pares (b1, c1), (b2, c2), . . . , (bn, cn), com b1 ≤ b2 ≤ · · · ≤ bn e bj ≤ cj , paratodo j. Queremos maximizar

S = b1c1 + b2c2 + · · ·+ bncn.

Para isso, vamos mostrar que c1 ≤ c2 ≤ · · · ≤ cn. De fato, se a sequencia (ci) nao for naodecrescente, existirao ındices i > j tais que ci ≤ cj . Neste caso, trocamos as posicoes de cie cj em S, apos o que a nova soma sera

S′ = S − bici − bjcj + bicj + bjci = S + (bi − bj)(cj − ci) ≥ S.

Logo, a soma sera maxima quando c1 ≤ c2 ≤ · · · ≤ cn.Finalmente, observe que, para todos i < j, temos bi ≤ ci ≤ cj. Suponha que, para

algum par i < j, tivessemos bj ≤ ci. Neste caso, trocamos ci por bj , de modo que a novasoma seja

S′′ = S − bici − bjcj + bibj + cicj = S + (bi − cj)(bj − ci) ≥ S.

Portanto, devemos ter, para todos i < j, bi ≤ ci ≤ bj ≤ cj . Em geral, teremos

b1 ≤ c1 ≤ b2 ≤ c2 ≤ · · · ≤ bn ≤ cn.

Logo, a soma maxima e a1a2 + a3a4 + · · ·+ a2n−1a2n.

A ultima desigualdade que discutiremos aqui e devida ao matematico noruegues doseculo XIX Niels Henrik Abel, sendo conhecida como a desigualdade de Abel.

Teorema 9 (Abel). Sejam n > 1 natural e a1, a2, . . . , an, b1, b2, . . . , bn numeros reais

dados, com a1 ≥ a2 ≥ · · · ≥ an ≥ 0. Se M e m denotam respectivamente os elementos

maximo e mınimo do conjunto de somas {b1, b1 + b2, . . . , b1 + b2 + . . .+ bn}, entao

ma1 ≤ a1b1 + a2b2 + · · ·+ anbn ≤ Ma1.

Prova. Provemos a desigualdade da direita, sendo a prova da desigualdade da esquerdatotalmente analoga.

Faca s0 = 0 e si = b1 + · · ·+ bi, para 1 ≤ i ≤ n. Entao

n∑

i=1

aibi =n∑

i=1

ai(si − si−1) =n∑

i=1

aisi −n−1∑

i=0

ai+1si

=n−1∑

i=1

(ai − ai+1)si + ansn

≤n−1∑

i=1

M(ai − ai+1) +Man = Ma1.

5

Page 17: Álgebra nível 3

POT 2012 - Algebra - Nıvel 3 - Aula 02 - Prof. Antonio Caminha

Para referencia futura, observamos que, nas notacoes da prova do teorema acima, aigualdade

n∑

i=1

aibi =n−1∑

i=1

(ai − ai+1)si + ansn (2)

e conhecida como a identidade de Abel, sendo quase tao util quanto a desigualdade deAbel em si.

Exemplo 10 (Romenia - adaptado). Sejam n > 1 inteiro e x1, . . . , xn, y1, . . . , yn reaispositivos tais que x1y1 < x2y2 < · · · < xnyn e, para 1 ≤ k ≤ n, x1+ · · ·+xk ≥ y1+ · · ·+yk.Prove que

1

x1+

1

x2+ · · · + 1

xn≤ 1

y1+

1

y2+ · · ·+ 1

yn.

Prova. Inicialmente, observe que

n∑

i=1

1

yi−

n∑

i=1

1

xi=

n∑

i=1

xi − yi

xiyi. (3)

Por outro lado, a condicao x1 + · · ·+xk ≥ y1+ · · ·+ yk para 1 ≤ k ≤ n pode ser escritacomo

(x1 − y1) + · · ·+ (xk − yk) ≥ 0

para 1 ≤ k ≤ n. Assim, fazendo ai = 1xiyi

e bi = xi − yi para 1 ≤ i ≤ n, temos

0 < a1 < a2 < · · · < an, b1 + · · · + bi ≥ 0 e xi−yixiyi

= aibi para 1 ≤ i ≤ n.Aplicando a desigualdade de Abel a (3), obtemos entao

n∑

i=1

1

yi−

n∑

i=1

1

xi≥ an ·min{b1 + · · ·+ bi; 1 ≤ i ≤ n} ≥ 0.

Problemas

1. Para n ∈ N, prove que

(

1 +1

n

)n

<

(

1 +1

n+ 1

)n+1

.

2. (Estados Unidos). Para m, n naturais, seja a = mm+1+nn+1

mm+nn . Prove que

am + an ≥ mm + nn.

6

Page 18: Álgebra nível 3

POT 2012 - Algebra - Nıvel 3 - Aula 02 - Prof. Antonio Caminha

3. (Croacia). Encontre todas as solucoes reais positivas do sistema de equacoes

{

x1 + x2 + · · ·+ x1994 = 1994x41 + x42 + · · ·+ x41994 = x31 + x32 + · · ·+ x31994

.

4. Sejam a1, a2, a3, a4 reais positivos. Prove que

1≤i<j<k≤4

a3i + a3j + a3k

ai + aj + ak≥ a21 + a22 + a23 + a24,

ocorrendo a igualdade se e so se a1 = a2 = a3 = a4.

5. Sejam n > 1 inteiro e a1, a2, . . . , an, b1, b2, . . . , bn reais dados, com a1 ≤ a2 ≤ · · · ≤ ane b1 ≤ b2 ≤ · · · ≤ bn. Se λ1 ≤ λ2 ≤ · · · ≤ λn sao reais positivos com soma igual a 1,prove que

(

n∑

i=1

λiai

)(

n∑

i=1

λibi

)

≤n∑

i=1

λiaibi

e de condicoes necessarias e suficientes para a igualdade. A que caso particularcorresponde a desigualdade de Chebyshev?

6. Sejam a, b, c reais positivos. Prove que

1

a+

1

b+

1

c≤ a8 + b8 + c8

a3b3c3.

7. Sejam a, b, c, d reais nao negativos tais que ab+ bc+ cd+ da = 1. Prove que

a3

b+ c+ d+

b3

a+ c+ d+

c3

a+ b+ d+

d3

a+ b+ c≥ 1

3.

8. Para a, b, c reais positivos e n ∈ N, prove que

an

b+ c+

bn

a+ c+

cn

a+ b≥ an−1 + bn−1 + cn−1

2.

9. Sejam x, y e z reais positivos tais que xyz = 1. Prove que

x3

(1 + y)(1 + z)+

y3

(1 + x)(1 + y)+

z3

(1 + x)(1 + y)≥ 3

4.

10. Sejam n > 1 inteiro e x1, x2, . . . , xn reais positivos dados, com soma igual a 1. Proveque

n∑

i=1

xi√1− xi

≥√

n

n− 1≥ 1√

n− 1

n∑

i=1

√xi.

11. Dados reais positivos a, b e c, mostre que a+b+cabc

≤ 1a2

+ 1b2

+ 1c2.

7

Page 19: Álgebra nível 3

POT 2012 - Algebra - Nıvel 3 - Aula 02 - Prof. Antonio Caminha

12. (IMO). Seja (ak)k≥1 uma sequencia de inteiros positivos dois a dois distintos. Proveque, para todo n ∈ N, temos

n∑

k=1

ak

k2≥

n∑

k=1

1

k.

13. (Torneio das Cidades). Sejam a1, a2, . . . , an reais positivos dados. Prove que

(

1 +a21a2

)(

1 +a22a3

)

· · ·(

1 +a2na1

)

≤n∏

k=1

(1 + ak).

14. Sejam ai, bi numeros reais tais que a1 ≥ a2 ≥ · · · ≥ a1 > 0 e b1 ≥ a1, b1b2 ≥ a1a2,. . . , b1b2 · · · bn ≥ a1a2 · · · an. Mostre que

b1 + b2 + · · · + bn ≥ a1 + a2 + · · ·+ an.

Bibliografia

1. T. Andreescu e R. Gelca. Mathematical Olympiad Challenges. Birkhauser, Boston,2009.

2. A. Caminha. Topicos de Matematica Elementar, Volume 1: Numeros Reais. Socie-dade Brasileira de Matematica, Rio de Janeiro, 2012.

3. A. Engel. Problem Solving Strategies. Springer-Verlag, Nova Iorque, 1998.

4. E. Lozansky e C. Rousseau. Winning Solutions. Springer-Verlag, Nova Iorque, 1996.

8

Page 20: Álgebra nível 3

POT 2012 - Algebra - Nıvel 3 - Aula 02 - Prof. Antonio Caminha

Dicas e Solucoes

1. Comece obsevando que(

1 + 1n+1

)n+1

(

1 + 1n

)n =

(

1 +1

n+ 1

)(

1− 1

(n+ 1)2

)n

Em seguida, aplique a desigualdade de Bernoulli.

2. Escreva am = mm(

1 + a−mm

)me, em seguida, aplique a desigualdade de Bernoulli.

Faca o mesmo com an e, por fim, some os resultados.

3. Escreva x41 + x42 + · · · + x41994 = x31 · x1 + x32 · x2 + · · · + x31994 · x1994 e, em seguida,aplique a desigualdade de Chebychev para obter

x41 + x42 + · · · + x41994 ≥ 1

1994(x31 + x32 + · · ·+ x31994)(x1 + x2 + · · ·+ x1994).

Por fim, use as equacoes do sistema.

4. Aplique a desigualdade de Chebyshev a cada parcela do somatorio.

5. Adapte a prova da desigualdade de Chebyshev ao caso em questao.

6. Aplique a desigualdade de Chebychev.

7. Aplique a desigualdade de Chebychev para concluir que a expressao do primeiromembro e maior ou igual que

1

4(a3 + b3 + c3 + d3)

(

1

b+ c+ d+

1

a+ c+ d+

1

a+ b+ d+

1

a+ b+ c

)

.

Em seguida, aplique o resultado do Exemplo 4, juntamente com o resultado do Exem-plo 4 da Aula 1, para concluir que a ultima expressao acima e maior ou igual que

(

a+ b+ c+ d

4

)3

· 42

3(a+ b+ c+ d)=

(a+ b+ c+ d)2

12.

Por fim, observe que a condicao do enunciado equivale a (a+ c)(b+ d) = 1 e apliquea desigualdade entre as medias.

8. Adapte a sugestao dada ao problema anterior.

9. Supondo, sem perda de generalidade, x ≥ y ≥ z, use a desigualdade de Chebychevpara concluir que a expressao do enunciado e maior ou igual que

(

x3 + y3 + z3

3

)

· (x+ 1) + (y + 1) + (z + 1)

(x+ 1)(y + 1)(z + 1).

Em seguida, aplique a desigualdade de Chebychev ao primeiro fator e a desigualdadeentre as medias ao segundo fator para concluir que a ultima expressao acima e maiorou igual que 3t3

(t+1)2, onde t = 1

3 (x+y+z). Por fim, use a desigualdade entre as medias

para concluir que t ≥ 1 e, em seguida, prove que t ≥ s ⇒ 3t3

(t+1)2≥ 3s3

(s+1)2.

9

Page 21: Álgebra nível 3

POT 2012 - Algebra - Nıvel 3 - Aula 02 - Prof. Antonio Caminha

10. Para obter a primeira desigualdade, aplique a desigualdade de Chebychev ao primeirosomatorio, seguida do resultado do Exemplo 4 da Aula 1. Para a segunda desigual-dade, utilize a desigualdade entre as medias aritmetica e quadratica (cf. Problema15 da Aula 1).

11. Suponha, sem perda de generalidade, que a ≤ b ≤ c. Basta aplicar a desigualdade dorearranjo, observando que a desigualdade a ser provada e equivalente a

a2bc+ ab2c+ abc2 ≤ (ab)2 + (bc)2 + (ca)2.

12. Aplique a desigualdade do rearranjo.

13. Faca a prova por inducao sobre n > 1 inteiro. Para o passo de inducao, basta provarque

(

1 +a2n

an+1

)(

1 +a2n+1

a1

)

≥(

1 +a2na1

)

(1 + an+1)

ou, equivalentemente, que

a2n+1 ·1

a1+ a2n · 1

an+1≥ an+1 +

a2na1

.

Por fim, observe que tal desigualdade e uma aplicacao imediata da desigualdade dorearranjo.

14. Faca λi = bi/ai, para 1 ≤ j ≤ n, e conclua que

b1 + · · ·+ bn ≥ a1 + · · ·+ an ⇔ a1(λ1 − 1) + a2(λ2 − 1) + · · ·+ an(λn − 1) ≥ 0.

Em seguida, aplique a desigualdade de Abel para concluir que

n∑

j=1

aj(λj − 1) ≥ a1 ·min {λ1 − 1, λ1 + λ2 − 2, . . . , λ1 + · · · + λn − n}.

Por fim, use a condicao do enunciado e a desigualdade entre as medias para mostrarque λ1 + · · ·+ λk ≥ k.

10

Page 22: Álgebra nível 3

Polos Olímpicos de TreinamentoCurso de Álgebra - Nível 3Prof. Antonio Caminha

Aula 3

Funcoes Definidas Implicitamente 1

Esta e a primeira de duas aulas devotadas ao estudo de funcoes definidas implicitamente.Para saber mais sobre o material aqui discutido, remetemos o leitor ao Capıtulo 1 de [2]ou ao Capıtulo 11 de [3].

Uma funcao pode ser definida implicitamente por um conjunto de propriedades. Porexemplo, sendo g(x) = x+ 1 e h(x) = x− 1, a funcao f : R → R dada por f(x) = x2 e talque

f(g(x)) = g(x)2 e f(h(x)) = h(x)2,

ou seja, ela e tal que

f(x+ 1) = (x+ 1)2 = x2 + 2x+ 1 e f(x+ 1) = (x− 1)2 = x2 − 2x+ 1.

Daı, temos que a funcao acima satisfaz, para todo x ∈ R, a relacao

f(x+ 1)− f(x− 1) = 4x.

Podemos tentar reverter os passos acima, perguntando agora quais sao as funcoes f : R → R

tais quef(x+ 1)− f(x− 1) = 4x, ∀ x ∈ R. (1)

E claro que a funcao f(x) = x2 nao e a unica, pois, como e facil verificar, para qualquerconstante real c a funcao fc(x) = x2 + c tambem satisfaz (1).

Como uma funcao f : R → R satisfazendo (1) nao esta dada por seus valores, e sim poruma relacao que deve satisfazer, dizemos que a funcao esta definida implicitamente. Noteque, a partir de (1), podemos descobrir outras relacoes que a funcao satisfaz. Por exemplo,se g : R → R e dada por g(x) = x2, temos

f(g(x) + 1)− f(g(x)− 1) = 4g(x),

ou aindaf(x2 + 1)− f(x2 − 1) = 4x2, ∀ x ∈ R. (2)

Assim, qualquer funcao que satisfizer (1) tambem satisfara (2). Entretanto, a relacao (2)pode nao ser muito util para ajudar a determinar as funcoes f que satisfazem (1). So a

Page 23: Álgebra nível 3

POT 2012 - Algebra - Nıvel 3 - Aula 03 - Prof. Antonio Caminha

experiencia dira que relacoes obtidas a partir de uma relacao inicialmente dada serao uteisnesse sentido.

Em geral, um problema interessante e o de encontrar todas as funcoes definidas impli-citamente por um certo conjunto de relacoes dadas. Uma vez que nao ha uma teoria gerala esse respeito, no que segue, veremos alguns exemplos que ilustram um certo numero detecnicas uteis no trato de funcoes definidas implicitamente.

Exemplo 1 (Canada). Ache todas as funcoes f : N → N, crescentes e tais que f(2) = 2 ef(mn) = f(m)f(n), para todos m,n ∈ N.

Solucao. De 1 ≤ f(1) < f(2) = 2 obtemos f(1) = 1. Agora f(4) = f(2)f(2) = 4 ef(8) = f(4)f(2) = 8. Suponha pois, por hipotese de inducao, que f(2k) = 2k para umcerto natural k. Entao

f(2k+1) = f(2k)f(2) = 2k · 2 = 2k+1,

e segue que f(2n) = 2n para todo inteiro nao negativo n. Portanto, fixado n natural, seguede f ser crescente que

2n = f(2n) < f(2n + 1) < · · · < f(2n+1 − 1) < f(2n+1) = 2n+1.

Mas, uma vez que f(2n +1), f(2n +2), . . ., f(2n+1 − 1) sao naturais, a unica possibilidadee termos

f(2n + 1) = 2n + 1, f(2n + 2) = 2n + 2, . . . , f(2n+1 − 1) = 2n+1 − 1.

Finalmente, como esse raciocınio e valido para todo n natural, segue que f(m) = m paratodo m natural.

Exemplo 2 (Olimp. Iberoamericana). Se D = R − {−1, 0, 1}, encontre todas as funcoesf : D → R tais que, para todo x ∈ D, tenhamos

f(x)2f

(

1− x

1 + x

)

= 64x.

Solucao. Note antes de tudo que, como x 6= 0, temos f(x)2f(

1−x1+x

)

6= 0 para todo x ∈ D.

Em particular, f(x) 6= 0 para todo x ∈ D. Seja agora g(x) = 1−x1+x

para x ∈ D. A definicaode D garante facilmente que g(D) ⊂ D, de modo que podemos compor f com g. Assim,para todo x ∈ D temos

f(g(x))2f

(

1− g(x)

1 + g(x)

)

= 64g(x). (3)

Substituindo a expressao de g na relacao acima, chegamos a

f

(

1− x

1 + x

)

f

(

1− 1−x1+x

1 + 1−x1+x

)

= 64

(

1− x

1 + x

)

,

2

Page 24: Álgebra nível 3

POT 2012 - Algebra - Nıvel 3 - Aula 03 - Prof. Antonio Caminha

ou ainda

f

(

1− x

1 + x

)2

f(x) = 64

(

1− x

1 + x

)

,

para todo x ∈ D. Elevando ao quadrado ambos os membros da relacao do enunciado edividindo o resultado pela relacao acima, vem que

f(x)3 = 64x2(

1− x

1 + x

)

,

e daı f(x) = 4 3

x2(

1−x1+x

)

.

Ate este ponto, mostramos apenas que, se f existir, deve ser dada por essa expressao.Temos, pois, de verificar que f , assim definida, realmente satisfaz a relacao do enunciadopara todo x ∈ D. Mas tal verificacao e imediata e sera deixada a cargo do leitor.

Ainda em relacao ao exemplo anterior, com um pouco mais de pratica poderıamosprescindir de definir a funcao g para em seguida compo-la com f a fim de obter (3). Aoinves disso, poderıamos apenas ter dito

Substituindo x por 1−x1+x

na relacao do enunciado, obtemos . . . ,

tendo em mente que essa substituicao e meramente uma composicao de funcoes. Doravante,sempre que nao houver perigo de confusao, adotaremos essa simplificacao de linguagem, aqual ja aparece no exemplo a seguir. Ao le-lo, tente identificar as composicoes que forammascaradas por substituicoes.

Exemplo 3. Encontre todas as funcoes f : R → R tais que f(1) = 1 e, para todos x, y ∈ R,tenhamos

(a) f(x+ y) = f(x) + f(y).

(b) f(xy) = f(x)f(y).

Solucao. Seja f uma funcao satisfazendo as condicoes do enunciado. Fazendo x = y = 0em (a), obtemos

f(0) = f(0 + 0) = f(0) + f(0) = 2f(0),

donde segue que f(0) = 0. Fazendo y = x em (a), obtemos

f(2x) = f(x+ x) = f(x) + f(x) = 2f(x)

para todo x ∈ R. Fazendo agora y = 2x em (a), segue que

f(3x) = f(x+ 2x) = f(x) + f(2x) = f(x) + 2f(x) = 3f(x)

para todo x ∈ R. Repetindo o argumento acima concluımos, por inducao sobre n ∈ N, que

f(nx) = nf(x), ∀ n ∈ N, x ∈ R (4)

3

Page 25: Álgebra nível 3

POT 2012 - Algebra - Nıvel 3 - Aula 03 - Prof. Antonio Caminha

Em particular, fazendo x = 1 em (4), obtemos f(n) = n para todo n ∈ N. Fazendo agorax = 1

nem (4), segue que

1 = f(1) = f

(

n · 1n

)

= nf

(

1

n

)

,

donde f(

1n

)

= 1n. Finalmente, x = 1

mem (4), com m ∈ N, fornece

f( n

m

)

= f

(

n · 1

m

)

= nf

(

1

m

)

= n · 1

m=

n

m.

Vamos ver o que ocorre com os racionais negativos. Para isso, facamos y = −x no item(a), obtendo

0 = f(0) = f(x+ (−x)) = f(x) + f(−x),

ou aindaf(x) = −f(−x), ∀ x ∈ R. (5)

Em particular, sendo x < 0 racional, segue de (5) e do fato de ser −x um racional positivoque f(x) = −f(−x) = −(−x) = x, e portanto f(x) = x para todo x ∈ Q.

Como f(x) = x para todo x ∈ Q, desconfiamos que a funcao identidade seja a unicasatisfazendo as condicoes do enunciado. Para confirmar tal suposicao, voltemos nossaatencao a condicao do item (b). Inicialmente, mostremos que se para um certo x ∈ R

tivermos f(x) = 0, entao x = 0. De fato, caso fosse x 6= 0, fazendo y = 1xem (b) terıamos

0 = f(x)f

(

1

x

)

= f

(

x · 1x

)

= f(1) = 1,

o que e uma contradicao. Agora, fazendo y = x 6= 0 em (b), obtemos

f(x2) = f(x · x) = f(x) · f(x) = f(x)2 > 0; (6)

portanto, se x, y ∈ R, com x < y, e a 6= 0 for tal que y − x = a2, entao, aplicandosucessivamente (a), (5) e (6), obtemos

f(y)− f(x) = f(y) + f(−x) = f(y − x) = f(a2) = f(a)2 > 0,

de sorte que f e crescente. Suponha, por fim, que existe a ∈ R tal que f(a) < a e tome (cf.Problema 1.5.2 de [1]) um racional r tal que f(a) < r < a; o carater crescente de f fornece

r = f(r) < f(a),

o que e uma contradicao. Analogamente, nao podemos ter f(a) > a, e a unica possibilidadee f(a) = a. Mas como a ∈ R foi escolhido arbitrariamente, devemos ter f(x) = x paratodo x ∈ R.

Para o proximo exemplo precisamos da seguinte

4

Page 26: Álgebra nível 3

POT 2012 - Algebra - Nıvel 3 - Aula 03 - Prof. Antonio Caminha

Definicao 4. Se X e um conjunto nao vazio e f : X → X e uma funcao dada, um elemento

x0 ∈ X e dito um ponto fixo de f se f(x0) = x0.

Se I ⊂ R e um intervalo, uma funcao decrescente f : I → I admite no maximo umponto fixo. De fato, se x1, x2 ∈ I fossem pontos fixos de f , com x1 < x2, seguiria de f serdecrescente que

x1 = f(x1) > f(x2) = x2,

uma contradicao a hipotese x1 < x2.

Exemplo 5 (Argentina). Seja f : R → R uma funcao decrescente e tal que f(x+ f(x)) =x+ f(x) para todo real x. Prove que f(f(x)) = x para todo real x.

Prova. As hipoteses sobre f garantem que x + f(x) e ponto fixo de f para todo x ∈ R.Por outro lado, o carater decrescente de f garante, de acordo com a discussao anterior, aexistencia de no maximo um ponto fixo para f , de sorte que deve existir a ∈ R tal quex+f(x) = a para todo x ∈ R, o que e o mesmo que f(x) = a−x para todo x ∈ R. Portanto

f(f(x)) = f(a− x) = a− (a− x) = x,

para todo x ∈ R.

Problemas

1. Encontre todas as funcoes f : Q → Q tais que f(

x+y2

)

= f(x)+f(y)2 , para todos

x, y ∈ Q.

2. (Austria). Encontre todas as funcoes f : Z\{0} → Q tais que, para todos x, y ∈ Z\{0}para os quais x+ y seja multiplo de 3, tenhamos

f

(

x+ y

3

)

=f(x) + f(y)

2.

3. (Vietna). Ache todas as funcoes f : R → R tais que

1

2f(xy) +

1

2f(xz)− f(x)f(yz) ≥ 1

4,

para todos x, y, z ∈ R.

4. (Espanha). Encontre todas as funcoes crescentes f : N → N tais que, para todon ∈ N, tenhamos f(n+ f(n)) = 2f(n).

5. Encontre todas as funcoes f : R → Z tais que:

(a) f(x+ a) = f(x) + a, para todo x ∈ R e todo a ∈ Z.

5

Page 27: Álgebra nível 3

POT 2012 - Algebra - Nıvel 3 - Aula 03 - Prof. Antonio Caminha

(b) f(f(x)) = 0 para x ∈ [0, 1).

6. (Austria-Polonia). Prove que nao existe funcao f : Z → Z tal que, para todos x, y ∈ Z,tenhamos f(x+ f(y)) = f(x)− y.

7. (Romenia). Ache todas as funcoes f : Z → Z tais que f(0) = 1 e

f(f(k)) + f(k) = 2k + 3

para todo k ∈ Z.

8. (Romenia). Sejam k > 1 um inteiro ımpar e A = {x1, x2, . . . , xk} um conjunto de k

numeros reais. Obtenha todas as funcoes injetivas f : A → A tais que

|f(x1)− x1| = |f(x2)− x2| = · · · = |f(xk)− xk|.

9. Ache todas as funcoes f : Q → Q∗+ tais que f(x+ y) = f(x)f(y) para todos x, y ∈ Q.

10. Ache todas as funcoes f : [0, 1] → [0, 1] tais que f(0) = 0, f(1) = 1 e

f(x+ y) + f(x− y) = 2f(x)

para todos os x, y tais que x− y, x+ y ∈ [0, 1].

Bibliografia

1. A. Caminha. Topicos de Matematica Elementar, Volume 1: Numeros Reais. Socie-dade Brasileira de Matematica, Rio de Janeiro, 2012.

2. A. Caminha. Topicos de Matematica Elementar, Volume 3: Introducao a Analise.Sociedade Brasileira de Matematica, Rio de Janeiro, 2012.

3. A. Engel. Problem Solving Strategies. Springer-Verlag, Nova Iorque, 1998.

6

Page 28: Álgebra nível 3

POT 2012 - Algebra - Nıvel 3 - Aula 03 - Prof. Antonio Caminha

Dicas e Solucoes

1. Fazendo g(x) = f(x) − f(0) na relacao do enunciado, conclua que basta considerar

o caso em que f(0) = 0. Sob tal suposicao, faca y = 0 para obter f(

x2

)

= f(x)2 para

todo x ∈ Q. Em seguida, use a relacao do enunciado para concluir que f(x + y) =f(x) + f(y) para todos x, y ∈ Q, de sorte que f(x) = f(1)x para todo x ∈ Q.

2. Use a relacao do enunciado para provar por inducao que f(n) = f(1) para todo n ∈ N;em seguida, considere o caso n < 0.

3. Faca x = y = z = 0 para obter f(0) = 12 ; em seguida, obtenha f(1) = 1

2 de maneiraanaloga. Faca y = z = 1 para concluir que f(x) ≥ 1

2 para todo x ∈ R. Por fim, utilizeuma substituicao analoga para concluir que f(x) ≤ 1

2 para todo x ∈ R.

4. Em cada um dos intervalos [n, n + f(n)] e [f(n), 2f(n)] ha f(n) + 1 naturais. Use,pois, o carater crescente de f , juntamente com f(n + f(n)) = 2f(n), para concluirque f(n + k) = f(n) + k, para todos 1 ≤ k ≤ f(n). Tome agora k, n naturais, comn > k. Use que f(n) ≥ n para mostrar que f(n) > k e daı, pelo que fizemos acima,que f(n) = n− 1 + f(1).

5. Faca x = a = 0 em (a) para concluir que f(0) = 0. Em seguida, fazendo x = 0em (a), mostre que f(a) = a para todo a ∈ Z. Tomando agora x ∈ [0, 1), use quef(x) ∈ Z e f(f(x)) = 0 para concluir que f(x) = 0. Por fim, para x ∈ R qualquer,troque x por {x} e faca a = ⌊x⌋ em (a) para concluir que f(x) = ⌊x⌋.

6. Comece calculando f(x+ f(y + f(0))) de duas maneiras distintas para concluir quef(0) = 0. Em seguida, deduza a partir daı que f(f(y)) = −y para todo y ∈ Z, de sorteque f e bijetiva. A partir dessa ultima relacao, calcule f(f(f(x))) de duas maneirasdistintas para concluir que f e ımpar. Troque x por f(x) na relacao do enunciadopara concluir que f(f(x) + f(y)) = f(f(x + y)), e daı que f(x) + f(y) = f(x + y)para todos x, y ∈ Z. Por fim, use inducao para obter f(x) = f(1)x para todo x ∈ Z,chegando assim a contradicao −1 = f(f(1)) = f(1)f(1) ≥ 0.

7. Faca k = 0 para concluir que f(1) = 2; em seguida, use inducao para mostrar quef(n) = n + 1 para todo n ∈ N. Use agora a relacao do enunciado para mostrar quese f(−1) = a, entao nao pode ser a < 0 nem a > 0. Por fim, faca uma nova inducaopara concluir que f(−n) = −n+ 1 para todo n ∈ N.

8. Inicialmente, podemos supor que f nao tem pontos fixos. Escreva agora A = B ∪C,onde B e C sao conjuntos disjuntos e tais que f(x) > x para todo x ∈ B e f(x) < x

para todo x ∈ C. Se |B| = l e a e o valor comum de |f(xj) − xj |, conclua que

0 =∑k

j=1(f(xj)−xj) = (2l−k)a e, a partir daı, que a = 0, o que e uma contradicao.

9. Faca x = y = 0 para obter f(0) = 1. Em seguida, use inducao para provar quef(nx) = f(x)n para todos x ∈ Q, n ∈ N; mostre a partir daı que f(x) = f(1)x paratodo x ∈ Q

∗+. Fazendo agora y = −x, mostre que f(−x) = f(x)−1; conclua entao

que f(x) = f(1)x para todo x ∈ Q. Por fim, use o fato de o contradomınio de f sero conjunto dos racionais positivos para concluir que f(1) = 1.

7

Page 29: Álgebra nível 3

POT 2012 - Algebra - Nıvel 3 - Aula 03 - Prof. Antonio Caminha

10. Fazendo x + y = a e x − y = b, mostre que f(a) + f(b) = 2f(

a+b2

)

, para todosa, b ∈ [0, 1]. Fazendo x = y, mostre que f(2x) = 2f(x) para todo x ∈ [0, 1], e concluaque f(a)+ f(b) = f(a+ b), para todos a, b ∈ [0, 1]. Mostre a partir daı que f(x) = x,para todo x ∈ [0, 1] ∩ Q. Para o que falta, imite a sugestao para a passagem de Q aR, no Exemplo 3.

8

Page 30: Álgebra nível 3

Polos Olímpicos de TreinamentoCurso de Álgebra - Nível 3Prof. Antonio Caminha

Aula 4

Funcoes Definidas Implicitamente 2

Nesta segunda aula sobre funcoes definidas implicitamente, examinamos e propomosalguns exemplos mais elaborados. Para muitos mais, remetemos o leitor a [3] ou ao Capıtulo11 de [4].

Comecamos observando que, por vezes, o que se pede nao e obter todas as funcoes defi-nidas implicitamente por certas relacoes, mas somente deduzir que tais funcoes satisfazemcertas propriedades ou que, ao contrario, nao existem. Vejamos alguns exemplos.

Exemplo 1. Sejam c e α reais positivos dados e Q um quadrado no plano, tambem dado.Prove que nao existe funcao sobrejetiva f : [0, 1] → Q tal que, para todos 0 ≤ x, y ≤ 1,tenhamos

|f(x)− f(y)| ≤ c|x− y|α+1/2.

Prova. Suponha que exista uma tal funcao. Sejam k > 1 inteiro e xj =jk, para 0 ≤ j ≤ k.

Dado um real x ∈ [xj, xj+1], temos

|f(x)− f(xj)| ≤ c|x− xj|α+1/2 ≤ c

kα+1/2.

Portanto, sendo Cj o cırculo do plano com centro f(xj) e raio c

kα+1/2 , segue que

x ∈ [xj , xj+1] ⇒ f(x) ∈ Cj .

EntaoQ = f([0, 1]) ⊂ C0 ∪ C1 ∪ · · · ∪ Ck−1,

o que implica (A(F ) denota a area da figura F )

1 = A(Q) = A(

∪k−1j=0Cj

)

≤k−1∑

j=0

A(Cj)

= kπ( c

kα+1/2

)2=

πc2

k2α.

Mas, como α > 0 e k > 1 e arbitrario, chegamos a uma contradicao, uma vez que a relacaok2α ≤ πc2 nao e verdadeira para k suficientemente grande.

Page 31: Álgebra nível 3

POT 2012 - Algebra - Nıvel 3 - Aula 04 - Prof. Antonio Caminha

Exemplo 2 (Franca). Seja f : N → N uma bijecao. Prove que existem naturais a < b < c,tais que f(a) + f(c) = 2f(b)

Prova. Se g = f−1, o problema equivale a provarmos a existencia de tres inteiros positivosx < y < z, em PA e tais que g(x) < g(y) < g(z). Faca x = 1 e seja α = g(1). Defina

A = {g−1(1), g−1(2), . . . , g−1(α− 1), g−1(α)}.

Se t ∈ N \ A, entao g(t) > α = g(1) = g(x). Portanto, basta provarmos que existem doisinteiros y < z em N \ A que formam uma PA com 1 e sao tais que g(y) < g(z).

Para o que falta, fixado t ∈ N \ A, considere os inteiros

t, 2t− 1, 4t− 3, 8t− 7, . . . , 2kt− (2k − 1), . . . .

Quaisquer dois elementos consecutivos dessa sequencia de numeros formam uma PA com1, de modo que, se existir k tal que

g(2kt− (2k − 1)) < g(2k+1t− (2k+1 − 1)),

nada mais havera a fazer. Mas, se este nao fosse o caso, a injetividade de g nos daria

g(t) > g(2t − 1) > g(4t− 3) > g(8t − 7) > · · · > g(1),

o que seria um absurdo.

Voltemo-nos, agora, a exemplos mais difıceis onde se pede, efetivamente, obter todasas funcoes que satisfazem certos conjuntos de condicoes.

Exemplo 3 (Ira). Obtenha todas as funcoes f : R → R tais que, para todos os x, y reais,tenhamos

f(f(x+ y)) = f(x+ y) + f(x)f(y)− xy.

Solucao. Seja f uma funcao satisfazendo as condicoes do enunciado. Fazendo y = 0,obtemos

f(f(x)) = f(x) + f(0) · f(x), ∀x ∈ R. (1)

Seja Im(f) a imagem de f e c = f(0). Segue de (1) que

f(z) = (c+ 1)z, ∀ z ∈ Im(f). (2)

Agora, como f(x+ y) ∈ Im(f), para todos x, y ∈ R, segue de (2) e da relacao originalque

(c+ 1)f(x+ y) = f(x+ y) + f(x)f(y)− xy

ou, ainda,c · f(x+ y) = f(x)f(y)− xy, ∀x ∈ R. (3)

Suponha que c 6= 0. Entao 0 /∈ Im(f). De fato, caso fosse 0 ∈ Im(f) terıamos de (2)que f(0) = 0, uma contradicao. Mas aı, pondo x = c, y = −c em (3), obterıamos

c2 = c · f(0) = f(c)f(−c) + c2,

2

Page 32: Álgebra nível 3

POT 2012 - Algebra - Nıvel 3 - Aula 04 - Prof. Antonio Caminha

donde f(c)f(−c) = 0 e, portanto, f(c) = 0 ou f(−c) = 0; isto, por sua vez, contradiria asuposicao de que 0 /∈ Im(f). Logo, c = 0.

Segue, pois, de (3), quef(x)f(y) = xy, ∀x ∈ R. (4)

Como a funcao identicamente nula nao satisfaz as condicoes do enunciado, existe z0 ∈Im(f) tal que z0 6= 0. Entao, temos a partir de (2) que f(z0) = z0. Assim, fazendo y = z0em (4), obtemos f(x) · z0 = xz0, para todo x ∈ R, donde f(x) = x para todo x ∈ R.

Por fim, e imediato verificar que a funcao identidade satisfaz a condicao do enunciado.

Antes de apresentar o proximo exemplo, precisamos enunciar um resultado que se revelaimportante tambem em outros contextos. Para tanto, recordamos o leitor de que umafuncao polinomial e uma funcao f : R → R da forma

f(x) = anxn + an−1x

n−1 + · · ·+ a1x+ a0, (5)

para todo x ∈ R, onde n e um inteiro nao negativo dado e a0, . . . , an sao numeros reaistambem dados.

Teorema 4. Seja f : R → R uma funcao polinomial. Se a, b e d sao numeros reais tais

que a < b e (f(a)− d)(f(b) − d) < 0, entao existe a < c < b tal que f(c) = d.

O teorema acima e conhecido em Matematica como o teorema do valor inter-

mediario. (Abreviamos TVI.) Intuitivamente, sua validade decorre do fato de que, ografico de uma funcao polinomial nao tem saltos; portanto, se ele passar pelos pontos(a, f(a)) e (b, f(b)), e d pertencer ao intervalo de extremos f(a) e f(b) (que e o que garantea condicao (f(a) − d)(f(b) − d) < 0), entao tal grafico deve intersectar a reta y = d ou, oque e o mesmo, deve existir a < c < b tal que f(c) = d.

O TVI, de fato, pode ser aplicado a uma classe de funcoes bem mais ampla que aqueladas funcoes polinomiais, qual seja, aquela das funcoes contınuas; intuitivamente, o leitorpode pensar numa funcao contınua f : I → R (I ⊂ R um intervalo), como uma funcaocujo grafico nao admite interrupcoes. Para uma discussao mais adequada desse conceito,juntamente com uma prova do TVI para tal classe de funcoes, remetemos o leitor a Secao4.2 de [1]. Tambem, uma prova do TVI para funcoes polinomiais, baseada na validade doteorema fundamental da Algebra, pode ser encontrada na Secao 5.1 de [2].

Precisamos, ainda, da seguinte consequencia do TVI.

Corolario 5. Se f : R → R e uma funcao polinomial como em (5), com n ımpar, entao a

imagem de f e todo o conjunto dos numeros reais.

Prova. Dado d ∈ R, faca g(x) = f(x)− d. Entao g : R → R tambem e polinomial, e bastagarantirmos a existencia de c ∈ R tal que g(c) = 0.

O argumento do paragrafo anterior reduz nosso problema a mostrar a existencia dec ∈ R tal que f(c) = 0. Para tanto, suponhamos, sem perda de generalidade, que an > 0.

3

Page 33: Álgebra nível 3

POT 2012 - Algebra - Nıvel 3 - Aula 04 - Prof. Antonio Caminha

Entao, para x 6= 0, repetidas aplicacoes da desigualdade triangular fornecem

f(x)

xn= an +

an−1

x+ · · ·+ a1

xn−1+

a0

xn

≥ an −∣

an−1

x+ · · · + a1

xn−1+

a0

xn

≥ an −∣

an−1

x

∣− · · · −∣

a1

xn−1

∣−∣

a0

xn

= an − |an−1||x| − · · · − |a1|

|x|n−1− |a0|

|x|n .

Portanto, se |x| ≥ 1, entao |x| ≤ |x|2 ≤ · · · ≤ |x|n e, daı,

f(x)

xn≥ an − 1

|x|

n−1∑

j=0

|aj |,

o qual, por sua vez, e positivo para |x| > 1an

∑n−1j=0 |aj |.

Em resumo, se

A > max

1,1

an

n−1∑

j=0

|aj |

,

entao f(x)xn > 0 para x = ±A. Mas, como n ımpar, segue que f(−A) < 0 < f(A) e o TVI

garante a existencia de c ∈ [−A,A] tal que f(c) = 0.

Podemos, por fim, discutir o exemplo a seguir.

Exemplo 6 (Bielorrussia). Encontre todas as funcoes f, g, h : R → R tais que, para todosx, y ∈ R, tenhamos

f(x+ y3) + g(x3 + y) = h(xy).

Solucao. Fazendo x = y = 0, obtemos f(0) + g(0) = h(0). Assim, definindo f1(x) =f(x)− f(0), g1(x) = g(x)− g(0) e h1(x) = h(x)− h(0), temos

f1(x+ y3) + g1(x3 + y) = h1(xy),

com f1(0) = g1(0) = h1(0) = 0. Podemos, portanto, comecar supondo que f(0) = g(0) =h(0) = 0.

Fazendo y = −x3 na relacao do enunciado, obtemos g(x − x9) = h(−x4) para todox. Fazendo x = −y3, obtemos f(−y9 + y) = h(−y4), para todo y ∈ R; daı, segue quef(x− x9) = g(x − x9). Mas a imagem da funcao polinomial x 7→ x− x9 e o conjunto dosnumeros reais, de modo que f(x) = g(x) para todo x ∈ R. A relacao do enunciado sereduz, entao, a

f(x+ y3) + f(x3 + y) = h(xy). (6)

Fazendo sucessivamente y = −x3 e x = −y3 em (6), e levando em conta que f(0) = 0,obtemos, respectivamente,

f(x− x9) = h(−x4) e f(−y9 + y) = h(−y4),

4

Page 34: Álgebra nível 3

POT 2012 - Algebra - Nıvel 3 - Aula 04 - Prof. Antonio Caminha

de modo que f(x− x9) = f(−x9 + x). Usando de novo o fato de que a imagem da funcaopolinomial x 7→ x− x9 e R, segue que f e uma funcao par, i.e., tal que que f(x) = f(−x),para todo x ∈ R.

Fazendo, agora, y = 0 em (6), obtemos f(x) + f(x3) = 0, donde f(x) = −f(x3).Voltando a (6), essa relacao nos da

f(x3 + y)− f((x+ y3)3) = h(xy).

Mas f par implica, entao, que

f(x3 + y)− f(−(x+ y3)3) = h(xy).

Segue, daı, que, se a ∈ R for tal que existam reais x, y satisfazendo o sistema de equacoes

{

xy = a

x3 + y = −(x+ y3)3,

entao h(a) = 0. Provemos que esse e o caso para todo a ≤ 0.Se a = 0, ja temos h(a) = 0. Se a 6= 0 entao a segunda equacao do sistema acima

equivale a y9+3y6x+3x2y3+y+2x3 = 0. Escrevendo x = ya, segue que basta verificarmos

a existencia de um real y tal que

y12 + 3ay8 + (3a2 + 1)y4 + 2a3 = 0.

Para tanto, seja p(y) = y12 + 3ay8 + (3a2 + 1)y4 + 2a3. Se a < 0, entao p(0) = 2a3 < 0 ep(−a) > 0, de modo que o TVI garante que p tem ao menos uma raiz real.

Para terminarmos, veja que f(x−x9) = h(−x4), de modo que f(x−x9) = 0 para todox ∈ R. Usando novamente a sobrejetividade de f , segue que f ≡ 0. Daı, (6) nos da h ≡ 0.

Entao, as funcoes f, g, h que satisfazem as condicoes do enunciado sao as funcoes cons-tantes f ≡ f(0), g ≡ g(0), h ≡ h(0), tais que f(0) + g(0) = h(0).

Problemas

1. (IMO). Seja G um conjunto nao vazio de funcoes afins, possuindo as seguintes pro-priedades:

(a) Se f, g ∈ G, entao f ◦ g ∈ G.

(b) Se f ∈ G, entao f−1 ∈ G.

(c) Para toda f ∈ G, existe xf ∈ R tal que f(xf ) = xf .

Prove que existe um real x0 tal que f(x0) = x0, para toda f ∈ G.

5

Page 35: Álgebra nível 3

POT 2012 - Algebra - Nıvel 3 - Aula 04 - Prof. Antonio Caminha

2. (IMO). Seja f : R → [0, 1] uma funcao tal que, para um certo a ∈ R, tenhamos

f(x+ a) =1

2+

f(x)− f(x)2,

para todo x ∈ R.

(a) Prove que f e periodica, i.e., que existe um real p > 0 tal que f(x+ p) = f(x),para todo x ∈ R.

(b) Para a = 1, de um exemplo de uma funcao nao constante f : R → [0, 1] quesatisfaca a relacao do enunciado.

3. (Lituania). Seja f : Z → Z uma funcao tal que f(m2+ f(n)) = f(m)2+n para todosm,n inteiros.

(a) Prove que f(0) = 0 e f(1) = 1.

(b) Ache todas essas funcoes.

4. (OIM). Encontre todas as funcoes f : N → N satisfazendo, para todos x, y ∈ N, asduas condicoes a seguir:

(a) x > y ⇒ f(x) > f(y).

(b) f(yf(x)) = x2f(xy)

5. (Bulgaria). Sejam n > 1 inteiro e a1, a2, . . . , an reais positivos dados. Prove que aequacao √

1 + a1x+√1 + a2x+ · · ·+

√1 + anx = nx

possui exatamente uma solucao real positiva

6. (Romenia – adaptado). Existe uma funcao polinomial f : R → R tal que

f(x) ∈ Q ⇔ f(x+ 1) /∈ Q?

7. (IMO). Ache todas as funcoes f : [0,+∞) → R satisfazendo as duas condicoes aseguir:

(a) f(xf(y))f(y) = f(x+ y), para todos x, y ∈ [0,+∞).

(b) f(2) = 0 e f(x) 6= 0 para 0 ≤ x < 2.

8. (IMO). Seja S = {x ∈ R; x > −1}. Obtenha todas as funcoes f : S → S quesatisfacam as duas condicoes a seguir:

(a) f(x+ f(y) + xf(y)) = y + f(x) + yf(x)), para todos x, y ∈ S.

(b) f(x)x

e crescente em cada um dos intervalos (−1, 0) e (0,+∞).

9. (Polonia). Encontre todas as funcoes f : Q∗+ → Q

∗+ satisfazendo, para todo racional

positivo x, as seguintes condicoes:

6

Page 36: Álgebra nível 3

POT 2012 - Algebra - Nıvel 3 - Aula 04 - Prof. Antonio Caminha

(a) f(x+ 1) = f(x) + 1.

(b) f(x3) = f(x)3.

10. (IMO). Decida se existe uma funcao f : N → N satisfazendo as condicoes a seguir:

(a) f(1) = 2.

(b) f(n) < f(n+ 1) para todo n ∈ N.

(c) f(f(n)) = f(n) + n para todo n ∈ N.

Bibliografia

1. A. Caminha. Topicos de Matematica Elementar, Volume 3: Introducao a Analise.Sociedade Brasileira de Matematica, Rio de Janeiro, 2012.

2. A. Caminha. Topicos de Matematica Elementar, Volume 6: Introducao a Analise.Sociedade Brasileira de Matematica, Rio de Janeiro, 2012.

3. D. Djukic, V. Jankovic, I. Matic e N. Petrovic. The IMO Compendium: a Collection

of Problems Suggested for The International Mathematical Olympiads from 1959-

2004. Springer-Verlag, Nova Iorque, 2006.

4. A. Engel. Problem Solving Strategies. Springer-Verlag, Nova Iorque, 1998.

7

Page 37: Álgebra nível 3

POT 2012 - Algebra - Nıvel 3 - Aula 04 - Prof. Antonio Caminha

Dicas e Solucoes

1. Inicialmente, mostre que basta provar que duas funcoes quaisquer em G comutamem relacao a operacao de composicao de funcoes. Para tanto, use os itens (a) e (b),juntamente com o fato de que a unica funcao h ∈ G da forma h(x) = x + a, paraalgum a ∈ R, e h = IdR.

2. Para o item (a), fazendo g(x) = f(x) + 12 , mostre que g(x+ a) =

14 − g(x)2 e, daı,

que g(x + 2a) = g(x), para todo x ∈ R. Quanto a (b), faca h(x) = 4g(x)2 − 12 e

conclua que h(x+ 1) = −h(x); em seguida, mostre que h(x) = sen 2(

πx2

)

− 12 e uma

possibilidade.

3. Para o item (a), comece fazendo m = 0 na relacao do enunciado para obter f(f(n)) =f(0)2 + n, concluindo, assim, que f e bijetiva. Em seguida, tome k ∈ Z tal quef(k) = 0 e seja l = f(0); entao temos l = f(0) = f(f(k)) = f(0)2 + k = l2 + k,ao passo que, fazendo m = k e n = 0 na relacao do enunciado, temos k2 + l = k.Logo, k = l = 0. Quanto a (b), faca m = 1 e n = 0 na relacao do enunciado paraobter f(1) = 1; em seguida, deduza que f(f(n) + 1) = n + 1 e, a partir daı, quef(n) = n para todo inteiro nao negativo n. Por fim, estenda o argumento aos inteirosnegativos, mostrando que f(n) = n para todo n ∈ Z.

4. Fazendo x = y = 1, conclua que f(1) = 1. Em seguida, mostre que f(f(xy)) =x2y2f(xy) = f(f(x)f(y)), obtendo, entao, que f(xy) = f(x)f(y), para todos x, y ∈ N;conclua a partir daı, que f(x2) = f(x)2 para todo x ∈ N. Por fim, se f(x) < x2 paraum certo x ∈ N, escreva f(x)3 = f(x3) > f(xf(x)) = x2f(x)2 para obter f(x) > x2,uma contradicao; analogamente, mostre que nao pode ser f(x) > x2, de sorte quef(x) = x2 para todo x ∈ N.

5. Divida ambos os membros da equacao por x e, em seguida, aplique o TVI.

6. Comece observando que a funcao g : R → R, dada por g(x) = f(x + 1) − f(x) econtınua e, pela condicao do enunciado, transforma todo numero real num irracional.Agora, use o TVI, juntamente com o fato de que todo intervalo nao degeneradocontem numeros irracionais, para concluir que g e constante; entao, use isto paramostrar que f(x+ 2) − f(x) e sempre irracional. Por fim, mostre que existe x0 ∈ R

tal que f(x0) ∈ Q e, daı, que f(x0 + 2)− f(x0) ∈ Q.

7. Troque x por x − 2 e y por 2 em (a) para concluir que f(x) = 0 para x ≥ 2. Faca,agora, x = y = 0 em (a) para obter, a partir de (b), que f(0) = 1. Por fim, para0 < x < 2, troque y por 2− x em (a) para concluir que f(x) ≥ 2

2−x; por outro lado,

mostre que f(

x+ 2f(x)

)

= f(2)f(x) = 0 e conclua que f(x) ≤ 22−x

para cada um de

tais x.

8. Observe inicialmente que, a partir de (b), f tem no maximo um ponto fixo em cada umdos intervalos (−1, 0), (0,+∞). Se existir um ponto fixo x0 de f tal que −1 < x0 < 0,faca x = y = x0 em (a) para concluir que x20 + 2x0 tambem e ponto fixo de f , e daı

8

Page 38: Álgebra nível 3

POT 2012 - Algebra - Nıvel 3 - Aula 04 - Prof. Antonio Caminha

que x20 +2x0 = x0, o que e uma contradicao. Argumente analogamente para concluirque f nao tem pontos fixos em (0,+∞). Por fim, fazendo x = y em (a), conclua quef(x) = − x

x+1 para todo x ∈ S.

9. Observe que, por (a), temos f(x + k) = f(x) + k, para todo k ∈ N. Tome, agora,

m,n ∈ N e use que (n2+m)3−n3

n3 ∈ N para calcular

f

(

m3

n3+

(n3 +m)3 − n3

n3

)

de duas maneiras distintas, obtendo a equacao

(a+ n2)3 = a3 + (n6 + 3n3m+ 3m2),

onde a = f(

mn

)

. Conclua, a partir daı, que f(x) = x para todo x ∈ Q∗+.

10. Considere a sequencia de Fibonacci (Fn)n≥1, dada por F1 = 1, F2 = 1 e Fk+2 =Fk+1 + Fk, para todo inteiro k ≥ 1. Comece mostrando, por inducao, o teorema

de Zeckendorff: todo numero natural pode ser unicamente escrito como soma denumeros de Fibonacci de ındices maiores que 1 e nao consecutivos. Voltando aoproblema em questao, observe que, se uma tal f existir, entao f(Fk) = Fk+1 para todok ∈ N. Agora, utilize o teorema de Zeckendorff para mostrar que uma possibilidadepara f e definirmos

f(Fi1 + Fi2 + · · ·+ Fij ) = Fi1+1 + Fi2+1 + · · · + Fij+1,

para todos 1 ≤ i1 < i2 < · · · < ij naturais.

9

Page 39: Álgebra nível 3

Polos Olímpicos de TreinamentoCurso de Álgebra - Nível 3Prof. Cícero Thiago / Prof. Marcelo Mendes

Aula 6

Numeros Complexos

Definicao 1

O conjunto dos numeros complexos, representado por C, consiste de todos os pares orde-nados (a, b) com a, b ∈ R. Definiremos que (a, b) + (c, d) = (a+ c, b+ d) e (a, b) · (c, d) =(ac− bd, ad+ bc). Chamaremos de unidade imaginaria o complexo i = (0, 1). Usando aoperacao de multiplicacao verificamos que i·i = i2 = (0, 1)·(0, 1) = (0 · 0− 1 · 1, 0 · 1 + 1 · 0) =(−1, 0). Vamos fazer um abuso de linguagem matematica e definir que (−1, 0) = −1, sendoassim definiremos os numeros complexos como as expressoes da forma z = a+ bi, onde a eb sao numeros reais e i2 = −1, ou seja,

C ={

z = a+ bi| a, b ∈ R, i2 = −1.}

O numero a e chamado de parte real de z e o numero b e chamado de parte imaginariade z. Com isso, fica facil perceber que se um numero complexo que tem parte imaginariaigual a zero sera um numero real.

Dois numeros complexos sao iguais se, e somente se, eles possuem a mesma parte real e amesma parte imaginaria, isto e,

a+ bi = c+ di ⇔ a = c e b = d.

A soma e o produto de dois numeros complexos z = a+ bi e w = c+ di sao definidos assim:

z +w = (a+ c) + (b+ d)i e z.w = (ac− bd) + (ad+ bc)i.

O numero complexo z = a−bi sera chamado de conjugado do numero complexo z = a+bi.E facil perceber que z e um numero real se, e somente se, z = z. Usaremos o conjugado do

numero complexo z = a+ bi para poder representar o complexo1

z:

1

z=

1

a+ bi=

1

a+ bi.a− bi

a− bi=

a− bi

a2 + b2=

z

|z|2 =a

a2 + b2+

−b

a2 + b2i.

A partir do resultado acima faremosz1

z2= z1.

1

z2, onde z1 e z2 sao numeros complexos.

Vejamos abaixo algumas propriedades as quais deixaremos as provas com o leitor:

Page 40: Álgebra nível 3

POT 2012 - Algebra - Nıvel 3 - Aula 5 - Prof. Marcelo Mendes/ Prof. Cıcero

Thiago

1. z = z

2. z + w = z + w

3. z.w = z.w

4.( z

w

)

=z

w.

O valor absoluto, ou modulo de um numero complexo z = a+ bi e definido por:

|z| =√

a2 + b2 =√z.z.

Vejamos agora algumas propriedades do modulo de um numero complexo:1. |z| = | − z| = |z|2. z.z = |z|23. |z1.z2| = |z1|.|z2|4.

1

z

=1

|z| , z 6= 0

5.

z1

z2

=|z1||z2|

, z2 6= 0

6. |z1 + z2| ≤ |z1|+ |z2|Prova:

Observe que |z1+z2|2 = (z1 + z2) (z1 + z2) = (z1 + z2) (z1 + z2) = |z1|2+z1·z2+z1·z2+|z2|2.Por outro lado, z1 · z2 = z1 · z2 = z1 · z2, assim:

z1 · z2 + z1 · z2 = 2 · Re (z1 · z2) ≤ 2|z1 · z2| = 2|z1| · |z2|,

portanto, |z1 + z2|2 ≤ (|z1|+ |z2|)2 . Finalmente, |z1 + z2| ≤ |z1|+ |z2|.7. |z1| − |z2| ≤ |z1 − z2| ≤ |z1|+ |z2|.A demonstracao das outras propriedades fica a cargo do leitor.

Exercıcios Resolvidos

1. Resolva a equacao z3 = 18 + 26i, onde z = x+ yi e x, y sao numeros inteiros.Solucao:(x+yi)3 = (x+yi)2(x+yi) = (x2−y2+2xyi)(x+yi) = (x3−3xy2)+(3x2y−y3) = 18+26i.Usando a definicao de igualdade de numeros complexos, obtemos:

{

x3 − 3xy2 = 183x2y − y3 = 26

Fazendo y = tx na igualdade 18(3x2y− y3) = 26(x3 − 3xy2), observamos que x 6= 0 e y 6= 0implica 18(3t−t3) = 26(1−3t2). A ultima relacao e equivalente a (3t−1)(3t2−12t−13) = 0.

A unica solucao racional da equacao e t =1

3, entao,

x = 3, y = 1 e z = 3 + i.

2. Prove a identidade

|z1 + z2|2 + |z1 − z2|2 = 2(|z1|2 + |z2|2)

2

Page 41: Álgebra nível 3

POT 2012 - Algebra - Nıvel 3 - Aula 5 - Prof. Marcelo Mendes/ Prof. Cıcero

Thiago

para todos os complexos z1 e z2.Solucao:Usando z.z = |z|2, temos que

|z1 + z2|2 + |z1 − z2|2 = (z1 + z2)(z1 + z2) + (z1 − z2)(z1 − z2)

= |z1|2 + z1.z2 + z2.z1 + |z2|2 + |z1|2 − z1.z2 − z2.z1 + |z2|2

2(|z1|2 + |z2|2).

3. (Croacia) No conjunto dos numeros complexos resolva a equacao(

x2 − a2)2−4ax−1 = 0,

em que a e um numero real.Solucao:

(

x2 − a2)2 − 4ax− 1 = 0 ⇔

(

x2 + a2)2 − 4a2x2 − 4ax− 1 = 0 ⇔

(

x2 + a2)2 − (2ax+ 1)2 = 0 ⇔

(

x2 + a2 − 2ax− 1) (

x2 + a2 + 2ax+ 1)

= 0 ⇔[

(x− a)2 − 1] [

(x+ a)2 + 1]

= 0 ⇔

(x− a− 1) (x− a+ 1) (x+ a− i) (x+ a+ i) = 0.

x = a+ 1, x = a− 1, x = −a+ i e x = −a− i.

4. (AIME) Sejam w1, w2, . . . , wn numeros complexos. Uma reta L no plano complexoe chamada de reta media para os pontos w1, w2, . . . , wn se L contem pontos (numeroscomplexos) z1, z2, . . . , zn tais que

n∑

k=1

(zk − wk) = 0.

Para os numeros w1 = 32 + 170i, w2 = −7 + 64i, w3 = −9 + 200i, w4 = 1 + 27i ew5 = −14 + 43i existe uma unica reta media que intersecta o eixo y no ponto (0, 3). De-termine o coeficiente angular desta reta media.

Solucao:

Seja y = mx + b uma reta media para os complexos wk = uk + ivk, onde uk e vk, ek = 1, 2, 3, . . . , n. Assuma que os numeros complexos zk = xk + iyk, onde xk e yk saonumeros reais escolhidos sobre a reta y = mx+ b, assim

n∑

k=1

(zk − wk) = 0.

Entao∑

xk =∑

uk,∑

yk =∑

vk,

3

Page 42: Álgebra nível 3

POT 2012 - Algebra - Nıvel 3 - Aula 5 - Prof. Marcelo Mendes/ Prof. Cıcero

Thiago

com yk = mxk + b, 1 ≤ k ≤ n. Consequentemente,

vk =∑

yk =∑

(mxk + b) = m∑

xk + nb =(

uk

)

m+ nb.

Nesse caso, n = 5, b = 3,∑

uk = 3 e∑

vk = 504. Segue que 504 = 3m+ 15 ⇔ m = 163.

5. Se a, b e n sao numeros inteiros e positivos, prove que existem inteiros x e y tais que(

a2 + b2)n

= x2 + y2.

Solucao:

Seja z = a+ bi. Entao,(

a2 + b2)n

=(

|z|2)n

= |z|2n = (|z|n)2 . Mas, zn = x+ iy, com x e

y inteiros (pois a e b sao inteiros). Portanto, (|z|n)2 = |x+ iy|2 = x2 + y2.

6. Seja f : C → C uma funcao tal que f(z)f(iz) = z2 para qualquer z ∈ C. Prove quef(z) + f(−z) = 0 para qualquer z ∈ C.Solucao:

Substitua z por iz na igualdade f(z)f(iz) = z2, entao f(iz)f(−z) = −z2. Somando asduas igualdades temos que f(iz) (f(z) + f(−z)) = 0 entao f(iz) = 0 ou f(z) + f(−z) = 0.Da igualdade f(z)f(iz) = z2 deduzimos que f(z) = 0 se, e somente se, z = 0. Se z 6= 0,entao f(iz) 6= 0 e, com isso, f(z)+ f(−z) = 0 e, se z = 0, entao f(z)+ f(−z) = 2f(0) = 0.Portanto, f(z) + f(−z) = 0 para qualquer z ∈ C. Um exemplo de funcao que satisfaz

f(z)f(iz) = z2 e f(z) =

(

−√2

2+ i

√2

2

)

z.

7. Se x e um numero real, prove que todos os numeros complexos de modulo 1 podem serescritos na forma

x+ i

x− i.

Solucao:

Para comecar seja (a+ bi)2 = (a− bi) (a+ bi)a+ bi

a− bi=(

a2 + b2)

a

b+ i

a

b− i

, e use o fato que

a2 + b2 = 1. Faca z = (a+ bi)2 ea

b= t ∈ R. Assim, |z| = | (a+ bi)2 | = a2 + b2 = 1 e

z =t+ i

t− i, t ∈ R.

Definicao 2

Todo numero complexo z = a+ bi pode ser escrito na forma trigonometrica

z = r(cos θ + i · sin θ),

onde r = |z| e θ o angulo (em radianos) que a reta, que liga a origem ao ponto z, formacom o eixo positivo real. O angulo θ e chamado de argumento de z.

4

Page 43: Álgebra nível 3

POT 2012 - Algebra - Nıvel 3 - Aula 5 - Prof. Marcelo Mendes/ Prof. Cıcero

Thiago

θ

Im

Rea

br

z = a+ bi = r(cos θ + i · sin θ)

O conjugado do complexo z = a+ bi, tem a forma trigonometrica

z = r(cos(−θ) + i · sin(−θ)) = r(cos θ − i · sin θ),

com isso,1

z=

z

|z|2 =r(cos θ − i · sin θ)

r2=

1

r(cos θ − i · sin θ).

Teorema 1

Seja n um inteiro, r e θ numeros reais, entao

[r(cos θ + i · sin θ)]n = rn (cos nθ + i · sin nθ) .

Prova:

Vamos provar que a igualdade e valida para n ∈ N e, em seguida, provemos para n ∈ Z.Para isso usaremos o princıpio da inducao finita.

Se n = 0 ⇒ z0 = 1 e r0 (cos 0 + i · sin 0) = 1. Vamos admitir a validade da formula paran = k − 1:

zk−1 = rk−1 · [cos (k − 1) θ + i · sin (k − 1) θ]

e agora provemos a validade da igualdade para n = k:

zk = zk−1 · z = rk−1 · [cos (k − 1) θ + i · sin (k − 1) θ] · r · (cos θ + i sin θ) =

rk (cos kθ + i · sin kθ)

Fica como exercıcio provar que a igualdade e valida para n negativo. Para isso, use que

z−n =1

zn, n ∈ N.

Teorema 2

Seja n um inteiro positivo e z um numero complexo. Existem n raızes n - esimas de z, quesao assim definidas

ωi = r1

n

(

cos

(

θ + 2kπ

n

)

+ i · sin(

θ + 2kπ

n

))

.

5

Page 44: Álgebra nível 3

POT 2012 - Algebra - Nıvel 3 - Aula 5 - Prof. Marcelo Mendes/ Prof. Cıcero

Thiago

para k = 0, 1, 2, . . . , n − 1.

Prova:

Para mostrar este resultado, inicialmente ω e z nas respectivas formas trigonometricas, ouseja,

ω = s(cosφ+ i · sinφ) e z = r(cos θ + i · sin θ).com s ≥ 0 e r ≥ 0. Com isso,

z = r(cos θ + i · sin θ) = ωn = sn(cosφ+ i · sinφ)n = sn(cos nφ+ i · sin nφ),

resultando,

s = r1

n e nφ = θ + 2kπ, para algum inteiro k.

Com isso, teremos n raızes n - esimas de z.

Exercıcios Resolvidos

01. Calcule

(

−1

2+ i ·

√3

2

)100

.

Solucao:

Seja z = −1

2+ i ·

√3

2, entao |z| =

(

−1

2

)2

+

(√3

2

)2

= 1, com isso cos θ = −1

2, sin θ =

√3

2.

Portanto, a forma trigonometrica sera, z = 1.

(

cos2π

3+ i · sin 2π

3

)

,

entao z100 = 1100(

cos200π

3+ i · sin 200π

3

)

= 1.

(

cos2π

3+ i · sin 2π

3

)

= −1

2+ i ·

√3

2.

02. As seis solucoes de z6 = −64 sao escritas na forma a+ bi, onde a e b sao numeros reais.Qual e o produto das solucoes com a > 0?Solucao:

O teorema 2 implica que as seis raızes sextas de

−64 = 64(cos π + i sinπ)

sao

zk = 641

6

(

cos

(

π + 2kπ

6

)

+ i · sin(

π + 2kπ

6

))

,

para k = 0, 1, 2, 3, 4 e 5.Verificando, concluımos que apenas z0 =

√3 + i e z5 =

√3 − i, tem a parte real positiva,

6

Page 45: Álgebra nível 3

POT 2012 - Algebra - Nıvel 3 - Aula 5 - Prof. Marcelo Mendes/ Prof. Cıcero

Thiago

entao z0 · z5 = 4.

3. Prove que cos2π

7+ cos

7+ cos

7+

1

2= 0.

Solucao:

Se z = cos θ + i · sin θ e z = cos θ − i · sin θ. Entao, cos θ =z + z

2. Assim, seja z =

cos2π

7+ i · sin 2π

7, entao z7 = 1. Portanto,

1

2

(

z +1

z

)

+1

2

(

z2 +1

z2

)

+1

2

(

z3 +1

z3

)

+1

2= 0.

Multiplicando tudo por 2z3 e organizando as parcelas, temos:

z6 + z5 + z4 + z3 + z2 + z + 1 = 0

ou seja,

z7 − 1

z − 1= 0.

4. (OPM) As raızes quintas do numero 1 sao as solucoes da equacao x5 − 1 = 0. Uma

dessas raızes e ǫ = cos2π

5+ i · sin 2π

5.

a) De as demais solucoes da equacao em funcao de ǫ.b) Calcule a soma dos cubos das 5 raızes.c) Calcule a soma das decimas potencias das 5 raızes.d) Generalize, se possıvel, os resultados anteriores para as n raızes n - esimas de 1.

Solucao:

a) As solucoes da equacao sao: cos2kπ

5+ i · sin 2kπ

5, k = 1, 2, 3, 4, 5 e sao, respectiva-

mente, iguais a ǫ, ǫ2, ǫ3, ǫ4 e ǫ5 = 1.b) S = ǫ3 + ǫ6 + ǫ9 + ǫ12 + ǫ15 = ǫ3 + ǫ+ ǫ4 + ǫ2 + 1 = 0 pois a soma das cinco raızes e ocoeficiente de x4 na equacao dada.c) ǫ10 + ǫ20 + ǫ30 + ǫ40 + ǫ50 = 1 + 1 + 1 + 1 + 1 = 5.d) A soma das potencias k das n raızes n - esimas da unidade e igual a 0 se k nao dividen e e igual a n, se k divide n.Demonstracao:

Tomando - se uma raiz primitiva z da unidade podemos escrever todas as raızes n -esimas como potencias de z: z1, z2, z3, . . . , zn. Logo as potencias de grau k serao:zk, z2k, z3k, . . . , znk.

Supondo que k nao divide n, zk 6= 1. Nesse caso a soma das potencias k das n raızes seradada por:

Sk =zk(

zkn − 1)

zk − 1.

7

Page 46: Álgebra nível 3

POT 2012 - Algebra - Nıvel 3 - Aula 5 - Prof. Marcelo Mendes/ Prof. Cıcero

Thiago

Como zn = 1, zkn = 1 e Sk = 0.Se k e multiplo de n, as potencias de z: zk, z2k, z3k,. . ., znk serao todas iguais a 1. Logo asoma sera Sk = 1 + 1 + . . .+ 1 = n.

5. Seja f(x) = cosx+ i · sinx.(a) Prove que f(0) = 1 e f(x)f(y) = f(x+ y) para quaisquer x e y reais.(b) Suponha que a funcao g satisfaz g(0) = 1 e g(x)g(y) = g(x + y) para quaisquer x e y

reais. Prove que essa funcao satisfaz:(i) g(x) 6= 0 para todo x.

(ii) g(x − y) =g(x)

g(y)para quaisquer x e y reais.

(iii) [g(x)]n = g(nx) para quaisquer x e y reais e n inteiro positivo.

(iv) g(−x) =1

g(x).

(c) De exemplo de uma funcao g tal que g(0) = 1 e que satisfaz g(x)g(y) = g(x + y) paraquaisquer x e y reais.

Solucao:

(a) Temos que f(0) = cos 0 + i · sin 0 = 1 + 0i = 1. Alem disso,

f(x)f(y) = (cosx+ i · sinx) (cos y + i · sin y)

= cos x cos y − sinx sin y + i · (sinx cos y + cos x sin y)

= cos (x+ y) + i · sin (x+ y)

f(x+ y)

(b) Fazendo y = −x em g(x)g(y) = g(x+ y) temos que g(x)g(−x) = g(0) = 1 para todo x.Portanto, nao podemos ter g(x) = 0 para algum x, assim g(x)g(−x) 6= 0.

Como g(x)g(y) = g(x+ y), dividindo ambos os lados por g(y) temos que g(x) =g(x+ y)

g(y).

Fazendo x + y =, temos que x = z − y, ou seja, g(z − y) =g(z)

g(y), o que a prova (ii). A

identidade [g(x)]n = g(nx) e uma consequencia imediata de g(x)g(y) = g(x + y). Vamosusar inducao para provar. Veja que [g(x)]1 = g(x) e se [g(x)]k = g(kx) para algum inteiropositivo k, entao

[g(x)]k+1 = g(x) [g(x)]k = g(x)g(kx) = g(x+ kx) = g((k + 1)x).

Isto mostra que se [g(x)]n = g(nx) ocorre para n = k, entao ocorre tambem para n = k+1.

Alem disso, se g(0) = 1 eg(x)

g(y)= g(x− y), fazendo x = 0 temos que

1

g(y)= g(−y).

(c) Um exemplo de funcao que satisfaz todas as propriedades e g(x) = 2y, pois g(0) = 20 = 1e

g(x)g(y) = 2x · 2y = 2x+y = g(x+ y).

8

Page 47: Álgebra nível 3

POT 2012 - Algebra - Nıvel 3 - Aula 5 - Prof. Marcelo Mendes/ Prof. Cıcero

Thiago

Isto nos motiva a fazer a seguinte definicao (nao sera exibida a prova desta afirmacao nestematerial):

Para todo x real, temos queeix = cos x+ i · sinx.

Fazendo x = π, na igualdade acima, encontramos uma maravilhosa igualdade

eiπ + 1 = 0.

Nas aulas 8 e 9 estudaremos varias aplicacoes das raızes da unidade. Deixaremos

a abordagem geometrica dos numeros complexos para a aula 21 de Geometria.

Exercıcios propostos

1. Prove que se |z1| = |z2| = 1 e z1z2 6= −1, entaoz1 + z2

1 + z1z2e um numero real.

2. Seja z ∈ C∗ tal que

z3 +1

z3

≤ 2. Prove que

z +1

z

≤ 2.

3. Ache todos os numeros complexos z tais que

|z| = 1 e |z2 + z2| = 1.

4. Seja zk = 3−k + 2−ki, k = 0, 1, 2, . . .. Calcule

∞∑

k=0

zk.

5. Seja zk = ak+ibk, k = 1, 2, onde ak, bk ∈ R. Se z1 = 3z2 e z1 = (1+2i)4. Ache a2 e b2.

6. Seja z = x+ yi. Se |z − 6| = 5 e |z| = 5, ache todos os possıveis valores de x e y.

7. Ache todos os numeros complexos z tais que

(3z + 1)(4z + 1)(6z + 1)(12z + 1) = 2.

8. Resolva o sistema de equacoes:

x+3x− y

x2 + y2= 3

y − x+ 3y

x2 + y2= 0.

9

Page 48: Álgebra nível 3

POT 2012 - Algebra - Nıvel 3 - Aula 5 - Prof. Marcelo Mendes/ Prof. Cıcero

Thiago

9. (OCM) Sejam a e z numeros complexos tais que |a| < 1 e az 6= 1. Mostre que se∣

z − a

1− az

< 1 entao |z| < 1.

10. (OCM) Uma lista de numeros complexos distintos z1, z2, . . . , zn e um ciclo de com-primento n para uma funcao f : C → C se z2 = f(z1), z3 = f(z2), . . . , zn = f(zn−1)e z1 = f(zn). Seja f(z) = z2 + 2003 e z1, z2, . . . , z2003 um ciclo de comprimento2003. Calcule

2003∏

i=1

(f(zi) + zi),

onde o sımbolo∏

indica o produto.

11. Sejam z1, z2 ∈ C numeros complexos tais que |z1+z2| =√3 e |z1| = |z2| = 1. Calcule

|z1 − z2|.

12. (Putnam) Prove que se 11z10 + 10iz9 + 10iz − 11 = 0, entao |z| = 1.

13. Seja z um numero complexo tal que z ∈ C − R e

1 + z + z2

1− z + z2∈ R.

Prove que |z| = 1.

14. (Espanha) Resolva o sistema se equacoes no conjunto dos numeros complexos:

|z1| = |z2| = |z3| = 1,

z1 + z2 + z3 = 1,

z1z2z3 = 1.

15. Ache todos os numeros complexos z tais que |z| = 1 e

z

z+

z

z

= 1.

16. Ache todos os pares ordenados (a, b) de numeros reais tais que (a+ bi)2002 = a− bi.

17. Prove que

cosπ

11+ cos

11+ cos

11+ cos

11+ cos

11=

1

2.

10

Page 49: Álgebra nível 3

POT 2012 - Algebra - Nıvel 3 - Aula 5 - Prof. Marcelo Mendes/ Prof. Cıcero

Thiago

18. Sejam a, b, c numeros reais tais que

cos a+ cos b+ cos c = sin a+ sin b+ sin c = 0.

Prove que

cos 2a+ cos 2b+ cos 2c = sin 2a+ sin 2b+ sin 2c = 0.

19. Prove que

cosπ

7+ cos

7+ cos

7=

1

2.

20. Prove que

sin2π

7+ sin

7+ sin

7=

√7

2.

21. Sejam a, b, c numeros reais tais que

cos a+ cos b+ cos c = sin a+ sin b+ sin c = 0.

Prove que

cos (a+ b+ c) =1

3(cos 3a+ cos 3b+ cos 3c),

sin (a+ b+ c) =1

3(sin 3a+ sin 3b+ sin 3c).

22. (Croacia) Determine os valores mınimo e maximo, caso existam, da expressao |z− 1

z|,

em que z e um numero complexo tal que |z| = 2.

23. (OCM) Determinar todos os subconjuntos S dos numeros complexos que satisfazemaos seguintes requisitos:1. Se x, y ∈ S, entao xy ∈ S.2. S possui 2002 elementos.

24. (Mandelbrot) O numero complexo

(

3 + i√3

2

)8

+

(

3− i√3

2

)8

e um numero inteiro.

Qual?

25. (Mandelbrot) Se ω1997 = 1 e ω 6= 1, entao determine o valor de

1

1 + ω+

1

1 + ω2+ . . . +

1

1 + ω1997.

11

Page 50: Álgebra nível 3

POT 2012 - Algebra - Nıvel 3 - Aula 5 - Prof. Marcelo Mendes/ Prof. Cıcero

Thiago

26. (Romenia) Resolva em C, o conjunto dos numeros complexos, as seguintes equacoes:a) |z − a|+ |z − b| = b− a.b) |z|+ |z − 1|+ |z − 2|+ |z − 3| = 4.

27. (AIME) Seja F (z) =z + i

z − ipara qualquer complexo z 6= i, e seja zn = F (zn−1) para

todo inteiro positivo n. Dado que z0 =1

137+ i e z2002 = a + bi, em que a e b sao

numeros reais, determine a+ b.

28. (AIME) Para quantos inteiros positivos n menores ou iguais a 1000

(sin t+ i · cos t)n = sinnt+ i · cosnt

para todo real t?

29. (AIME) Uma funcao f e definida no conjunto dos numeros complexos por f(z) =(a+ bi) z em que a e b sao numeros positivos. Esta funcao tem a propriedade de quea imagem de cada ponto no plano complexo e equidistante do ponto e da origem.

Dado que |a+ bi| = 8 e b2 =m

n, em que m e n sao inteiros positivos primos entre si.

Determine m+ n.

30. (Croacia) Determine todos os inteiros a, b e c que satisfazem a igualdade (a+ bi)3 −107i = c, onde i =

√−1 e a unidade imaginaria.

31. (Croacia) Calcule

(

1 + cosπ

7+ i · sin 6π

7

)14

.

Sugestoes / Solucoes

1. Use que z ∈ R ⇔ z = z.

2. Faca a = |z + 1

z|, desenvolva

(

z +1

z

)3

e use a desigualdade triangular.

3. Use que z · z = |z|2.

4. Use progressoes geometricas.

7. Perceba que8 (3z + 1) 6 (4z + 1) 4 (6z + 1) 2 (12z + 1) = 768,

⇔ (24z + 8) (24z + 6) (24z + 4) (24z + 2) = 768.

12

Page 51: Álgebra nível 3

POT 2012 - Algebra - Nıvel 3 - Aula 5 - Prof. Marcelo Mendes/ Prof. Cıcero

Thiago

Faca u = 24z + 5 e w = u2 assim,

(u+ 3) (u+ 1) (u− 1) (u− 3) = 768,

(

u2 − 1) (

u2 − 9)

= 768,

w2 − 10w − 759 = 0,

(w − 33) (w + 23) = 0.

Portanto, z =±√33− 5

24e z =

±√23i− 5

24.

8. Multiplicando a segunda equacao por i e adicionando a primeira equacao, temos:

x+ yi =(3x− y)− (x+ 3y) i

x2 + y2= 3 ⇔

x+ yi+3 (x− yi)

x2 + y2− i (x− yi)

x2 + y2= 3.

Seja z = x+ yi. Entao,1

z=

x− yi

x2 + y2.

A ultima equacao e equivalente a

z +3− i

z= 3 ⇔

z =3±

√−3 + 4i

2=

3± (1 + 2i)

2,

assim, (x, y) = (2, 1) ou (x, y) = (1,−1)

9. Use o fato que | z − a

1− az|2 =

(

z − a

1− az

)(

z − a

1− az

)

.

10. Usando a lei de formacao da funcao conseguimos a seguinte relacao z2i −z2i−1 = zi+1−zi.

11. Use que z ∈ R ⇔ z = z.

13. Faca z9 =11− 10iz

11z + 10ie z = a+ bi.

14. Use que z ∈ R ⇔ z = z.

16. Seja z = a + bi, z = a − bi, e |z| =√a2 + b2. Temos que z2002 = z. Note que

|z|2002 = |z2002| = |z| = |z|, segue que |z|(

|z|2001 − 1)

= 0. Entao |z| = 0 ou |z| = 1. Se|z| = 0 teremos apenas uma solucao z = 0. No caso em que |z| = 1, nos temos z2002 = z,que e equivalente a z2003 = z ·z = |z|2 = 1. A equacao z2003 = 1 tem 2003 solucoes distintas.

13

Page 52: Álgebra nível 3

POT 2012 - Algebra - Nıvel 3 - Aula 5 - Prof. Marcelo Mendes/ Prof. Cıcero

Thiago

Nos problema 18 e 20 use que se z = cos θ+i·sin θ e z = cos θ−i·sin θ. Entao, cos θ =z + z

2.

Nos problemas 19, 21 e 22 use que se z = cos θ + i · sin θ e z = cos θ − i · sin θ. Entao,

cos θ =z + z

2e sin θ =

z − z

2i.

22. Faca z = a+ bi. Assim,4 = |z|2 = a2 + b2.

Portanto,

|z − 1

z| = |a+ bi− 1

a+ bi| = |a+ bi− a− bi

a2 + b2|2

|a+ bi− a− bi

4|2 = |3

4a+

5

4bi|2

9

16a2 +

25

16b2 =

9

16

(

4− b2)

+25

16b2 =

9

4+ b2.

Assim, o menor valor de |z − 1

z|2 e

9

4que e obtido quando b = 0. Entao, o menor valor de

|z − 1

z| e 3

2. Observe que a2 = 4 ⇔ a = ±2. Por outro lado, se a2 + b2 = 4, o maximo de

b2 e 4. Entao, o maximo valor de |z − 1

z|2 e

9

4+ 4 e e obtido quando b2 = 4 ⇔ b = ±2.

Entao, o maior valor de |z − 1

z| e 5

2para b = ±2. Observe que nesse caso a2 = 0 ⇔ a = 0.

24. Faca C =

(

3 + i√3

2

)

e D =

(

3− i√3

2

)

. E facil ver que C +D = 3 e CD = 3, entao

C8 +D8 =(

C4 +D4)2 − 2 (CD)4. Faca o mesmo com C4 +D4.

25. Calcule1

1 + ωk+

1

1 + ω1997−k, k = 1, 2, . . . , 998.

26. a) A interpretacao geometrica da distancia no plano complexo e a desigualdade trian-gular mostram que z e um ponto sobre o segmento [a, b].b) Usando os argumentos do item (a) temos:⋆ |z| + |z − 3| ≥ |z − z + 3| = 3, com igualdade acontecendo se, e somente se, z e umnumero real e 0 ≤ z ≤ 3.⋆ |z − 1| + |z − 2| ≥ |z − 1− z + 2| = 1, com igualdade acontecendo se, e somente se, z ereal e 1 ≤ z ≤ 2. Adicionando as desigualdades obtemos |z|+ |z − 1|+ |z − 2|+ |z − 3| = 4se, e somente se, z e um numero real e 1 ≤ z ≤ 2.

27. F (F (z)) =

z + i

z − i+ i

z + i

z − i− i

=z + i+ iz + i

z + i− iz − i=

1 + i

1− i· z + 1

z − 1= i · z + i

z − i. Agora, e facil ver que

F (F (F (z))) = z, o que mostra que zn = zn−3, ∀n ≥ 3. Em particular, z2002 = z2002−667·3 =

14

Page 53: Álgebra nível 3

POT 2012 - Algebra - Nıvel 3 - Aula 5 - Prof. Marcelo Mendes/ Prof. Cıcero

Thiago

z1 =

1

137+ 2i

1

137

= 1 + 274i. Portanto, a+ b = 275.

28. Note que

(sin t+ i · cos t)n = sinnt+ i · cosnt =[

cos(π

2− t)

+ i · sin(π

2− t)]n

=

cosn(π

2− t)

+ i · n sin(π

2− t)

=

cos(nπ

2− nt

)

+ i · sin(nπ

2− nt

)

.

e que

sinnt+ i · cosnt = cos(π

2− nt

)

+ i · sin(π

2− nt

)

.

Tal condicao e equivalente a cos(nπ

2− nt

)

= cos(π

2− nt

)

e sin(nπ

2− nt

)

= sin(π

2− nt

)

,

entaonπ

2− nt− π

2+ nt = 2πk o que implica que n = 4k + 1. Como 1 ≤ n ≤ 1000 entao

0 ≤ k ≤ 249, ou seja, existem 250 valores de n que satisfazem a igualdade.

29. Como (a+ bi) z e equidistante de z e 0, entao | (a+ bi) z − z| = | (a+ bi) z|. Assim,

|a − 1 + bi| = |a + bi|, ou seja, (a− 1)2 + b2 = a2 + b2. Portanto, a =1

2. Sabemos que

|a+ bi| = 8, entao b =255

4. Finalmente, m+ n = 259.

30. (a+ bi)3 − 107i = c ⇔ a3 − 3ab2 +(

3a2b− b3 − 107)

i = c. Comparando as partesreais e imaginarias temos que a3 − 3ab2 = c e 3a2b − b3 − 107 = 0. A segunda equacao eequivalente a

(

3a2b− b2)

b = 107. Mas 107 e um numero primo e a e b inteiros positivos,segue que:

3a2b− b2 = 107, b = 1

ou

3a2b− b2 = 1, b = 107.

No primeiro caso teremos a = 6 enquanto que no segundo caso nao encontraremos um valorinteiro para a. Portanto, c = 198.

31. Temos que 1 + cosπ

7= 2 cos2

π

14e sin

7= sin

π

7= 2 sin

π

14cos

π

14. Entao,

1 + cosπ

7+ i · sin 6π

7= 2 cos2

π

14

(

cosπ

14+ i · sin π

14

)

.

Finalmente,

(

1 + cosπ

7+ i · sin 6π

7

)14

=(

2 cos2π

14

(

cosπ

14+ i · sin π

14

))14

15

Page 54: Álgebra nível 3

POT 2012 - Algebra - Nıvel 3 - Aula 5 - Prof. Marcelo Mendes/ Prof. Cıcero

Thiago

= 214(

cosπ

14

)14 (

cosπ

14+ i · sin π

14

)14

= 214(

cosπ

14

)14(cos π + i · sinπ)

= −214(

cosπ

14

)14

Bibliografia

1. Topicos de Matematica Elementar, vol. 6.Polinomios.Antonio Caminha Muniz NetoSBM

2. Complex Numbers from A to Z.Titu Andreescu e Dorin AndricaBirkhauser

3. 101 Problems in Algebra: From the training of the USA IMO team.Titu AndreescuAMT publishing

4. Olimpıadas Cearenses de Matematica, Ensino Medio, 1981 - 2005Emanuel Carneiro, Francisco Antonio M. de Paiva e Onofre Campos

5. The USSR Olympiad Problem BookSelected Problems and Theorems of Elementary MathematicsD.O. Shklarsky, N.N. Chentzov e I.M. Yaglom

6. First Steps for Math OlympiansUsing the American Mathematics CompetitionsJ. Douglas Faires

7. Problem - Solving Through ProblemsLoren C. LarsonSpringer

8. Olimpıada de Matematica de 1977 a 1997Questoes e Solucoes - 2o grau.

9. Fundamentos de Matematica Elementar, vol. 6Complexos, Polinomios e EquacoesGelson Iezzi

16

Page 55: Álgebra nível 3

POT 2012 - Algebra - Nıvel 3 - Aula 5 - Prof. Marcelo Mendes/ Prof. Cıcero

Thiago

10. Mathematical Olympiad TreasuresTitu Andreescu e Bogdan EnescuBirkhauser

11. 360 Problems for Mathematical ContestsTitu Andreescu e Dorin AndricaGIL

12. The Mandelbrot Problem BookSam Vandervelde

13. The First Five YearsSam Vandervelde

14. Winning SolutionsCecil Rousseau e Edward Lozansky

15. Curso de Algebra, Vol. 1.Abramo HefezIMPA

16. Plane Trigonometry and Complex NumbersDusan Jevtic

17. A problem book in algebraV. A. Krechmar

18. Matematica em nıvel IME - ITANumeros Complexos e PolinomiosCaio dos Santos Guimaraes

19. PrecalculusRichard Rusczykthe Art of Problem Solving

20. A Matematica do ensino medio, vol. 4Enunciados e solucoes dos exercıciosElon Lages Lima, Paulo Cezar Pinto Carvalho, Eduardo Wagner e Augusto Cesar Morgado

21. A Decade of Berkeley Math CircleThe American Experience, vol. 1Zvezdelina Stankova e Tom Rike

17

Page 56: Álgebra nível 3

Polos Olímpicos de TreinamentoCurso de Álgebra - Nível 3Prof. Cícero Thiago / Prof. Marcelo Mendes

Aula 6

Miscelanea sobre raızes de polinomios I

Definicao 1

Um polinomio na variavel x e uma expressao que pode ser escrita na forma

P (x) = anxn + an−1x

n−1 + . . .+ a1x+ a0,

onde n ∈ N e ai (i = 0, 1, . . . , n), chamados coeficientes, sao numeros em algum dosconjuntos (Z,Q,R,C). O numero n sera chamado de grau do polinomio. Chamamos decoeficiente lıder o coeficiente do termo de maior grau, nesse caso an, e chamamos de termo

independente o coeficiente a0. Um polinomio com todos os coeficientes iguais a zero echamado de polinomio nulo. Um polinomio com coeficiente lıder igual a 1 e chamado depolinomio monico.

Definicao 2

Seja c um numero, o numero P (c) = ancn + an−1c

n−1 + . . .+ a1c+ a0 e chamado de valordo polinomio aplicado ao numero c. Se P (c) = 0, dizemos que c e um zero ou raiz dopolinomio P (x).

Definicao 3

Dados dois polinomios P (x) e M(x) 6= 0 , dividir P (x) por M(x) e determinar dois outrospolinomios Q(x) e R(x) de modo que se verifiquem as duas condicoes seguintes:(a) P (x) = M(x) ·Q(x) +R(x).(b) O grau de R(x) e menor que o grau deM(x) ou R(x) = 0, caso em que a divisao e exata.

Teorema 1. Seja P (x) um polinomio tal que x− a e um fator de P (x), entao P (a) = 0.Demonstracao. Se x − a e um fator de P (x), entao P (x) = (x − a) · Q(x) para algumpolinomio Q(x). Fazendo x = a temos que P (a) = (a− a) ·Q(a) = 0 ·Q(a) = 0, entao a euma raiz de P (x).

Teorema 2. Seja P (x) um polinomio tal que P (a) = 0, entao x− a e um fator de P (x).Demonstracao. Se a e uma raiz de P (x), entao P (a) = 0. Pelo algoritmo da divisao,

Page 57: Álgebra nível 3

POT 2012 - Algebra - Nıvel 3 - Aula 6 - Prof. Cıcero Thiago/ Prof. Marcelo

Mendes

temos que o resto quando P (x) e dividido por x− a e P (a). Como P (a) = 0 entao o restoe zero. Isto mostra que x− a e um fator de P (x).

Dispositivo de Briot - Ruffini

Dados os polinomios P (x) = anxn + an−1x

n−1 + . . . + a1x + a0, an 6= 0 e M(x) = x − a.Nosso desejo e determinar o quociente Q(x) e o resto R(x) da divisao de P (x) por M(x).Seja Q(x) = qn−1x

n−1 + qn−2xn−2 + . . .+ q0, entao:

(qn−1xn−1 + qn−2x

n−2 + . . .+ q0) · (x− a) =

qn−1xn + qn−2x

n−1 + . . .+ q0x− aqn−1xn−1 − aqn−2x

n−2 − . . . − aq1x− aq0 =

qn−1xn + (qn−2 − aqn−1)x

n−1 + . . .+ (q0 − aq1)x− aq0

Fazendo P (x) = Q(x) · (x− a) +R(x), temos:

qn−1 = an

qn−2 − aqn−1 = an−1 ⇒ qn−2 = aqn−1 + an−1

...

q0 − aq1 = a1 ⇒ q0 = aq1 + a1

R(x)− aq0 = a0 ⇒ R(x) = aq0 + a0

Exemplo

Vamos achar o quociente e o resto da divisao de P (x) = 2x4−7x2+3x−1 porM(x) = x−3.Para isso usaremos o dispositivo de Briot - Ruffini:

3 2 0 −7 3 −1

2 2 · 3 + 0 6 · 3− 7 11 · 3 + 3 36 · 3− 1

3 2 0 −7 3 −1

2 6 11 36 107

Portanto, Q(x) = 2x3 + 6x2 + 11x+ 36 e R(x) = 107.

Teorema 3. (Teorema Fundamental da Algebra) Todo polinomio P (x) de grau n ≥ 1possui ao menos uma raiz complexa.Uma demonstracao desse teorema pode ser encontrada em [1].

Teorema 4. Seja P (x) = anxn+an−1x

n−1+ . . .+a1x+a0 um polinomio de grau n (n ≥ 1)e an 6= 0, entao

P (x) = an(x− x1)(x− x2) . . . (x− xn),

em que x1, x2, . . . , xn sao as raızes de P (x).Para a demonstracao desse teorema use os teoremas 2 e 3.

2

Page 58: Álgebra nível 3

POT 2012 - Algebra - Nıvel 3 - Aula 6 - Prof. Cıcero Thiago/ Prof. Marcelo

Mendes

Teorema 5. Se o polinomio

P (x) = anxn + an−1x

n−1 + . . .+ a1x+ a0,

de grau n, possui n + 1 raızes, entao este polinomio e identicamente igual a 0, ou seja,an = an−1 = . . . = a0 = 0.Demonstracao. Vamos demonstrar usando inducao sobre n. Para n = 1, a prova eimediata. Vamos provar que se a afirmacao e verdadeira para n − 1, entao tambem seraverdadeira para n. Seja x0, x1, . . . , xn sao raızes de P entao

P (x) = (x− xn)Q(x),

com o polinomio Q(x) tendo grau n−1 e n raızes distintas x0, x1, . . . , xn−1. Pela inducao,Q(x) e identicamente nulo. Segue que P (x) e tambem nulo.

Teorema 6. (Raızes Irracionais) Seja ti = bi + ci√d e ti = bi − ci

√d , em que bi, ci e d

sao numeros racionais e√d e irracional. Entao,

(a) t1 + t2 = t1 + t2.

(b) t1 · t2 = t1 · t2.(c) se P (x) e um polinomio com coeficientes racionais tais que t1 e uma raiz de P (x), entaot1 e tambem uma raiz.

Demonstracao. (a) Temos que t1+ t2 = (b1+c1√d)+(b2+c2

√d) = (b1+b2)+(c1+c2)

√d,

entao t1 + t2 = (b1 + b2) − (c1 + c2)√d. Por outro lado, t1 = b1 − c1

√d e t2 = b2 − c2

√d,

portanto t1 + t2 = t1 + t2.

(b) Temos que

t1 · t2 = (b1 + c1√d)(b2 + c2

√d) =

(b1b2 + c1c2d) + (b1c2 + b2c1)√d =

(b1b2 + c1c2d)− (b1c2 + b2c1)√d.

Alem disso, t1 · t2 = (b1 − c1√d)(b2 − c2

√d) = (b1b2 + c1c2d)− (b1c2 + b2c1)

√d. Portanto,

t1 · t2 = t1 · t2.

(c) Seja P (x) = anxn + an−1x

n−1 + . . .+ a1x+ a0. Aplicando (a) e (b), temos que:

P (t1) = an(t1)n + an−1(t1)

n−1 + . . . + a1(t1) + a0

= an(t1)n + an−1(t1)n−1 + . . .+ a1(t1) + a0

= antn1 + an−1t

n−11 + . . .+ a1t1 + a0

= P (t1) = 0 = 0,

pois t1 e uma raiz de P (x).

3

Page 59: Álgebra nível 3

POT 2012 - Algebra - Nıvel 3 - Aula 6 - Prof. Cıcero Thiago/ Prof. Marcelo

Mendes

Teorema 7. (Raızes Racionais) Se P (x) = anxn + an−1x

n−1 + . . . + a1x + a0 e um

polinomio com coeficientes inteiros, tal quep

qe uma raiz, com p e q inteiros, q 6= 0 e

mdc(p, q) = 1, entao p|a0 e q|an.

Demonstracao. Sep

qe uma raiz de P (x), entao

an ·(

p

q

)n

+ an−1 ·(

p

q

)n−1

+ . . . + a1 ·p

q+ a0 = 0 ⇔

an · pn

qn+ an−1 ·

pn−1

qn−1+ . . . + a1 ·

p

q+ a0 = 0 ⇔

anpn + an−1p

n−1q + an−2pn−2q2 + . . .+ a1pq

n−1 + a0qn = 0 ⇔

anpn = −q[an−1p

n−1 + an−2pn−2q + . . .+ a1pq

n−2 + a0qn−1] ⇒

p|a0pois mdc(p, q) = 1.

De maneira analoga, e facil provar que q|an.

Teorema 8. (Raızes Complexas) Se P (x) = anxn + an−1x

n−1 + . . . + a1x + a0 e umpolinomio com coeficientes reais tal que z = a+ bi e uma raiz, com a e b reais, b 6= 0, entaoz = a− bi e tambem uma raiz.

Demonstracao. Se z e uma raiz de P (x) = anxn + an−1x

n−1 + . . . + a1x + a0, entaoP (z) = 0. Assim,

P (z) = an(z)n + an−1(z)

n−1 + . . . + a1(z) + a0 =

anzn + an−1zn−1 + . . . + a1z + a0 =

an · zn + an−1 · zn−1 + . . .+ a1 · z + a0 =

anzn + an−1zn−1 + . . . + a1z + a0 =

anzn + an−1zn−1 + . . . + a1z + a0 =

P (z) = 0 = 0.

Teorema 9. (Relacoes de Girard) Seja P (x) = anxn + an−1x

n−1 + . . . + a1x+ a0 umpolinomio e x1, x2, . . . , xn suas raızes (reais ou complexas). Entao:

x1 + x2 + . . .+ xn = −an−1

an,

x1x2 + x1x3 + . . .+ xn−1xn =an−2

an

4

Page 60: Álgebra nível 3

POT 2012 - Algebra - Nıvel 3 - Aula 6 - Prof. Cıcero Thiago/ Prof. Marcelo

Mendes

x1x2x3 + x1x2x4 + . . .+ xn−2xn−1xn = −an−3

an...

x1x2 . . . xn = (−1)na0

an

Exercıcios Resolvidos

1. (Torneio as Cidades) Prove que se a, b e c sao numeros inteiros e as somasa

b+

b

c+

c

a

ea

c+

c

b+

b

asao tambem inteiros, entao |a| = |b| = |c|.

Solucao. Seja

p =a

b+

b

c+

c

ae

q =a

c+

c

b+

b

a.

As raızes de x3−px2+qx−1 = 0 saoa

b,b

cec

a. Como os coeficientes sao inteiros e as

raızes racionais, os unicos pssıveis valores para as raızes sao ±1. Entao |a| = |b| = |c|.

2. (OCM) Sejam a, b, c e d as raızes (nos complexos) do polinomio x4 + 6x2 + 4x+ 2.Encontre um polinomio p(x), do quarto grau, que tenha como raızes a2, b2, c2 e d2.

Solucao. Seja Q(x) = x4 + 6x2 + 4x + 2 = (x − a)(x − b)(x − c)(x − d). Queremosencontrar

P (x) = (x− a2)(x− b2)(x− c2)(x− d2).

Fazendo x = y2,

P (y2) = (y2 − a2)(y2 − b2)(y2 − c2)(y2 − d2)

= (y + a)(y − a)(y + b)(y − b)(y + c)(y − c)(y + d)(y − d)

[(y − a)(y − b)(y − c)(y − d)][(y + a)(y + b)(y + c)(y + d)]

Q(y)Q(−y).

Assim,P (y2) = (y4 + 6y2 + 4y + 2)(y4 + 6y2 − 4y + 2)

= (y4 + 6y2 + 2)2 − (4y2)2

y8 + 12y6 + 40y4 + 8y2 + 4.

Voltando para a variavel x pela substituicao y2 = x, temos:

P (x) = x4 + 12x3 + 40x2 + 8x+ 4.

5

Page 61: Álgebra nível 3

POT 2012 - Algebra - Nıvel 3 - Aula 6 - Prof. Cıcero Thiago/ Prof. Marcelo

Mendes

3. (Torneio das Cidades) Sabendo que a equacao

x4 + ax3 + 2x2 + bx+ 1 = 0

possui uma raiz real, prove quea2 + b2 ≥ 8.

Solucao. Temos que

x4 + ax3 + 2x2 + bx+ 1 = (x2 + px+ q)(x2 + sx+ t) (1)

em que p, q, s, t sao reais. Como, pelo menos uma das raızes sao reais, iremosassumir que ela e raiz de x2 + sx+ t, entao:

s2 ≥ 4t.

Igualando os coeficientes em (1), temos:

a = p+ s;

2 = q + t+ ps;

b = pt+ qs;

1 = qt.

Portanto,a2 + b2 = p2 + q2 + 2ps+ p2t2 + q2s2 + 2ptqs

= p2(1 + t2) + s2(1 + q2) + 4ps

≥ p2(1 + t2) + 4(t+ q + ps)

≥ 8.

4. (Bulgaria) Os comprimentos das alturas do ∆ABC sao solucoes da equacao cubica

x3 + kx2 + lx+m = 0.

Determine o raio do cırculo inscrito no ∆ABC.

(a)k

m(b) − l

k(c) − l

m(d)

m

k(e) −m

lSolucao. (C) Temos que

1

r=

p

S=

a

2S+

b

2S+

c

2S=

1

ha+

1

hb+

1

hc.

Usando as Relacoes de Girard, temos:

1

ha+

1

hb+

1

hc=

hbhc + hahc + hahb

hahbhc=

l

−m= − l

m.

6

Page 62: Álgebra nível 3

POT 2012 - Algebra - Nıvel 3 - Aula 6 - Prof. Cıcero Thiago/ Prof. Marcelo

Mendes

5. (Bulgaria) Determine o numero de raızes reais da equacao

x1994 − x2 + 1 = 0.

(a) 0 (b) 2 (c) 4 (d) 1994

Solucao. (A) Se |x| < 1 entao 1− x2 > 0 e x1994 ≥ 0 o que implica

x1994 − x2 + 1 > 0.

Se |x| ≥ 1 entao x1994 − x2 = x2(x1992 − 1) e, com isso,

x1994 − x2 + 1 > 0,

portanto a equacao nao possui raızes reais.

6. (IMTS) Seja f(x) = x4 +17x3 +80x2 +203x+125. Determine o polinomio, g(x), demenor grau possıvel, tal que f(3±

√3) = g(3 ±

√3) e f(5±

√5) = g(5 ±

√5).

Solucao. Seja g(x) o polinomio que desejamos encontrar e h(x) um polinomio talque h(x) = f(x)− g(x). Com isso h(3±

√3) = 0 e h(5±

√5) = 0. Portanto,

f(x)− g(x) = h(x) = a(x)(x− 3−√3)(x− 3 +

√3)(x− 5−

√5)(x− 5 +

√5) ⇔

f(x)− g(x) = h(x) = a(x)(x4 − 16x3 + 86x2 − 180x+ 120) ⇔g(x) = f(x)− a(x)(x4 − 16x3 + 86x2 − 180x+ 120) ⇔

g(x) = x4 + 17x3 + 80x2 + 203x+ 125 − a(x)(x4 − 16x3 + 86x2 − 180x + 120).

Finalmente, g(x) tera grau menor que 4 se, e somente se, a(x) ≡ 1. Nesse casog(x) = 33x3 − 6x2 + 383x + 5.

7. (Australia) Seja P (x) um polinomio cubico com raızes r1, r2 e r3. Suponha que

P

(

1

2

)

+ P

(

−1

2

)

P (0)= 1000.

Determine o valor de1

r1r2+

1

r2r3+

1

r1r3.

Solucao. Seja P (x) = a3x3 + a2x

2 + a1x + a0. Pela relacoes de Girard, temos que

r1 + r2 + r3 = −a2

a3e r1r2r3 = −a0

a3. Assim,

7

Page 63: Álgebra nível 3

POT 2012 - Algebra - Nıvel 3 - Aula 6 - Prof. Cıcero Thiago/ Prof. Marcelo

Mendes

1

r1r2+

1

r2r3+

1

r1r3=

r1 + r2 + r3

r1r2r3=

a2

a0.

Mas,

P

(

1

2

)

=a3

8+

a2

4+

a1

2+ a0,

e

P

(

−1

2

)

= −a3

8+

a2

4− a1

2+ a0.

Portanto, 1000 =

P

(

1

2

)

+ P

(

−1

2

)

P (0)=

a2

2+ 2a0

a0=

a2

2a0+ 2.

Finalmente,1

r1r2+

1

r2r3+

1

r1r3=

a2

a0= 2(1000 − 2) = 1996.

8. (Australia) Determine todos os polinomios f com coeficientes reais tais que

(x− 27)f(3x) = 27(x− 1)f(x)

para todo numero real x.

Solucao. Considere a equacao

(x− 27)f(3x) = 27(x − 1)f(x) (1)

Se x = 27 temos que 0 = 0 · f(81) = 27 · 26 · f(27), entao f(27) = 0. De maneiraanaloga, se x = 1 temos que 0 = 27 · 0 · f(1) = −26 · f(3) entao f(3) = 0.

Com isso, f(x) = (x − 3)(x − 27)q(x). Substituindo esse resultado encontrado naequacao inicial temos que

(x− 27)(3x − 3)(3x − 27)q(3x) = 27(x− 1)(x− 3)(x − 27)q(x).

Para x 6= 1, 27 temos que

(x− 9)q(3x) = 3(x− 3)q(x). (2)

Agora, se x = 3 temos que 0 = 3 · 0 · q(3) = −6q(9), ou seja, q(9) = 0, assimq(x) = (x− 9)g(x) que substituiremos na equacao (2) obtendo

(x− 9)(3x − 9)g(3x) = 3(x− 3)(x− 9)g(x),

8

Page 64: Álgebra nível 3

POT 2012 - Algebra - Nıvel 3 - Aula 6 - Prof. Cıcero Thiago/ Prof. Marcelo

Mendes

que para x 6= 1, 3, 9, 27 resulta g(3x) = g(x).

Em particular, se x = 2 entao g(2) = g(6) = g(18) = . . . = g(2 · 3k), ∀k. Assim,g(x) = g(2) possui infinitas raızes, o que e impossıvel, ou g(x) e uma constante, di-gamos g(x) = a.Finalmente, q(x) = a(x− 9) e f(x) = a(x− 3)(x− 9)(x − 27), a ∈ R.

9. (Australia) Prove que o polinomio 4x8− 2x7+x6− 3x4+x2−x+1 nao possui raızesreais.

Solucao. Temos que

P (x) = 4x8 − 2x7 + x6 − 3x4 + x2 − x+ 1 ⇔

P (x) = 3 ·(

x4 − 1

2

)2

+[

x3 (x− 1)]2

+

(

x− 1

2

)2

.

Portanto, P (x) e uma soma de quadrados. Para que P (x) = 0 todos os quadrados

precisam ser iguais a zero, assim teremos que as duas igualdades x = ± 4

1

2e x =

1

2devem acontecer simultaneamente, absurdo. Finalmente, P (x) nao possui raızes reais.

10. (AIME) Sejam a, b, c e d numeros reais tais que a equacao x4+ax3+bx2+cx+d = 0possui quatro raızes nao reais. O produto de duas dessas quatro raızes e 13 + i e asoma das outras duas e 3 + 4i em que i =

√−1. Determine b.

Solucao. Sejam r1, r2, r3 e r4 as raızes. Se r1r2 = 13 + i e r3 + r4 = 3 + 4i. Comoo polinomio possui coeficientes reais e nenhuma das raızes sao reais, entao r3 = r1 er4 = r2. Segue que r3r4 = r1r2 = 13 + i e r1 + r2 = r3 + r4 = 3 − 4i. Com isso, opolinomio sera

[x2 − (3− 4i)x+ (13 + i)][x2 − (3 + 4i)x+ (13 + i)]

= x4 − 6x3 + 51x2 − 70x+ 170.

Em particular, b = 51 = (13 + i) + (3− 4i)(3 + 4i) + (13− i).

Exercıcios propostos

1. (Espanha) Sejam a, b, c numeros reais. Prove que se x3 + ax2 + bx + c possui tresraızes reais, entao 3b ≤ a2.

9

Page 65: Álgebra nível 3

POT 2012 - Algebra - Nıvel 3 - Aula 6 - Prof. Cıcero Thiago/ Prof. Marcelo

Mendes

2. (Espanha) Dado o polinomio p(x) = x3 +Bx2+Cx+D, prove que se o quadrado deuma de suas raızes e igual ao produto das outras duas, entao B3D = C3.

3. Seja f(x) um polinomio de grau n, n > 1, com coeficientes inteiros e n raızes reais,nem todas iguais, no intervalo (0, 1). Prove que se a e o coeficiente lıder de f(x),entao

|a| ≥ 2n + 1.

4. (Czech and Slovak) Seja a e b numeros reais. Prove que se a equacao

x4 − 4x3 + 4x2 + ax+ b = 0

possui duas raızes reais distintas tais que a soma e igual ao produto, entao a equacaonao possui outras raızes reais e, alem disso, a+ b > 0.

5. (Canada) O polinomio

P (x) = xn + a1xn−1 + a2x

n−2 + . . .+ an−1x+ an

com coeficientes inteiros a1, a2, . . . , an, e tal que existem quatro inteiros distintosa, b, c, d tais que

P (a) = P (b) = P (c) = P (d) = 5,

mostre que nao existe um inteiro k tal que P (k) = 8.

6. Sejam a e b duas raızes do polinomio x4 + x3 − 1. Prove que ab e uma raiz do po-linomio x6 + x4 + x3 − x2 − 1.

7. (Romenia) Sejam a, b, c, a 6= 0, tais que a e 4a+3b+2c tem o mesmo sinal. Mostreque a equacao ax2 + bx+ c = 0 nao pode ter duas raızes no intervalo (1, 2).

8. (OBM) a, b, c, d sao numeros reais distintos tais que a e b sao as raızes da equacaox2 − 3cx− 8d = 0, e c e d sao as raızes da equacao x2− 3ax− 8b = 0. Calcule a somaa+ b+ c+ d.

9. (TST Romenia) Sejam a, n numeros inteiros, e p um numero primo tal que p > |a|+1.Prove que o polinomio f(x) = xn+ax+p nao pode ser representado como o produtode dois polinomios com coeficientes inteiros.

10

Page 66: Álgebra nível 3

POT 2012 - Algebra - Nıvel 3 - Aula 6 - Prof. Cıcero Thiago/ Prof. Marcelo

Mendes

10. (Ibero) Ache as raızes r1, r2, r3 e r4 da equacao 4x4 −ax3+bx2−cx+5 = 0, sabendo

que as raızes sao reais positivas, a, b, c sao reais e que

r1

2+

r2

4+

r3

5+

r4

8= 1.

11. (Espanha) Sejam x1, x2 as raızes do polinomio P (x) = 3x2 + 3mx +m2 − 1, sendom um numero real. Prove que P (x31) = P (x32).

12. (Espanha) Prove que nao existem inteiros a, b, c e d tais que o polinomio P (x) =ax3 + bx2 + cx+ d, a 6= 0 satisfaz P (4) = 1 e P (7) = 2.

13. (OBM) Seja f(x) = x2+2007x+1. Prove que, para todo n inteiro positivo, a equacaof(f(. . . (f︸ ︷︷ ︸

n vezes

(x)) . . .)) tem pelo menos uma solucao real.

14. (TST Brasil) Sejam a, b, c, d numeros reais distintos tais que

a =

4 +√5 + a,

b =

4−√5 + b,

c =

4 +√5− c,

d =

4−√5− d.

Determine abcd.

15. Sejam a, b e c lados, com medidas inteiras, de um triangulo.(a) Prove que se a equacao

x2 + (a+ 1)x+ b− c = 0

possui raızes inteiras, entao o triangulo e isosceles.

(b) Prove que se a equacao

x2 + (2ab+ 1)x+ a2 + b2 − c2 = 0

possui raızes inteiras, entao o triangulo e retangulo.

11

Page 67: Álgebra nível 3

POT 2012 - Algebra - Nıvel 3 - Aula 6 - Prof. Cıcero Thiago/ Prof. Marcelo

Mendes

(c) Prove que se a equacao

x2 + (a2 + b2 + c2 + 1)x+ ab+ bc+ ac = 0

possui raızes inteiras, entao o triangulo e equilatero.

16. (AIME) Sejam a, b e c as raızes de x3 + 3x2 + 4x − 11 = 0 e a + b, b + c e c + a asraızes de x3 + rx2 + sx+ t = 0. Determine t.

17. (AIME) Considere os polinomios P (x) = x6−x5−x3−x2−x e Q(x) = x4−x3−x2−1.Dado que z1, z2, z3 e z4 sao as raızes de Q(x) = 0, ache P (z1)+P (z2)+P (z3)+P (z4).

18. (IMO Short List) Sejam a, b, c, d, e e f numeros inteiros positivos. Se S =a + b + c + d + e + f divide abc + def e ab + bc + ca − de − ef − fd. Prove queS e composto.

19. (Iugoslavia) Ache todos os racionais positivos a ≤ b ≤ c tais que os numeros

a+ b+ c,1

a+

1

b+

1

c, abc

sejam todos inteiros.

20. (Australia) Os polinomios x2 + x e x2 + 2 sao escritos em um quadro. Beatriz deveescrever no mesmo quadro a soma, a diferenca ou o produto de quaisquer dois po-linomios que estiverem escritos no quadro. Se Beatriz repetir o processo quantasvezes quiser, em algum momento ela conseguira chegar ao polinomio x?

21. (Czech and Slovak) Determine todos os numeros reais s tais que

4x4 − 20x3 + sx2 + 22x− 2 = 0

possui quatro raızes reais e distintas tais que o produto de duas dessas raızes seja −2.

22. (Romenia) Sejam a, b, c e d numeros reais tais que f : R → R, f(x) = ax3+bx2+cx+d

e f(2) + f(5) < 7 < f(3) + f(4). Prove que existem u, v ∈ R tais que u + v = 7 ef(u) + f(v) = 7.

12

Page 68: Álgebra nível 3

POT 2012 - Algebra - Nıvel 3 - Aula 6 - Prof. Cıcero Thiago/ Prof. Marcelo

Mendes

23. (Romenia) Seja P (x) = a1998x1998 + a1997x

1997 + . . . + a1x + a0 um polinomio comcoeficientes reais tal que P (0) 6= P (−1), e sejam a, b numeros reais. Seja Q(x) =b1998x

1998 + b1997x1997 + . . . + b1x + b0 um polinomio com coeficientes reais tal que

bk = aak + b, ∀k = 0, 1, . . . , 1998. Prove que se Q(0) = Q(−1) 6= 0, entao opolinomio Q nao possui raızes reais.

24. Determine todos os polinomios satisfazendo a equacao polinomial (x + 1)P (x) =(x− 10)P (x+ 1).

25. Determine todos os polinomios P (x) com coeficientes reais para os quais existe uminteiro positivo n tal que para todo x,

P

(

x+1

n

)

+ P

(

x− 1

n

)

= 2P (x).

Solucoes

1. Sem perda de generalidade sejam α ≥ β ≥ γ as raızes, entao:

x3+ax2+bx+c = (x−α)(x−β)(x−γ) = x3+(−α−β−γ)x2+(αβ+αγ+βγ)x−αβγ.

Temos que a = −α− β − γ e b = αβ + αγ + βγ. Portanto,

a2 − 3b = (−α− β − γ)2 − 3(αβ + αγ + βγ) = α2 + β2 + γ2 − αβ − αγ − βγ

=1

2[(α− β)2 + (α− γ)2) + (β − γ)2 ≥ 0.

2. Sejam r, s, t as raızes, entao 0 polinomio pode ser escrito da seguinte forma

p(x) = (x− r)(x− s)(x− t) = x3 − (r + s+ t)x2 + (rs+ st+ tr)x− rst.

Igualando os coeficientes, temos:

r + s + t = −B

rs + st + tr = C

rst = −D

Como r2 = st, temos que{

C = rs + r2 + tr = r(r + s+ t) = −rB

−D = rst = r3

Finalmente,C3 = (−rB)3 = −r3B3 = B3D.

13

Page 69: Álgebra nível 3

POT 2012 - Algebra - Nıvel 3 - Aula 6 - Prof. Cıcero Thiago/ Prof. Marcelo

Mendes

3. Seja f(x) = a(x−x1)(x−x2) . . . (x−xn), em que x1, x2, . . . , xn sao as raızes reais,nem todas iguais, pertencentes ao intervalo (0, 1). Como os coeficientes sao inteirosentao f(0) e f(1) sao numeros inteiros. Assim,

f(0) = a(0 − x1)(0− x2) . . . (0− xn) = a(−1)nx1x2 . . . xn

e

f(1) = a(1− x1)(1− x2) . . . (1− xn).

Por outro lado, |f(0) ·f(1)| = |a2(−1)nx1(1−x1)x2(1−x2) . . . xn(1−xn)| = a2x1(1−x1)x2(1 − x2) . . . xn(1 − xn) ≥ 1, pois f(0) e f(1) sao numeros inteiros. Mas, se

0 < x < 1, entao x(1 − x) ≤ 1

4, com igualdade acontecendo se, e somente se, x =

1

4(MA ≥ MG).

Como as raızes nao sao todas iguais, nao teremos igualdade acontecendo em x1(1 −x1)x2(1− x2) . . . xn(1− xn) ≤ a2

(

1

4

)n

, portanto:

1 ≤ a2x1(1− x1)x2(1− x2) . . . xn(1− xn) < a2(

1

4

)n

22n < a2 ⇔ |a| ≥ 2n + 1,

pois a e um numero inteiro.

4. Sejam x1 e x2 raızes reais e distintas de x4 − 4x3 + 4x2 + ax + b = 0 tais quex1 + x2 = x1x2 = p, entao:

x4 − 4x3 + 4x2 + ax+ b = (x2 − px+ p)(x2 + rx+ s), (1)

em que r e s sao numeros reais. A igualdade (1) garante que:

−4 = −p+ r, (2)

4 = p+ s− pr, (3)

a = −ps+ pr, (4)

b = ps. (5)

Da equacao (2) temos que r = p− 4, (6). Substituindo (6) em (3) temos

s = 4− p+ p(p− 4) = (p− 4)(p − 1). (7)

14

Page 70: Álgebra nível 3

POT 2012 - Algebra - Nıvel 3 - Aula 6 - Prof. Cıcero Thiago/ Prof. Marcelo

Mendes

Mas a equacao quadratica x2 − px+ p = 0 possui raızes reais e distintas x1 e x2, comisso seu discriminante e positivo, ou seja

p2 − 4p > 0. (8)

Adicionando as equacoes (4) e (5) e substituindo r em (6), temos

a+ b = pr = p(p− 4) = p2 − 4p > 0.

Seja D o discriminante da equacao

x2 + rx+ s = 0.

Das equacoes (6), (7) e (8) temos

D = r2 − 4s = (p− 4)2 − 4(p− 4)(p − 1) = −3p(p− 4) = −3(p2 − 4p) < 0.

Portanto, a equacao possui apenas duas raızes reais.

5. Seja M(x) = P (x)− 5 temos que:

M(x) = (x− a)(x− b)(x− c)(x− d)Q(x)

e o polinomio Q(x) com coeficientes inteiros. Vamos admitir a existencia de uminteiro k tal que P (k) = 8 ⇒ M(k) = 3. Dessa forma,

3 = (k − a)(k − b)(k − c)(k − d)Q(k).

Mas essa ultima igualdade nao pode existir pois 3 nao pode ser escrito como o pro-duto de pelo menos 4 inteiros distintos. Portanto, nao existe tal k.

6. Sejam c e d as outras duas raızes de x4 + x3 − 1. Pelas relacoes de Girard, temos:

a+ b+ c+ d = −1,

ab+ ac+ ad+ bc+ bd+ cd = 0,

abc+ abd+ acd+ bcd = 0,

abcd = −1.

Fazendo m = a+ b, n = c+ d, r = ab e s = cd, entao

m+ n = −1, (1)

r + s+mn = 0, (2)

15

Page 71: Álgebra nível 3

POT 2012 - Algebra - Nıvel 3 - Aula 6 - Prof. Cıcero Thiago/ Prof. Marcelo

Mendes

rn+ sm = 0, (3)

rs = −1. (4)

Fazendo s = −1

re n = −1−m em (2) e (3), respectivamente, temos

r − 1

r−m2 −m = 0, (5)

er(−1−m)− m

r= 0. (6)

De (6) encontramos m = − r2

r2 + 1. Substituindo em (5), temos

r − 1

r− r4

(r2 + 1)2+

r2

r2 + 1= 0 ⇔

r6 + r4 + r3 − r2 − 1 = 0,

ou seja, r = ab e uma raiz de x6 + x4 + x3 − x2 − 1.

7. Temos que

0 ≤ 4a+ 3b+ c

a= 4+3

b

a+2

c

a= 2x1x2−3(x1+x2)+4 = (x1−1)(x2−2)+(x1−2)(x2−1).

Se x1 e x2 pertencerem ao intervalo (1, 2), entao cada termo da soma acima seraestritamente negativo, o que e uma contradicao.

8. E facil perceber que a + b = 3c e que c + d = 3a. Somando e subtraindo membro amembro as duas igualdades obteremos b+ d = 2 (a+ c) e b− d = 4 (c− a). Como a

e raiz de x2 − 3cx− 8d = 0, segue que

a2 − 3ac− 8d = 0 (1).

Do mesmo modo, como c e raiz de x2 − 3ax− 8b = 0, temos que

c2 − 3ac− 8d = 0 (2).

Subtraindo as igualdades (1) e (2) e utilizando as relacoes anteriormente obtidas,vem:a2 − c2 = 8(d − b) ⇒ (a− c)(a+ c) = 8× 4(a − c). Como a− c 6= 0, concluımos quea+ c = 32.Portanto, a+ c = 32 e b+ d = 2(a+ c) = 64, donde a+ b+ c+ d = 96.

16

Page 72: Álgebra nível 3

POT 2012 - Algebra - Nıvel 3 - Aula 6 - Prof. Cıcero Thiago/ Prof. Marcelo

Mendes

9. Seja z uma raiz complexa do polinomio. Vamos provar que |z| > 1. Suponha que|z| ≤ 1, entao zn + az = −p, entao:

p = |zn + az| = |z||zn−1 + a| ≤ |zn−1|+ |a| ≤ 1 + |a|,

contrariando o fato que p > |a|+ 1. Agora, seja f(x) = g(x)h(x) uma decomposicaode f(x) em polinomios com coeficientes inteiros entao p = g(0)h(0), entao |g(0)| = 1ou |h(0)| = 1. Suponha que |g(0)| = 1. Se z1, z2, . . . , zk sao raızes de g(x) entaosao tambem raızes de f(x), assim:

1 = |g(0)| = |z1z2 . . . zk| = |z1||z2| . . . |zk| >,

que e uma contradicao.

10. Sejam r1, r2, r3 e r4 as raızes da equacao, entao:

r1 · r2 · r3 · r4 =5

4.

Usando a desigualdade entre as medias aritmetica e geometrica, temos que:

r1

2+

r2

4+

r3

5+

r4

84

≥ 4

r1

2· r24

· r35

· r48

1

4≥ 1

4,

ou seja, aconteceu a igualdade entre as medias, portanto:

r1

2=

r2

4=

r3

5=

r4

8=

1

4⇔

r1 =1

2, r2 = 1, r3 =

5

4, r4 = 2.

11. Temos que x1 + x2 = −m e x1 · x2 =m2 − 1

3e

P (x31)− P (x32) = 3x61 + 3mx31 +m2 − 1− (3x62 + 3mx32 +m2 − 1)

= 3(x61 − x62) + 3m(x31 − x32)

= 3(x31 + x32)(x31 − x32) + 3m(x31 − x32)

3(x31 − x32)(x31 + x32 +m).

Mas x31 + x32 = (x1 + x2)3 − 3x1x2(x1 + x2) = (−m)3 − 3

m2 − 1

3(−m) = −m. Entao,

x31 + x32 +m = 0 ⇒ P (x31) = P (x32).

17

Page 73: Álgebra nível 3

POT 2012 - Algebra - Nıvel 3 - Aula 6 - Prof. Cıcero Thiago/ Prof. Marcelo

Mendes

12. Vamos admitir a existencia do polinomio. Pelo teorema do resto P (x) = (x −4)Q(x) + 1, sendo Q(x) um polinomio de grau 2 com coeficientes inteiros. Entao

P (7) = 2 = (7 − 4)Q(7) + 1 ⇒ Q(7) =1

3, que nao e inteiro, contrariando o fato que

Q(x) possui coeficientes inteiros.

13. Sejam f1(x) = f(x) e para cada n ≥ 1, fn+1(x) = f(fn(x)). Sejam ∆1 = 20072 − 4,

x1 =−2007 +

√∆1

2. Temos f(x1 = 0. Vamos mostrar por inducao que existe uma

sequencia de reais positivos (∆n) tal que, definindo xn =−2007 +

√∆n

2, temos

f(xn+1) = xn, para todo n, donde fn+1(xn+1) = fn(xn) = 0. Para isso, note que a

maior raiz de x2+2007x+1 = xn e−2007 +

√∆n+1

2, onde ∆n+1 = 20072−4+4xn =

20072 − 4018 + 2√∆n > 0.

14.

a =

4 +√5 + a ⇒

a2 = 4 +√5 + a ⇒

a2 − 4 =√5 + a ⇒

a4 − 8a2 − a+ 11 = 0.

Analogamente,b4 − 8b2 − b+ 11 = 0,

c4 − 8c2 + c+ 11 = 0,

d4 − 8d2 + d+ 11 = 0.

Seja f(x) = x4 − 8x2 − x+ 11 e g(x) = x4 − 8x2 + x+ 11. Entao a e b sao raızes def(x) = 0, e c e d sao raızes de g(x) = 0. Mas f(−x) = x4 − 8x2 + x + 11 = g(x).Entao, as raızes de f(x) = 0 sao a, b, −c e −d. Portanto, pelas relacoes de Girardtemos que ab(−c)(−d) = abcd = 11.

15. (a) Se b > c entao (a + 1)2 − 4(b − c) e um quadrado perfeito menor e de mesmaparidade que (a+ 1)2. Portanto,

(a+ 1)2 − 4(b− c) ≤ (a− 1)2

assim a+ c ≤ b, contrariando a desigualdade triangular. O caso b < c e analogo.

18

Page 74: Álgebra nível 3

POT 2012 - Algebra - Nıvel 3 - Aula 6 - Prof. Cıcero Thiago/ Prof. Marcelo

Mendes

(b) Se a2 + b2 > c2 entao (2ab + 1)2 − 4(a2 + b2 − c2) e o quadrado de um numeroımpar menor que 2ab+ 1. Portanto,

(2ab+ 1)2 − 4(a2 + b2 − c2) ≤ (2ab− 1)2,

assim c2 ≤ (a− b)2, contradicao. O caso c2 > a2 + b2 e analogo.

(c) Se a equacao tem raızes inteiras, entao (a2 + b2 + c2 + 1)2 − 4(ab+ bc+ ac) e umquadrado perfeito menor e de mesma paridade que (a2 + b2 + c2 + 1)2. Portanto,

(a2 + b2 + c2 + 1)2 − 4(ab + bc+ ac) ≤ (a2 + b2 + c2 + 1)2 ⇔

(a− b)2 + (b− c)2 + (c− a)2 ≤ 0 ⇔a = b = c.

16. A primeira equacao garante que a + b + c = −3. A segunda equacao garanteque t = −(a + b)(b + c)(c + a). Segue que t = −(3 − c)(3 − a)(3 − b), ou seja,t = 27 + 9(a + b + c) + 3(ab + ac + bc) + abc. A primeira equacao garante queab+ bc+ ca = 4 e abc = 11, entao t = 27− 27 + 12 + 11 = 23.

17. Usando o algoritmo da divisao temos que P (x) = Q(x)(x2+1)+x2−x+1. Portanto,

4∑

i=1

P (zi) =4∑

i=1

z2i −4∑

i=1

zi + 4 =

(

4∑

i=1

zi

)2

− 2∑

i<j

zizj −4∑

i=1

zi + 4.

Usando as relacoes de Girard, temos que

4∑

i=1

P (zi) = 1 + 2− 1 + 4 = 6.

18. Todos os coeficientes do polinomio

f(x) = (x+ a)(x+ b)(x+ c)− (x− d)(x − e)(x− f)

= Sx2 + (ab+ bc+ ca− de− ef − fd)x+ (abc+ def)

sao multiplos de S. Entao, f(d) = (a + d)(b + d)(c + d) e um multiplo de S. Istoimplica que S e composto pois a+ d, b+ d e c+ d sao menores que S.

19. Seja P (x) = (x − a)(x − b)(x − c) = x3 − (a + b + c)x2 + (ab + bc + ca)x − abc

um polinomio cujas raızes sao a, b e c. Mas ab + bc + ca = abc

(

1

a+

1

b+

1

c

)

∈ Z,

portanto P (x) possui coeficientes inteiros. Pelo teorema das raızes racionais temosque a, b e c sao inteiros. Sabemos que a, b e c sao positivos entao nosso problema

e achar todos os valores de a, b e c tais que1

a+

1

b+

1

ce natural. As solucoes sao:

(a, b, c) = (1, 1, 1); (1, 2, 2); (3, 3, 3); (2, 4, 4); (2, 3, 6).

19

Page 75: Álgebra nível 3

POT 2012 - Algebra - Nıvel 3 - Aula 6 - Prof. Cıcero Thiago/ Prof. Marcelo

Mendes

20. Sejam P (x) = x2 + x e Q(x) = x2 + 2. E facil ver que P (2) = Q(2) = 6. Suponhaque A(x) e B(x) sao dois polinomios quaisquer tais que A(2) e B(2) sao divisıveispor 6. Segue que A(2) + B(2), A(2) − B(2) e A(2) · B(2) sao todos divisıveis por 6.Portanto, qualquer polinomio R(x) gerado tem R(2) divisıvel por 6. Como T (x) =tem T (2) = 2 entao T (x) = x nunca aparecera no quadro.

21. Sejam x1, x2, x3 e x4 as raızes tais que x1x2 = −2, (0). Pelas relacoes de Girardtemos que:

x1 + x2 + x3 + x4 = 5, (1)

x1x2 + x1x3 + x1x4 + x2x3 + x2x4 + x3x4 =s

4, (2)

x1x2x3 + x1x2x4 + x1x3x4 + x2x3x4 = −11

2, (3)

x1x2x3x4 = −1

2. (4)

Das equacoes (0) e (4) temos que

x3x4 =1

4.

Fatorando a equacao (3) e, em seguida, substituindo os valores de x1x2 e x3x4 temos:

(x1 + x2)x3x4 + (x3 + x4)x1x2 = −11

2⇔

1

4(x1 + x2)− 2(x3 + x4) = −11

2.

A ultima equacao e (1) garantem que x1+x2 = 2 e x3+x4 = 3. Fatorando a equacao(2) e substituindo os valores ja encontrados temos que

x1x2 + (x1 + x2)(x3 + x4) + x3x4 =s

4⇔

s = 17.

As raızes do polinomio sao x1,2 = 1±√3 e x3,4 =

3

√2.

22. Precisamos provar que f(x) + f(7− x) = 7 possui solucao. Seja g(x) = f(x) + f(7−x)− 7. E facil ver que g e um polinomio com grau no maximo 2. Temos que g(2) < 0e g(3) > 0. Segue que g(x) = 0 possui pelo menos uma raiz real.

20

Page 76: Álgebra nível 3

POT 2012 - Algebra - Nıvel 3 - Aula 6 - Prof. Cıcero Thiago/ Prof. Marcelo

Mendes

23. E facil ver que Q(x) = aP (x) + b(x1998 + x1997 + . . . + 1). De Q(0) = Q(−1) temosque a(P (0) − P (−1)) = 0, entao a = 0. Com isso, Q(x) = b(x1998 + x1997 + . . . + 1,com b 6= 0 pois Q(0) 6= 0. E facil ver que Q(x) nao possui raızes positivas. Para x ≤ 1temos

x1998 + x1997 + . . .+ 1 = x1997(x+ 1) + x1995(x+ 1) + . . .+ x(x+ 1) + 1 ≥ 1,

e para x ∈ (−1, 0),

x1998 + x1997 + . . .+ 1 = x1998 + x1996(x+ 1) + . . .+ x2(x+ 1) + x+ 1 > 0.

Conclusao: Q(x) nao possui raızes reais.

24. Temos que (x+1)P (x) = (x− 10)P (x+1) mostra que P (x) e divisıvel por (x− 10).Trocando x + 1 por x na igualdade inicial temos que xP (x − 1) = (x − 11)P (x), ouseja, P (x) e divisıvel por x. Entao P (x) = x(x− 10)P1(x). Substituindo na equacaooriginal e cancelando os termos temos que

xP1(x) = (x− 9)P1(x+ 1).

Repetindo o argumento encontramos P1(x) = (x − 1)(x − 9)P2(x) e P (x) = x(x −1)(x − 2) . . . (x − 10)Q(x), em que Q(x) = Q(x + 1). Segue que Q(x) e constante, ea solucao do problema e

P (x) = ax(x− 1)(x− 2) . . . (x− 10),

em que a e uma constante arbitraria.

25. Seja m o grau do polinomio P (x), entao

P (x) = amxm + am−1xm−1 + . . .+ a0.

Usando binomio de Newton para

(

x± 1

n

)m

e

(

x± 1

n

)m−1

temos que

2amxm + 2am−1xm−1 + 2am−2x

m−2 + amm(m− 1)

n2xm−2 +Q(x)

= 2amxm + 2am−1xm−1 + 2am−2x

m−2 +R(x),

em que Q e R sao polinomios de grau no maximo m− 3. Comparando os coeficientes

encontramos amm(m− 1)

n2= 0. Mas am 6= 0, pois e o coeficiente lıder do polinomio,

entao m(m− 1) = 0, ou seja, m = 0 ou m = 1. Uma simples verificacao garante quetodos os polinomios de grau 0 ou 1 satisfazem a condicao inicial.

21

Page 77: Álgebra nível 3

POT 2012 - Algebra - Nıvel 3 - Aula 6 - Prof. Cıcero Thiago/ Prof. Marcelo

Mendes

Bibliografia

1. Topicos de Matematica Elementar, vol. 6PolinomiosAntonio Caminha Muniz NetoSBM

2. Intermediate Algebrathe Art of Problem SolvingRichard Rusczyk e Mathew Crawford

3. Equations and InequalitiesElementary Problems and Theorems in Algebra and Number TheoryJiri Herman, Radan Kucera e Jaromir Simsa

4. Fundamentos de Matematica Elementar, vol. 6Complexos, Polinomios e EquacoesGelson Iezzi

5. Equations and InequalitiesMIR Publishers MoscowV. V. Vavilov, I. I. Melnikov, S. N. Olekhnik e P. I. Pasichenko

6. The Ussr Olympiad Problem BookSelected Problems and Theorems of Elementary MathematicsD. O. Shklarsky, N. N. Chentzov e I. M. Yaglom

7. Mathematical Olympiad TreasuresTitu Andreescu e Bogdan Enescu

8. Lecture Notes on Mathematical Olympiad CoursesFor Junior Section, vol. 1Xu Jiagu

9. Lecture Notes on Mathematical Olympiad CoursesFor Junior Section, vol. 2Xu Jiagu

10. Problem - Solving Through ProblemsLoren C. Larson

11. Kvant Selecta: Algebra and Analysis, ISerge Tabachnikov

12. Kvant Selecta: Algebra and Analysis, II

22

Page 78: Álgebra nível 3

POT 2012 - Algebra - Nıvel 3 - Aula 6 - Prof. Cıcero Thiago/ Prof. Marcelo

Mendes

Serge Tabachnikov

13. Bulgarian Mathematics Competition, 1999 - 2001BJ Lazarov, JB Tabov, PJ Taylor e AM StorozhevAMT

14. Australian Mathematical Olympiads, 1996 - 2011H Lausch, A Di Pasquale, DC Hunt e PJ TaylorAMT

15. Mathematical Competitions - Baltic Way - 1990 - 2005

16. Olimpıadas Cearenses de Matematica, Ensino Medio, 1981 - 2005Emanuel Carneiro, Francisco Antonio M. de Paiva e Onofre Campos

17. 101 Problems in Algebra: From the training of the USA IMO team.Titu AndreescuAMT publishing

18. Winning SolutionsCecil Rousseau e Edward Lozansky

19. Mathematical Olympiad TreasuresTitu Andreescu e Bogdan EnescuBirkhauser

20. Tournament of Towns - 1993 - 1997 (Book 4)PJ Taylor e AM StorozhevAMT

21. Olimpiada Matematica Espanola15000 Problemas de diferentes Olimpiadas de Matematica en el Mundo

22. 360 Problems for Mathematical ContestsTitu Andreescu e Dorin AndricaGIL

23. International Mathematical Talent SearchPart 1G BerzsenyiAMT

24. International Mathematical Talent SearchPart 1

23

Page 79: Álgebra nível 3

POT 2012 - Algebra - Nıvel 3 - Aula 6 - Prof. Cıcero Thiago/ Prof. Marcelo

Mendes

G BerzsenyiAMT

25. Putnam and BeyondRazvan Gelca e Titu AndreescuSpringer

26. A problem book in algebraV. A. Krechmar

27. Matematica em nıvel IME - ITANumeros Complexos e PolinomiosCaio dos Santos Guimaraes

28. A Matematica do ensino medio, vol. 4Enunciados e solucoes dos exercıciosElon Lages Lima, Paulo Cezar Pinto Carvalho, Eduardo Wagner e Augusto Cesar Morgado

29. Curso de Algebra, Vol. 1.Abramo HefezIMPA

30. Problem - Solving StrategiesArthur EngelSpringer

31. Problems from the bookTitu Andreescu e Gabriel DospinescuXYZ Press

24

Page 80: Álgebra nível 3

Polos Olímpicos de TreinamentoCurso de Álgebra - Nível 3Prof. Cícero Thiago / Prof. Marcelo Mendes

Aula 7

Miscelanea sobre raızes de polinomios II

Definicao 1: Seja P (x) = anxn + an−1x

n−1 + . . . + a1x+ a0 um polinomio com an 6= 0 en > 0. Definiremos P ′(x) = nanx

n−1+(n− 1)an−1xn−2+ . . .+ a1 como sendo o polinomio

que e a derivada do polinomio P (x). As derivadas dos polinomios P (x) e Q(x) satisfazem(1) P (x) = k, k constante ⇒ P ′(x) = 0.(2) (P +Q)′(x) = P ′(x) +Q′(x).(3) (P −Q)′(x) = P ′(x)−Q′(x).(4) (P ·Q)′(x) = P ′(x)Q(x) + P (x)Q′(x).

Como consequencia de (3) temos que se x1, x2, . . ., xn sao raızes de, um polinomio de graun, P (x), entao

P ′(x)

P (x)=

1

x− x1+

1

x− x2+ . . .+

1

x− xn.

Em seguida, um teorema bem interessante sobre raızes multiplas de um polinomio.

Teorema 1. Se r e raiz de multiplicidade m do polinomio P (x), entao r e raiz de multi-plicidade m− 1 do polinomio P ′(x).

Demonstracao. Temos que P (x) = (x − r)m · Q(x) ⇒ P ′(x) = m(x − r)m−1Q(x) + (x −r)mQ′(x) = (x−r)m−1[m·Q(x)+(x−r)·Q′(x)] e, comom·Q(r)+(r−r)·Q′(r) = m·Q(r) 6= 0,ou seja, r e uma raiz de multiplicidade m− 1 de P ′(x).

1. Determine um polinomio P (x), de grau 5, tal que P (x) + 1 e divisıvel por (x− 1)3 eP (x)− 1 e divisıvel por (x+ 1)3.

Solucao. Se 1 e uma raiz de multiplicidade 3 de P (x) entao 1 e raiz de multiplicidade2 do polinomio P ′(x). Da mesma forma −1 e uma raiz de multiplicidade 2 de P ′(x).Segue que P ′(x) e divisıvel pelo polinomio (x−1)2(x+1)2. Mas, P ′(x) e um polinomiode grau 4. Entao,

P ′(x) = c(x− 1)2(x+ 1)2 = c(x4 − 2x2 + 1),

Page 81: Álgebra nível 3

POT 2012 - Algebra - Nıvel 3 - Aula 7 - Prof. Cıcero Thiago/ Prof. Marcelo

Mendes

para alguma constante c. Agora, P (x) = c ·(

1

5x5 − 2

3x3 + x

)

+ d, para c e d reais.

Como P (−1) = 1 e P (1) = −1, entao c = −15

8e d = 0 e P (x) = −3

8x5 +

5

4x3 − 15

8x.

Vamos ver uma outra solucao.Note que (x− 1)3 divide P (x) + 1 e P (−x)− 1, entao (x− 1)3 divide P (x) +P (−x).Alem disso, (x+1)3 divide P (x)−1 e P (−x)+1, entao (x−1)3 divide P (x)+P (−x).Dessa forma, (x−1)3(x+1)3 divide P (x)+P (−x), que e um polinomio de grau 5, as-sim P (x)+P (−x) = 0, ∀x. Portanto, os coeficientes dos termos de grau par de P (x)sao iguais a zero. Agora, P (x)+1 = (x− 1)3(Ax2+Bx− 1). Com isso, B− 3A = 0 e

3+3B−A = 0 , ou seja, A = −3

8e B = −9

8. Finalmente, P (x) = −3

8x5+

5

4x3− 15

8x.

2. Sejam x1, x2, . . . , xn−1, as raızes diferentes de 1 do polinomio P (x) = xn−1, n ≥ 2.Prove que

1

x− x1+

1

x− x2+ . . . +

1

x− xn−1=

n− 1

2.

Solucao. Seja R(x) um polinomio de grau n− 1, cujas raızes sao x1, x2, . . . , xn−1.Segue que

R′(x)

R(x)=

1

x− x1+

1

x− x2+ . . .+

1

x− xn−1.

Por outro lado, R(x) =xn − 1

x− 1= xn−1 + xn−2 + . . . + x + 1, entao R(1) = n e

R′(1) = (n− 1) + (n − 2) + . . . + 1 =n(n− 1)

2. Dessa forma,

1

1− x1+

1

1− x2+ . . .+

1

1− xn−1=

R′(1)

R(1)=

n− 1

2.

3. Prove que o polinomio P (x) = 1 +x

1+

x2

2!+ . . .+

xn

n!nao possui raızes multiplas.

Solucao. O polinomio P possui uma raiz multipla r se P (r) = P ′(r) = 0. Mas

P (x) = P ′(x)+xn

n!. Dessa forma, se r for uma raiz entao P (r) = P ′(r)+

rn

n!⇔ r = 0.

Por outro lado, P (0) = 1. Assim, P nao possui raızes multiplas.

4. Determine a para que −1 seja uma raiz multipla de P (x) = x5 − ax2 − ax+ 1.

Solucao. Temos que P (−1) = −1− a+ a+ 1 = 0. Mas, P ′(−1) = 0 ⇒ 5 + 2a− a =0 ⇔ a = −5.

2

Page 82: Álgebra nível 3

POT 2012 - Algebra - Nıvel 3 - Aula 7 - Prof. Cıcero Thiago/ Prof. Marcelo

Mendes

5. Prove que (x− 1)2|nxn+1 − (n+ 1)xn + 1.

Solucao. Temos que P (1) = n− (n+ 1) + 1 = 0 e P ′(1) = n(n + 1)− (n + 1)n = 0.Portanto, 1 e raiz com multiplicidade 2.

Exercıcios propostos

1. Sejam x1, x2, . . ., xn as raızes do polinomio xn + xn−1 + . . .+ x+ 1. Prove que

1

1− x1+

1

1− x2+ . . . +

1

1− xn=

n

2.

2. Demonstre que, se a equacao x3 − ax+ b = 0 (ab 6= 0), com a, b reais, tiver uma raizdupla, entao a sera sempre positivo.

3. (ITA) Seja k ∈ R tal que a equacao 2x3 + 7x2 + 4x+ k = 0 possua uma raiz dupla einteira x1 e uma raiz x2, distinta de x1. Entao, (k + x1)x2 e igual a(a) −6. (b) −3. (c) 1. (d) 2. (e) 8.

4. Prove que (x+ 1)2|x4n+2 + 2x2n+1 + 1.

5. Determine todos os polinomios P (x), com coeficientes inteiros, que satisfazem P (P ′(x)) =P ′(P (x)), ∀x ∈ R.

6. Se a equacao x3 + ax2 + 3x+ 1 = 0 tem raiz tripla, qual o valor de a?

7. Sejam P (z) e Q(z) polinomios com coeficientes complexos, de grau maior ou igual a1, tais que P (z) = 0 se, e somente se, Q(z) = 0 e P (z) = 1 se, e somente se, Q(z) = 1.Prove que os polinomios sao iguais.

Solucoes/Sugestoes

1. Vamos fazer uma solucao com uma ideia diferente das que foram trabalhadas nessaaula. Observe o polinomio com raızes

yk =1

1− xk, k = 1, 2, . . . , n.

Da igualdade acima temos que

xk =yk − 1

yk,

3

Page 83: Álgebra nível 3

POT 2012 - Algebra - Nıvel 3 - Aula 7 - Prof. Cıcero Thiago/ Prof. Marcelo

Mendes

em que xk e uma raiz de xn + xn−1 + . . .+ x+ 1, dessa forma

(

yk − 1

yk

)n

+

(

yk − 1

yk

)n−1

+ . . .+yk − 1

yk+ 1 = 0.

A ultima igualdade e equivalente a

(yk − 1)n + yk(yk − 1)n−1 + . . .+ yn−1k

(yk − 1) + ynk = 0.

Segue que yk e uma raiz do polinomio

P (x) = (x− 1)n + x(x− 1)n−1 + . . .+ xn−1(x− 1) + xn.

Queremos calculary1 + y2 + . . . + yn.

Observe que

P (x) = (n+ 1)xn − xn−1

((

n

1

)

+

(

n− 1

1

)

+ . . . +

(

1

1

))

+ . . . .

Usando relacoes de Girard, temos que

y1 + y2 + . . .+ yn =

(

n

1

)

+(

n−11

)

+ . . . +(

11

)

n+ 1=

n(n+ 1)

2(n + 1)=

n

2.

4. Seja P (x) = x4n+2 + 2x2n+1 + 1. Entao, P (−1) = 1 − 2 + 1 = 0 e P ′(1) =−(4n+ 2) + 2(2n + 1) = 0.

5. Vamos primeiro considerar o caso em que n ≥ 2. Seja P (x) = anxn + an−1x

n−1 +. . .+ a0, an 6= 0. Entao

P ′(x) = nanxn−1 + (n− 1)an−1x

n−2 + . . .+ a1.

Fazendo a identidade dos coeficientes de xn(n−1) na igualdade P (P ′(x)) = P ′(P (x)),obtemos

an+1n · nn = ann · n.

Isto implica que annn−1 = 1, ou seja, an =

1

nn−1. Como an deve ser inteiro, entao

n = 1, o que e uma contradicao. Se n = 1, entao P (x) = ax+ b. Dessa forma, temosque a2 + b = a ⇔ b = a − a2. A resposta do problema sao todos os polinomios daforma P (x) = ax2 + a− a2.

4

Page 84: Álgebra nível 3

POT 2012 - Algebra - Nıvel 3 - Aula 7 - Prof. Cıcero Thiago/ Prof. Marcelo

Mendes

Bibliografia

1. Problem - Solving StrategiesArthur Engel

2. Putnam and BeyondRazvan Gelca e Titu Andreescu

3. Fundamentos de Matematica Elementar, vol.6Gelson Iezzi

4. 101 Problems in Algebra: Form the training of the USA IMO team.Titu Andreescu e Zuming Feng

5. Mathematical Olympiad TreasuresTitu Andreescu e Bogdan Enescu

6. Topicos de Matematica Elementar, vol. 6Antonio Caminha Muniz Neto

5

Page 85: Álgebra nível 3

Polos Olímpicos de TreinamentoCurso de Álgebra - Nível 3Prof. Cícero Thiago / Prof. Marcelo Mendes

Aula 8

Aplicacoes de raızes da unidade

Nesta aula vamos nos concentrar nas raızes do polinomio

P (x) = xn − 1,

em que n e um inteiro positivo. As raızes desse polinomio sao chamadas de raızes n -

esimas da unidade. E facil ver que 1 e uma raiz desse polinomio. Mas quais sao asoutras? Quais as aplicacoes legais dessas raızes?

Teorema 1. As raızes de P (x) = xn − 1 sao

ωk = cos

(

2kπ

n

)

+ i · sin(

2kπ

n

)

, 0 ≤ k < n, k ∈ Z.

Demonstracao. Use o Teorema 2 da aula 5.

Dessa formaω0 = cos 0 + i · s sin 0 = 1;

ω1 = cos2π

n+ i · sin 2π

n= ω;

ω2 = cos4π

n+ i sin ·4π

n= ω2;

. . .

ωn−1 = cos2(n − 1)π

n+ i · sin 2(n − 1)π

n= ωn−1.

As raızes n - esimas da unidade determinam um polıgono regular inscrito em um cırculounitario tal que um dos vertices e o ponto (1, 0).

Para n = 3, as raızes cubicas da unidade sao

ωk = cos2kπ

3+ i · sin 2kπ

3, k ∈ {0, 1, 2},

Page 86: Álgebra nível 3

POT 2012 - Algebra - Nıvel 3 - Aula 8 - Prof. Cıcero Thiago/ Prof. Marcelo

Mendes

ou seja,

ω0 = 1, ω1 = cos2π

3+ i · sin 2π

3= −1

2+ i ·

√3

2= ω

e

ω2 = cos4π

3+ i · sin 4π

3= −1

2− i ·

√3

2= ω2.

As raızes cubicas da unidade determinam um triangulo equilatero inscrito em um cırculocom centro em (0, 0) e raio 1, como podemos ver na figura a seguir.

y

x1

ω

ω2

bO

b

b

b

Exercıcios resolvidos

1. Prove que 1 + ω + ω2 + ω3 + . . .+ ωn−1 = 0 em que 1, ω, ω2, ω3, . . . , ωn−1 sao asraızes n - esimas da unidade.

Solucao.

Temos que

1 + x+ x2 + x3 + . . . + xn−1 =xn − 1

x− 1.

Assim, fazendo x = ω, temos 1 + ω + ω2 + ω3 + . . . + ωn−1 =ωn − 1

ω − 1= 0.

2. Calcule(2000

2

)

+(2000

5

)

+(2000

8

)

+ . . .+(20002000

)

.

Solucao.

Seja

f(x) = (1 + x)2000 =

2000∑

k=0

(

2000

k

)

xk.

2

Page 87: Álgebra nível 3

POT 2012 - Algebra - Nıvel 3 - Aula 8 - Prof. Cıcero Thiago/ Prof. Marcelo

Mendes

Seja ω = −1

2+ i ·

√3

2. Entao, ω3 = 1 e ω2 + ω + 1 = 0. Entao

3

((

2000

2

)

+

(

2000

5

)

+

(

2000

8

)

+ . . .+

(

2000

2000

))

= f(1) + ωf(ω) + ω2f(ω2)

= 22000 + ω(1 + ω)2000 + ω2(1 + ω2)2000

= 22000 + ω(−ω2)2000 + ω2(−ω)2000

22000 + ω2 + ω = 22000 − 1.

Dessa forma o valor da soma e22000 − 1

3.

3. (Leningrado) Dizemos que uma sequencia a0, a1, . . . , an de numeros reais e p -balanceada para algum inteiro positivo p se

a0 + ap + a2p + . . . = a1 + ap+1 + a2p+1 + . . . = . . . =

= ap−1 + a2p−1 + a3p−1 . . . .

Se a sequencia a0, a1, . . . , a49 e p - balanceada para p = 3, 5, 7, 11, 13, 17, proveque a0 = a1 = . . . = a49 = 0.

Solucao. Suponha que a sequencia a0, a1, . . . , an e p - balanceada para algum p eseja S o valor das somas

a0 + ap + a2p + . . . , a1 + ap+1 + a2p+1 + . . . , . . . , ap−1 + a2p−1 + a3p−1 + . . . .

O proximo passo e introduzir o polinomio P (x) = anxn + an−1x

n−1 + . . .+ a1x+ a0.Seja ω uma raiz p - esima da unidade (ω 6= 1), ou seja, ωp = 1. Assim, ωr = ωp+r =ω2p+r = ω3p+r = . . ., para cada r = 0, 1, 2, . . . , p − 1. Dessa forma, ωk e igual aalguma das potencias basicas 1, ω, ω2, . . . , ωp−1 para todo inteiro positivo k. Entao

P (ω) = anωn + an−1ω

n−1 + . . .+ a1ω + a0

= (a0+ap+a2p+. . .)+(a1+ap+1+a2p+1+. . .)ω+. . .+(ap−1+a2p−1+a3p−1+. . .)ωp−1

= S(1 + ω + ω2 + . . .+ ωp−1) = 0.

Portanto, ω e uma raiz de P . Voltando ao problema, o polinomio P possui grau 49.Dessa forma, para p = 3 o polinomio possuira duas raızes, para p = 5 o polinomio pos-suira quatro raızes e, assim sucessivamente, ate que para p = 17 o polinomio possuira16 raızes. Finalmente, o polinomio P tera 2+ 4+6+10+12+16 = 50 raızes. ComoP possui grau 49 entao ele sera identicamente nulo, ou seja, a0 = a1 = . . . = a49 = 0.

3

Page 88: Álgebra nível 3

POT 2012 - Algebra - Nıvel 3 - Aula 8 - Prof. Cıcero Thiago/ Prof. Marcelo

Mendes

4. (USAMO) Se P (x), Q(x), R(x) e S(x) sao polinomios tais que P (x5) + xQ(x5) +x2R(x5) = (x4 + x3 + x2 + x+ 1)S(x), prove que (x− 1) e um fator de P (x).

Solucao.

Seja ω = cos2π

5+ i · sin 2π

5, ou seja, ω5 = 1. Substituindo x por ω, ω2, ω3 e ω4 na

igualdade P (x5) + xQ(x5) + x2R(x5) = (x4 + x3 + x2 + x+ 1)S(x), temos

P (1) + ωQ(1) + ω2R(1) = 0,

P (1) + ω2Q(1) + ω4R(1) = 0,

P (1) + ω3Q(1) + ωR(1) = 0,

P (1) + ω4Q(1) + ω3R(1) = 0.

Multiplicando cada uma das equacoes acima por −ω, −ω2, −ω3 e −ω4, temos

−ωP (1)− ω2Q(1)− ω3R(1) = 0,

−ω2P (1)− ω4Q(1)− ωR(1) = 0,

−ω3P (1)− ωQ(1)− ω4R(1) = 0,

−ω4P (1) − ω3Q(1)− ω2R(1) = 0.

Somando as 8 igualdades e usando o fato que 1 + ω + ω2 + ω3 + ω4 = 0 encontramos5P (1) = 0 ⇔ P (1) = 0, ou seja, x− 1|P (x).

5. Mostre que para cada n ∈ N maior que 1 vale

sinπ

nsin

nsin

n. . . sin

(n− 1)π

n=

n

2n−1.

Solucao.

Sejam 1, ω, ω2, . . ., ωn−1, ω = cos2π

n+ i · sin 2π

n, as raızes do polinomio xn − 1.

Assim,xn − 1 = (x− 1)(x− ω)(x− ω2) . . . (x− ωn−1).

Se dividirmos os dois lados da igualdade acima por x− 1, encontraremos

xn−1 + xn−1 + . . . + x+ 1 = (x− ω)(x− ω2) . . . (x− ωn−1).

Fazendo x = 1 na ultima equacao temos

n = (1− ω)(1− ω2) . . . (1− ωn−1).

4

Page 89: Álgebra nível 3

POT 2012 - Algebra - Nıvel 3 - Aula 8 - Prof. Cıcero Thiago/ Prof. Marcelo

Mendes

A ultima igualdade implica que n = |1− ω||1−ω2| . . . |1−ωn−1|. Usando a definicaode modulo de um numero complexo temos

|1− ωk| =

(

1− cos2kπ

n

)2

+ sin22kπ

n=

2− 2 cos2kπ

n= 2 sin

n

para k = 0, 1, . . . , n− 1. Multiplicando tudo encontramos a igualdade desejada.

Exercıcios propostos

1. (ARML) Seja z uma raiz de x5 − 1 = 0, com z 6= 1. Determine o valor de z15 + z16 +z17 + . . . + z50.

2. (AIME) Seja z um numero complexo tal que z +1

z= 2cos 3◦. Determine o menor

inteiro maior que z2000 +1

z2000.

3. (Harvard - MIT) O polinomio f(x) = x2007+17x2006+1 tem raızes distintas r1, r2, . . . ,

r2007. Um polinomio P de grau 2007 tem a propriedade que P

(

rj +1

rj

)

= 0 para

j = 1, . . . , 2007. Determine o valor deP (1)

P (−1).

4. (OCM) Seja A1, A2, . . ., An os vertices de um polıgono regular de n lados inscritona circunferencia unitaria S e A um ponto dessa circunferencia. Encontre o valormaximo do produto P dos n segmentos A1A, A2A, . . . AnA e a posicao de A para oqual esse maximo ocorre.

5. Prove que para todo natural n e α real satisfazendo n > 1 e sinα 6= 0, o polinomio

P (x) = xn sinα− x sinnα+ sin(n− 1)α

e divisıvel pelo polinomio Q(x) = x2 − 2x cosα+ 1.

6. (AIME) A equacaox10 + (13x − 1)10 = 0

possui 10 raızes complexas r1, r1, r2, r2, r3, r3, r4, r4, r5, r5. Determine o valorde

1

r1r1+

1

r2r2+

1

r3r3+

1

r4r4+

1

r5r5.

5

Page 90: Álgebra nível 3

POT 2012 - Algebra - Nıvel 3 - Aula 8 - Prof. Cıcero Thiago/ Prof. Marcelo

Mendes

7. Seja f(x) = x2004+2x2003+3x2002+. . .+2004x+2005 e z = cos( π

1003

)

+i·sin( π

1003

)

.

Expresse o produtof(z)f(z2) . . . f(z2005)

na forma ab, em que a e b sao inteiros.

8. (AIME) Seja v e w numeros distintos escolhidos arbitrariamente entre as raızes de

z1997 − 1 = 0. Determine a probabilidade de acontecer√

2 +√3 ≤ |v + w|.

9. (Romenia) Seja Un o conjunto das raızes n - esimas da unidade. Prove que asafirmacoes a seguir sao equivalentes:a) existe um α ∈ Un tal que 1 + α ∈ Un;b) existe β ∈ Un tal que 1− β ∈ Un.

Bibliografia

1. Complex Numbers form A to ZTitu Andreescu e Dorin Andrica

2. Equations and InequalitiesJiri Herman, Radan Kucera e Jaromir Simsa

3. PrecalculusRichard Rusczyk

4. Problem - Solving StrategiesArthur Engel

5. 101 Problems in Algebra - From the traninig of the USA IMO teamTitu Andreescu e Zuming Feng

6. Mathematical MiniaturesSvetoslav Savchev e Titu Andreescu

6

Page 91: Álgebra nível 3

Polos Olímpicos de TreinamentoCurso de Álgebra - Nível 3Prof. Cícero Thiago / Prof. Marcelo Mendes

Aula 9

Somas de Newton

Chamaremos de somas de Newton as somas das k - esimas potencias das raızes de umpolinomio. Iniciaremos este material com alguns problemas que servem de motivacao paraa sequencia da teoria.

Exemplo 1. Calcule

(

1 +√5

2

)10

+

(

1−√5

2

)10

.

Solucao. Seja x =1 +

√5

2e y =

1−√5

2. Defina σ1 = x+ y = 1, σ2 = x · y = −1 e Sk =

xk + yk, k natural. Temos que x e y sao raızes do polinomio quadratico P (z) = z2 − z − 1.Dessa forma,

x2 − x− 1 = 0 (1)

ey2 − y − 1 = 0. (2)

Multiplique (1) por xk−2 e (2) por yk−2, k ≥ 2, assim

xk − xk−1 − xk−2 = 0 (3)

e

yk − yk−1 − yk−2 = 0. (4)

Adicionando (3) e (4) temos

xk + yk − (xk−1 + yk−1)− (xk−2 + yk−2) = 0. ⇔Sk − Sk−1 − Sk−2 = 0 ⇔

Sk = Sk−1 + Sk−2.

Portanto,S0 = x0 + y0 = 1 + 1 = 2.

S1 = x1 + y1 = x+ y = 1

Page 92: Álgebra nível 3

POT 2012 - Algebra - Nıvel 3 - Aula 9 - Prof. Cıcero Thiago/ Prof. Marcelo

Mendes

S2 = S1 + S0 = 3

S3 = S2 + S1 = 4

S4 = S3 + S2 = 7

S5 = S4 + S3 = 11

S6 = S5 + S4 = 18

S6 = S5 + S4 = 18

S7 = S6 + S5 = 29

S8 = S7 + S6 = 47

S9 = S8 + S7 = 76

S10 = S9 + S8 = 123

Exemplo 2. Escreva Sk = xk+ yk em funcao de Sk−1 = xk−1+ yk−1, Sk−2 = xk−2+ yk−2,σ1 = x+ y e σ2 = x · y, k ≥ 2.

Solucao.

Seja P (z) = z2 − σ1 · z + σ2 um polinomio cujas raızes sao x e y. Entao,

x2 − σ1 · x+ σ2 = 0 (1)

e

y2 − σ1 · y + σ2 = 0. (2)

Multiplicando (1) por xk−2 e (2) por yk−2, k ≥ 2, temos

xk − σ1 · xk−1 + σ2 · xk−2 = 0 (3)

e

yk − σ1 · yk−1 + σ2 · yk−2 = 0. (4)

Somando (3) e (4) temos

xk + yk − σ1(xk−1 + yk−1) + σ2(x

k−2 + yk−2) = 0 ⇔Sk − σ1 · Sk−1 + σ2 · Sk−2 = 0 ⇔

Sk = σ1 · Sk−1 − σ2 · Sk−2.

Exemplo 3. Escreva Sk = xk + yk + zk em funcao de Sk−1 = xk−1 + yk−1 + zk−1,Sk−2 = xk−2 + yk−2 + zk−2, Sk−3 = xk−3 + yk−3 + zk−3, σ1 = x+ y + z, σ2 = xy+ yz + zx

e σ3 = xyz, k ≥ 3.

Solucao.

2

Page 93: Álgebra nível 3

POT 2012 - Algebra - Nıvel 3 - Aula 9 - Prof. Cıcero Thiago/ Prof. Marcelo

Mendes

Seja P (z) = z3 − σ1 · z2 + σ2 · z − σ3 um polinomio cujas raızes sao x, y e z. Entao,

x3 − σ1 · x2 + σ2 · x− σ3 = 0 (1)

y3 − σ1 · y2 + σ2 · y − σ3 = 0 (2)

z3 − σ1 · z2 + σ2 · z − σ3 = 0. (3)

Multiplicando (1) por xk−3, (2) por yk−3 e (3) por zk−3, k ≥ 3, encontramos

xk − σ1 · xk−1 + σ2 · xk−2 − σ3 · xk−3 = 0 (4)

yk − σ1 · yk−1 + σ2 · yk−2 − σ3 · yk−3 = 0 (5)

zk − σ1 · zk−1 + σ2 · zk−2 − σ3 · zk−3 = 0. (6)

Somando (4), (5) e (6), temos

xk+yk+zk−σ1·(xk−1+yk−1+zk−1)+σ2·(xk−2+yk−2+zk−2)−σ3·(xk−3+yk−3+zk−3) = 0 ⇔

Sk − σ1 · Sk−1 + σ2 · Sk−2 − σ3 · Sk−3 = 0 ⇔Sk = σ1 · Sk−1 − σ2 · Sk−2 + σ3 · Sk−3.

Teorema 1. (Newton) Seja P (x) = anxn + an−1x

n−1 + . . . + a1x + a0 um polinomioe sejam r1, r2, . . ., rn as raızes do polinomio. Seja Sk = rk1 + rk2 + . . . + rkn, k ≥ n.Entao, anSk + an−1Sk−1 + . . . + a0Sk−n = 0. Em particular, quando k = n, temosanSn + an−1Sn−1 + . . .+ na0 = 0.

Demonstracao. Como r1, r2, . . ., rn sao as raızes de P (x) entao

P (ri) = anrn

i + an−1rn−1i

+ . . .+ a1ri + a0 = 0, i = 1, 2, . . . , n.

Multiplicando cada uma das equacoes por rk−n

iencontramos

anrk

1 + an−1rk−11 + . . . + a0r

k−n

1 = 0

anrk

2 + an−1rk−12 + . . . + a0r

k−n

2 = 0

...

anrk

n + an−1rk−1n + . . . + a0r

k−n

n = 0

Somando todas as equacoes encontramos

an(rk

1 + . . . + rkn) + an−1(rk−11 + . . .+ rk−1

n ) + . . .+ a0(rk−n

1 + . . .+ rk−n

n ) = 0 ⇔

3

Page 94: Álgebra nível 3

POT 2012 - Algebra - Nıvel 3 - Aula 9 - Prof. Cıcero Thiago/ Prof. Marcelo

Mendes

anSk + an−1Sk−1 + . . .+ a0Sk−n = 0.

Em particular, quando k = n, Sk−n = S0 = r01+r02+ . . .+r0n = n, assim anSn+an−1Sn−1+. . .+ na0 = 0.

Exercıcios Resolvidos

1. Sejam r1, r2, . . . , r1000 as raızes de x1000 − 10x + 10 = 0. Determine o valor der10001 + r10002 + . . .+ r10001000 .

Solucao.

Temos que a1000, a1 e a0 sao os unicos coeficientes diferentes de 0. Entao, pelo te-orema de Newton, S1000 − 10S1 + 1000 · 10 = 0. Como o coeficiente de x999 e zerotemos que S1 = 0 e S1000 = −10000.

2. (Bulgaria) Sejam x e y numeros reais que satisfazem as equacoes

x2 + y2 = 2

x3 + y3 = 2√2.

Ache o valor de x4 + y4.(a) 2 (b) 3 (c) 4 (d) 4

√2 (e) nao pode ser determinado.

Solucao.

Seja σ1 = x+ y e σ2 = xy e usando as ideias do exemplo 2 temos que

Sk = σ1 · Sk−1 − σ2 · Sk−2.

Assim

S2 = σ1 · S1 − σ2 · S0 ⇔S2 = σ1 · σ1 − 2σ2 ⇔

S2 = σ21 − 2σ2.

Da mesma forma

S3 = σ1 · S2 − σ2 · S1 ⇔S3 = σ1 · (σ2

1 − 2σ2)− σ2 · σ1 ⇔S3 = σ3

1 − 3σ1 · σ2,

4

Page 95: Álgebra nível 3

POT 2012 - Algebra - Nıvel 3 - Aula 9 - Prof. Cıcero Thiago/ Prof. Marcelo

Mendes

e

S4 = σ1 · S3 − σ2 · S2 ⇔S4 = σ1 · (σ3

1 − 3σ1 · σ2)− σ2 · (σ21 − 2σ2) ⇔

S4 = σ41 − 4σ2

1 · σ2 + 2σ22 .

O enunciado diz que S2 = 2 e S3 = 2√2, assim

σ21 − 2σ2 = 2

e

σ31 − 3σ1 · σ2 = 2

√2.

Queremos S4 = x4 + y4 = σ41 − 4σ2

1 · σ2 + 2σ22 , com x, y reais. Por outro lado,

(σ21 − 2σ2)

2 = 22 ⇔ σ4 − 4σ21 · σ2 + 4σ2

2 = 4 ⇔ S4 + 2σ22 = 4 ⇔ S4 = 4 − 2σ2

2 . Alemdisso,

σ31 − 3σ1 · σ2 = 2

√2 ⇔

σ1 · (σ21 − 3σ2) = 2

√2 ⇔

σ21 · (σ2

1 − 3σ2)2 = (2

√2)2 ⇔

(2 + 2σ2)(2 − σ2)2 = 8 ⇔

(1 + σ2)(4− 4σ2 + σ22) = 4 ⇔

4− 4σ2 + σ22 + 4σ2 − 4σ2

2 + σ32 = 4 ⇔

σ32 − 3σ2

2 = 0 ⇔σ2 = 0 ou σ2 = 3.

Finalmente, S4 = 4 ou S4 = −14. Como x, y sao reais temos que S4 ≥ 0, ou seja,S4 = 4.

3. Determine todas as solucoes reais da equacao 4√1− x+ 4

√15 + x = 2.

Solucao.

Faca 4√1− x = a e 4

√15 + x = b, assim a4 = 1− x e b4 = 15 + x. Dessa forma,

{

a4 + b4 = 16a + b = 2

Se σ1 = a + b = 2 e σ2 = a · b. Assim, S4 = a4 + b4 = σ41 − 4σ2

1 · σ2 + 2σ22 =

24 − 16σ2 + 2σ22 = 16. Assim, σ2 = 0 ou σ2 = 8.

5

Page 96: Álgebra nível 3

POT 2012 - Algebra - Nıvel 3 - Aula 9 - Prof. Cıcero Thiago/ Prof. Marcelo

Mendes

1◦ caso: σ2 = 0

{

a + b = 2a · b = 0

Assim, a = 0 e b = 2 ou a = 2 e b = 0. Se a = 0 e b = 2 entao x = 1. Se a = 2 eb = 0 entao x = −15.

2◦ caso: σ2 = 8

{

a + b = 2a · b = 8

Nesse caso, a e b nao sao reais e, alem disso, x nao e real.

4. (OBM) Sejam a, b e c numeros reais nao nulos tais que a + b + c = 0. Calcule ospossıveis valores de

(a3 + b3 + c3)2 · (a4 + b4 + c4)

(a5 + b5 + c5)2.

Solucao. Usando as ideias do exemplo 3, ou seja, Sk = σ1 ·Sk−1−σ2 ·Sk−2+σ3 ·Sk−3,com σ1 = a+ b+ c, σ2 = ab+ bc+ ca e σ3 = abc.

Alem disso, S1 = σ1 = 0 entao

S2 = a2 + b2 + c2 = (a+ b+ c)2 − 2(ab+ bc+ ca) = σ21 − 2σ2 = −2σ2,

S3 = σ1 · S2 − σ2 · S1 + σ3 · S0 = 3σ3,

S4 = σ1 · S3 − σ2 · S2 + σ3 · S1 = 2σ22 ,

S5 = σ1 · S4 − σ2 · S3 + σ3 · S2 = −5σ2 · σ3.

Portanto,(a3 + b3 + c3)2 · (a4 + b4 + c4)

(a5 + b5 + c5)2=

(3σ3)2 · 2σ2

2

(−5σ2 · σ3)2=

18

25.

5. Sejam x, y numeros reais nao nulos satisfazendo x2 + xy + y2 = 0. Determine(

x

x+ y

)2001

+

(

y

x+ y

)2001

.

Solucao.

6

Page 97: Álgebra nível 3

POT 2012 - Algebra - Nıvel 3 - Aula 9 - Prof. Cıcero Thiago/ Prof. Marcelo

Mendes

Observe quex

x+ y+

y

x+ y= 1 e

x

x+ y· y

x+ y=

xy

x2 + 2xy + y2=

xy

xy= 1. E facil ver

quex

x+ ye

y

x+ ysao as raızes de t2− t+1 = 0. Assim, Sk =

(

x

x+ y

)k

+

(

y

x+ y

)k

satisfaz

{

Sk+2 = Sk+1 − Sk, k ≥ 0S0 = 2, S1 = 1

A sequencia Sk, k ≥ 0, e 2, 1,−1,−2,−1, 1, 2, 1, . . . e Sk = Sl para k ≡ l (mod 6).Portanto, S2001 = S3 = −2.

Exercıcios propostos

1. Sejam r1, r2, . . . , r20 as raızes de x20 − 19x+ 2. Determine r201 + r202 + . . .+ r2020.

2. Sejam r1, r2, . . . , r20 as raızes de x20 − 19x2 + 2. Determine r201 + r202 + . . .+ r2020 .

3. Fatore x3 + y3 + z3 − 3xyz.

4. Sejam x1 e x2 as raızes do polinomio P (x) = x2 − 6x + 1. Prove que xn1 + xn2 e uminteiro nao divisıvel por 5 para todo inteiro nao negativo n.

5. Determine todos os valores de a ∈ R tais que as raızes x1, x2 e x3 de x3−6x2+ax+a =

0 satisfazem(x1 − 3)3 + (x2 − 3)3 + (x3 − 3)3 = 0.

6. Mostre que se a, b e c ∈ R e a+ b+ c = 0, entao

a4 + b4 + c4 = 2(ab+ ac+ bc)2.

7. Resolva o sistema de equacoesx+ y + z = 2,

x2 + y2 + z2 = 6,

x3 + y3 + z3 = 8.

8. Sejam a, b, c numeros reais nao nulos tais que a+b+c = 0 e a3+b3+c3 = a5+b5+c5.

Prove que a2 + b2 + c2 =6

5.

7

Page 98: Álgebra nível 3

POT 2012 - Algebra - Nıvel 3 - Aula 9 - Prof. Cıcero Thiago/ Prof. Marcelo

Mendes

9. Se a3 + b3 + c3 = a2 + b2 + c2 = a+ b+ c = 1, prove que abc = 0.

10. Determine todas as solucoes reais do sistema

x+ y + z = 1

x3 + y3 + z3 + xyz = x4 + y4 + z4 + 1.

11. Prove que se a+ b+ c = 0, entao

a7 + b7 + c7

7=

a5 + b5 + c5

5· a

2 + b2 + c2

2.

12. Prove que se a+ b+ c = 0, entao

a5 + b5 + c5

5=

a3 + b3 + c3

3· a

2 + b2 + c2

2.

13. Sejam x1 e x2 as raızes do polinomio P (x) = x2 + x+ c. Determine o valor de c se

2x312 + x2

+2x32

2 + x1= −1.

14. Prove que o numero

c =3

1

9+

3

−2

9+

3

4

9

e uma raiz de F (x) = x3 + 3√6x2 − 1.

Bibliografia

1. Equations and inequalities - Elementary Problems and Theorems in Algebra and Num-ber TheoryJiri Herman, Radan Kucera e Jaromir Simsa

2. the Art of Problem Solving, vol. 2: and BeyondRichard Rusczyk e Sandor Lehoczky

3. Problem - Solving StrategiesArthur Engel

4. Topicos de Matematica Elementar, vol. 6 - PolinomiosAntonio Caminha Muniz Neto

5. Polinomios Simetricos - Revista Eureka 25Carlos A. Gomes

8

Page 99: Álgebra nível 3

Polos Olímpicos de TreinamentoCurso de Álgebra - Nível 3Prof. Cícero Thiago / Prof. Marcelo Mendes

Aula 10

Diferencas finitas e o polinomio interpolador de Lagrange.

1. Diferencas Finitas

Seja P (x) um polinomio de grau m. Defina ∆k+1P (n) = ∆kP (n + 1) −∆kP (n), ∀k ≥ 1,com ∆1P (n) = P (n + 1)− P (n).

Teorema 1. Seja P (x) um polinomio de grau m, em que m > 0. Entao ∆mP (n) e umaconstante diferente de zero.Demonstracao.

Seja P (x) = amxm + am−1xm−1 + . . .+ a1x+ a0 um polinomio qualquer. Entao

∆P (n) = P (n+ 1)− P (n) =

am(n+1)m+am−1(n+1)m−1+ . . .+a1(n+1)+a0− (amnm+am−1nm−1+ . . .+a1n+a0).

E facil ver que o grau de ∆P (n) e m − 1 e seu termo de maior grau e am(

m1

)

nm−1, poisam 6= 0.Dessa forma, para k ≤ m, o grau de ∆kP (n) e m− k. Portanto, quando k = m o grau de∆mP (n) e 0, assim ∆mP (n) e uma constante diferente de zero.

Problema 1. Determine todos os polinomios P (x) tais que P (x+ 1)− P (x) = 2x+ 1, ∀x.Solucao. Temos que ∆P (n) = 2n+ 1, assim ∆2P (n) = ∆P (n+ 1)−∆P (n) = 2(n+ 1) +1− (2n+1) = 2. Como ∆2P (n) e constante e diferente de zero entao P (x) tem grau 2, ouseja, P (x) = ax2 + bx+ c, assim

a(x+ 1)2 + b(x+ 1) + c− ax2 − bx− c = 2x+ 1 ⇔

2ax+ a+ b = 2x+ 1,∀x.Dessa forma, 2a = 2 e a + b = 1. Portanto, a = 1 e b = 0 e P (x) = x2 + c, para algumaconstante c.

Page 100: Álgebra nível 3

POT 2012 - Algebra - Nıvel 3 - Aula 10 - Prof. Cıcero Thiago/ Prof. Marcelo

Mendes

Problema 2. (AIME) Sejam x1, x2, . . . , x7 numeros reais tais que

x1 + 4x2 + 9x3 + 16x4 + 25x5 + 36x6 + 49x7 = 1,

4x1 + 9x2 + 16x3 + 25x4 + 36x5 + 49x6 + 64x7 = 12,

9x1 + 16x2 + 25x3 + 36x4 + 49x5 + 64x6 + 81x7 = 123.

Determine o valor de 16x1 + 25x2 + 36x3 + 49x4 + 64x5 + 81x6 + 100x7.Solucao.

Defina P (n) = (n+1)2x1+(n+2)2x2+(n+3)2x3+ . . .+(n+7)2x7. Temos que P (0) = 1,P (1) = 12, P (2) = 123 e que P (n) e um polinomio quadratico. Desejamos calcular o valorde P (3). Assim

∆P (0) = P (1) − P (0) = 11,

∆P (1) = P (2) − P (1) = 111,

∆2P (0) = ∆P (1) −∆P (0) = 100.

Dessa forma, ∆2P (n) = 100, ∀n. Assim, ∆2P (1) = ∆P (2) − ∆P (1) ⇔ 100 = ∆P (2) −111 ⇔ ∆P (2) = 211. Mas, ∆P (2) = P (3)− P (2) ⇔ 211 = P (3)− 123 ⇔ P (3) = 334.

2. Polinomio interpolador de Lagrange

Vamos resolver um problema que serve de motivacao para a construcao do polinomio in-terpolador de Lagrange.

Problema 3. Determine um polinomio quadratico tal que P (−1) = −4, P (1) = 2 eP (2) = −1.

Solucao. Seja P (x) = ax2 + bx+ c, com a 6= 0, entao

P (−1) = a·(−1)2+b·(−1)+c = −4, P (1) = a·12+b·1+c = 2 e P (2) = a·22+b·2+c = −1.

Resolvendo o sistema linear encontramos P (x) = −2x2 + 3x+ 1.

Resolver um sistema linear pode ser muito trabalhoso e difıcil se o grau do polinomio au-mentar muito. Para resolver problemas deste tipo e outros problemas vamos estudar opolinomio interpolador de Lagrange.

Teorema 2. Dados n ∈ N, a0, a1, . . . , an e b0, b1, . . . , bn numeros complexos coma0, a1, . . . , an distintos, existe um unico polinomio P (x) tal que

P (αi) = βi, 0 ≤ i ≤ n.

Demonstracao:

2

Page 101: Álgebra nível 3

POT 2012 - Algebra - Nıvel 3 - Aula 10 - Prof. Cıcero Thiago/ Prof. Marcelo

Mendes

Vamos inicialmente determinar o polinomio. Para isto, observe os polinomios

Dk(x) =(x− α0)(x− α1) . . . (x− αk−1)(x− αk+1) . . . (x− αn)

(αk − α0)(αk − α1) . . . (αk − αk−1)(αk − αk+1) . . . (αk − αn)

e facil ver que

Dk(αi) =

{

1, i = k

0, i 6= k

Agora multiplique Dk(x) pelo numero βk e, entao, adicione todos esses polinomios resul-tando no polinomio

P (x) =n∑

k=0

βkDk(x)

que satisfaz as condicoes do enunciado. Para demonstrar a unicidade sejam P1 e P2 doispolinomios, que satisfazem as condicoes impostas. O Polinomio H(x) = P1(x)−P2(x) temgrau no maximo n e possui n + 1 raızes α0, α1, . . . , αn, portanto, e identicamente nulo.Com isso, P1(x) ≡ P2(x).

O polinomio Dk(x) nao caiu do ceu entao vamos ver a sua construcao. Temos que

Dk(αi) =

{

1, i = k

0, i 6= k

Como Dk(αi) = 0 para todo i 6= k, entao

Dk(x) = C(x− α0) . . . (x− αk−1)(x− αk+1) . . . (x− αn).

Para determinar o valor de C vamos substituir x = αk e usar a condicao Dk(αk) = 1.Entao,

1 = C(αk − α0) . . . (αk − αk−1)(αk − αk+1) . . . (αk − αn).

Portanto,

Dk(x) =(x− α0)(x− α1) . . . (x− αk−1)(x− αk+1) . . . (x− αn)

(αk − α0)(αk − α1) . . . (αk − αk−1)(αk − αk+1) . . . (αk − αn).

Problema 4. (Mandelbrot) Seja P (x) um polinomio de grau 2 tal que P (0) = cos3 10◦,P (1) = cos 10◦ sin2 10◦ e P (2) = 0. Determine P (3).

Solucao. Usando o polinomio interpolador de Lagrange temos

P (x) = P (0) · (x− 1)(x− 2)

(0− 1)(0− 2)+ P (1) · (x− 0)(x− 2)

(1− 0)(1 − 2)+ P (2) · (x− 0)(x − 1)

(2− 0)(2 − 1).

Como P (0) = cos3 10◦, P (1) = cos 10◦ sin2 10◦ e P (2) = 0 entao

3

Page 102: Álgebra nível 3

POT 2012 - Algebra - Nıvel 3 - Aula 10 - Prof. Cıcero Thiago/ Prof. Marcelo

Mendes

P (x) = cos3 10◦ · (x− 1)(x − 2)

(0− 1)(0 − 2)+ cos 10◦ sin2 10◦ · (x− 0)(x − 2)

(1− 0)(1 − 2)+ 0 · (x− 0)(x− 1)

(2− 0)(2 − 1).

Queremos P (3) assim

P (3) = cos3 10◦ · (3− 1)(3− 2)

(0− 1)(0− 2)+ cos 10◦ sin2 10◦ · (3− 0)(3 − 2)

(1− 0)(1 − 2)+ 0 · (3− 0)(3 − 1)

(2− 0)(2 − 1)⇔

P (3) = cos3 10◦ · 1 + cos 10◦ sin2 10◦ · (−3) ⇔

P (3) = cos3 10◦ − 3 cos 10◦ sin2 10◦ ⇔

P (3) = cos3 10◦ − 3 cos 10◦(1− cos2 10◦) ⇔

P (3) = 4 cos3 10◦ − 3 cos 10◦ ⇔

P (3) = cos 3 · 10◦ = cos 30◦ =

√3

2.

Problema 5. (IMO Short List)Seja F1 = F2 = 1, Fn+2 = Fn+1 +Fn e seja f um polinomiode grau 990 tal que f(k) = Fk, k ∈ {992, 993, . . . , 1982}. Mostre que f(1983) = F1983−1.

Solucao. Temos que f(k + 992) = Fk+992, para k = 0, 1, . . . , 990 e precisamos provar quef(992 + 991) = F1983 − 1. Seja g(x) = f(x + 992), que tambem possui grau 990. Nossonovo problema e tal que se g(k) = Fk+992, para k = 0, 1, . . . , 990, entao g(991) = F1983 − 1.Usando o polinomio interpolador de Lagrange temos que

g(x) =

990∑

k=0

g(k) · (x− 0)(x − 1) . . . (x− k + 1)(x − k − 1) . . . (x− 990)

(k − 1)(k − 2) . . . 1 · (−1) . . . (k − 990).

Entao

g(991) =

990∑

k=0

g(k)

(

991

k

)

(−1)k =

990∑

k=0

(

991

k

)

Fk+992(−1)k.

Sabemos que Fn =an − bn√

5, em que a =

1 +√5

2e b =

1−√5

2. Assim,

990∑

k=0

(

991

k

)

Fk+992(−1)k =1√5

[

990∑

k=0

(

991

k

)

ak+992(−1)k −990∑

k=0

(

991

k

)

bk+992(−1)k

]

.

4

Page 103: Álgebra nível 3

POT 2012 - Algebra - Nıvel 3 - Aula 10 - Prof. Cıcero Thiago/ Prof. Marcelo

Mendes

Usando binomio de Newton temos que

990∑

k=0

(

991

k

)

ak+992(−1)k = a992990∑

k=0

(

991

k

)

(−a)k = a992[

(1− a)991 + a991]

.

Mas a2 = a+ 1, entao

a992[

(1− a)991 + a991]

= a(a− a2)991 + a1983 = −a+ a1983.

Temos que b2 = b+ 1 entao

990∑

k=0

(

991

k

)

Fk+992(−1)k =1√5(a1983 − b1983 − a+ b)

=a1983 − b1983√

5− a− b√

5= F1983 − 1.

1. Determine o polinomio P , de menor grau possıvel, que satisfaz P (1) = 3, P (2) = 7,P (3) = 13, P (4) = 21 e P (5) = 31. (AHSME)

2. Um polinomio cubico f(n) satisfaz f(0) = 5, f(1) = 4, f(2) = 17 e f(3) = 56. De-termine f(4).

3. (AIME) A partir de uma sequencia de numeros reais A = a1, a2, a3, . . ., defina ∆A

como a sequencia de numeros reais a2 − a1, a3 − a2, a4 − a3, . . ., em que o n - esimotermo e an+1 − an. Se todos os termos da sequencia ∆(∆A) sao iguais a 1, e quea19 = a92 = 0, determine a1.

4. Determine um polinomio de grau 3 tal que:(a) P (1) = 2, P (2) = 1, P (3) = 4 e P (4) = 3.(b) P (1) = 1, P (i) = 2, P (−1) = 3 e P (−i) = 4.

5. Se f e um polinomio de grau n tal que f(i) = 2i para i = 0, 1, . . . , n, determinef(n+ 1).

6. (Mandelbrot) Se P (x) e um polinomio de grau n tal que P (0) = 1, P (1) = −1,P (2) = 1, P (3) = −1, . . ., P (n) = (−1)n. Determine P (n+ 1).

7. (Mandelbrot) Se P (x) e um polinomio de grau n com P (1) = 1, P (2) = 3, P (4) = 9,. . . e P (2n) = 3n. Determine P (2n+1).

5

Page 104: Álgebra nível 3

POT 2012 - Algebra - Nıvel 3 - Aula 10 - Prof. Cıcero Thiago/ Prof. Marcelo

Mendes

8. (USAMO)Se P (x) e um polinomio de grau n tal que P (k) =k

k + 1, k = 0, 1, . . . , n,

determine P (n+ 1).

9. (IMO Short List)Se P (x) e um polinomio de grau n tal que P (i) =1

(

n+1k

) , para

i = 0, 1, 2, . . . , n. Determine P (n+ 1).

10. Resolva o sistema

ax1 + a2x2 + a3x3 + a4x4 = 1bx1 + b2x2 + b3x3 + b4x4 = 1cx1 + c2x2 + c3x3 + c4x4 = 1dx1 + d2x2 + d3x3 + d4x4 = 1

se a, b, c, d sao reais nao - nulos e distintos.

11. (Reino Unido) Sejam a1, a2, . . . , an numeros inteiros positivos distintos. Prove que

para qualquer inteiro positivo k o numero

n∑

i=1

aki∏

j 6=i

(ai − aj)e um inteiro.

12. Sejam x1, x2, . . . , xn, n ≥ 2, n numeros reais distintos no intervalo [−1, 1]. Proveque

1

t1+

1

t2+ . . .+

1

tn≥ 2n−2,

em que∏

j 6=i

|xi − xj|.

13. Prove que se m e n sao inteiros, 1 < m < n, entao

n∑

k=1

(−1)kkm(

n

k

)

= 0.

14. Prove a identidaden∑

k=0

(−1)n−kkn+1

(

n

k

)

=n(n+ 1)!

2.

15. Seja f ∈ R [X] um polinomio de grau n com coeficiente lıder igual a 1, e sejamx0 < x1 < . . . < xn numeros inteiros. Prove que existe k ∈ {0, 1, . . . , n} tal que

|f(xk)| ≥n!

2n.

6

Page 105: Álgebra nível 3

POT 2012 - Algebra - Nıvel 3 - Aula 10 - Prof. Cıcero Thiago/ Prof. Marcelo

Mendes

16. (IMO Short List - 1997) Seja f um polinomio com coeficientes inteiros, e seja p umnumero primo tal que f(0) = 0, f(1) = 1 e f(k) e congruente a 0 ou 1 modulo p,para todo inteiro positivo k. Mostre que o grau de f e no mınimo p− 1.

17. (USAMO) Prove que qualquer polinomio monico de grau n, com coeficientes reais,pode ser escrito como media aritmetica de dois polinomios monicos de grau n com n

raızes reais cada.

18. Sejam a1, a2, a3, a4, b1, b2, b3, b4 numeros reais tais que bi − aj 6= 0 para i, j =1, 2, 3, 4. Suponha que exista um unico conjunto de numeros X1, X2, X3, X4 taisque

X1

b1 − a1+

X2

b1 − a2+

X3

b1 − a3+

X4

b1 − a4= 1.

X1

b2 − a1+

X2

b2 − a2+

X3

b2 − a3+

X4

b2 − a4= 1.

X1

b3 − a1+

X2

b3 − a2+

X3

b3 − a3+

X4

b3 − a4= 1.

X1

b4 − a1+

X2

b4 − a2+

X3

b4 − a3+

X4

b4 − a4= 1.

Determine X1 +X2 +X3 +X4 em funcao de a1, a2, a3, a4, b1, b2, b3, b4.

Bibliografia

1. Problems from the bookTitu Andreescu e Gabriel Dospinescu

2. Winning SolutionsEdward Lozansky e Cecil Rousseau

3. The Mandelbrot Problem BookSam Vandervelde

4. Kvant Selecta: Algebra and Analysis, IISerge Tabachnikov

5. Lagrange Interpolation FormulaKin Y. LiMathematical Excalibur - July - September, 2010

7

Page 106: Álgebra nível 3

Polos Olímpicos de TreinamentoCurso de Álgebra - Nível 3Prof. Cícero Thiago / Prof. Marcelo Mendes

Aula 11

Polinomios em Z[x] (1)

Dizemos que P (x) ∈ Z[x] se P (x) = anxn + an−1x

n−1 + . . .+ a1x+ a0, com an, an−1, . . .,a1 e a0 inteiros.

Teorema 1. Sejam a e b numeros inteiros distintos e P (x) um polinomio com coeficientesinteiros, entao a− b|P (a)− P (b).

Demonstracao.

Seja P (x) = anxn + an−1x

n−1 + . . .+ a1x+ a0com an, an−1, . . ., a1 e a0 inteiros. Entao,

P (a)− P (b) = (anan + an−1a

n−1 + . . . + a1a+ a0)− (anbn + an−1b

n−1 + . . .+ a1b+ a0)

=

n∑

i=0

ai(ai − bi)

n∑

i=0

ai(a− b)(ai−1 + . . .+ bi−1)

= (a− b)

[

n∑

i=0

ai(ai−1 + . . . + bi−1)

]

= (a− b) · k, k ∈ Z.

Portanto, a− b|P (a)− P (b).

Problema 1. Seja P um polinomio monico com coeficientes inteiros. Seja x um real talque:(i) P (x) = 0(ii) P (⌊x⌋ + ⌈x⌉) = 2P (1) + P (0) + 1.Prove que x e irracional.

Solucao.

Page 107: Álgebra nível 3

POT 2012 - Algebra - Nıvel 3 - Aula 11 - Prof. Cıcero Thiago/ Prof. Marcelo

Mendes

Lema: O numero ⌊x⌋+ ⌈x⌉ e par se, e somente se, x e inteiro.

Demonstracao. Se x e inteiro, entao ⌊x⌋ = ⌈x⌉ = x, entao ⌊x⌋ + ⌈x⌉ = 2x, que e umnumero par. Se x nao e inteiro, entao k < x < k + 1, para algum inteiro k. Logo ⌊x⌋ = k

e ⌈x⌉ = k + 1, onde ⌊x⌋+ ⌈x⌉ = 2k + 1, que e um numero ımpar. Por outro lado, x e umaraiz do polinomio pela condicao i. Pelo teorema das raızes racionais e como P e monicoentao toda raiz racional e inteira. Portanto, basta mostrar que x nao e inteiro.//Se x e inteiro entao ⌊x⌋+ ⌈x⌉ e par. Dessa forma,

⌊x⌋+ ⌈x⌉|P (⌊x⌋ + ⌈x⌉)− P (0) = 2P (1) + 1,

ou seja, ⌊x⌋+ ⌈x⌉ deveria ser ımpar. Portanto, x e irracional.

Problema 2. Seja P (x) um polinomio com coeficientes inteiros tal que P (21) = 17, P (32) =−247 e P (37) = 33. Prove que se P (N) = N + 51 para algum inteiro N , entao N = 26.

Solucao. Usando o teorema 1 temos

N − 21|N + 34 ⇒ N − 21|N + 34− (N − 21) ⇒ N − 21|55 (1)

N − 32|N + 298 ⇒ N − 32|N + 298− (N − 32) ⇒ N − 32|330 (2)

N − 37|N + 18 ⇒ N − 37|N + 18− (N − 37) ⇒ N − 37|55 (3)

De (1) temos que N − 21 = ±1, ±5, ±11 ou ±55. Os possıveis valores de N sao:22, 20, 26, 16, 32, 10, 76 e −34. O unico destes valores de N que nao contradiz 2 e3 e N = 26.

Problema 3. (Peru TST) Seja k um inteiro positivo e seja P (x) um polinomio com coefi-cientes inteiros. Prove que existe um inteiro positivo n tal que

P (1) + P (2) + . . .+ P (n)

e divisıvel por k.

Solucao. Pelo teorema 1 temos que P (kj+ i) ≡ P (i) (mod kj), em particular, P (kj+ i) ≡P (i) (mod k). Logo,

P (1) + P (2) + . . .+ P (k2) =

k−1∑

j=0

k∑

i=1

P (kj + i)

≡k−1∑

j=0

k∑

i=1

P (i) (mod k)

2

Page 108: Álgebra nível 3

POT 2012 - Algebra - Nıvel 3 - Aula 11 - Prof. Cıcero Thiago/ Prof. Marcelo

Mendes

≡ k

(

k∑

i=1

P (i)

)

(mod k)

≡ 0 (mod k)

Logo, considerando n = k2, temos que P (1) + P (2) + . . .+ P (n) e divisıvel por k.

Exercıcios propostos

1. (USAMO) Sejam a, b e c numeros inteiros distintos. Prove que nao e possıvel encon-trar um polinomio P (x) de coeficientes inteiros tal que P (a) = b, P (b) = c e P (c) = a.

2. (AIME) Seja P (x) um polinomio com coeficientes inteiros tal que P (17) = 10 eP (24) = 17. Alem disso, a equacao P (n) = n + 3 possui duas solucoes inteiras dis-tintas n1 e n2. Calcule n1 · n2.

3. (IMO) Seja P (x) um polinomio de grau n > 1 com coeficientes inteiros e seja k uminteiro positivo. Considere o polinomio Q(x) = P (P (. . . P (P (x)) . . .)), onde P ocorrek vezes. Prove que existem no maximo n inteiros t tais que Q(t) = t.

4. (Romenia TST) Seja n um inteiro positivo e

f(x) = amxm + am−1xm−1 + . . . + a1x+ a0,

com m ≥ 2, um polinomio com coeficientes inteiros, tal que:i. a2, a3, . . . , am sao divisıveis por todos os fatores primos de n,ii. a1 e n sao primos entre si.Prove que para qualquer inteiro positivo k, existe um inteiro positivo c, tal que f(c)e divisıvel por nk.

5. (Baltic Way) Seja P um polinomio com coeficientes inteiros tal que P (−n) < P (n) <n para algum inteiro n. Prove que P (−n) < −n.

Bibliografia

1. III Olimpiada Nacional Escolar de Matematica - 2006Jorge Tipe, John Cuya, Claudio Espinoza e Sergio Vera

3

Page 109: Álgebra nível 3

Polos Olímpicos de TreinamentoCurso de Álgebra - Nível 3Prof. Cícero Thiago / Prof. Marcelo Mendes

Aula 12

Irredutibilidade de polinomios.

Continuaremos, neste artigo, trabalhando com polinomios em Z[x]. Alem disso,vamos di-zer que um polinomio com coeficientes inteiros P (x) e irredutıvel sobre Z se, e somentese, nao for possıvel escrever P (x) como produto de dois polinomios (nao constantes) comcoeficientes inteiros. Vamos comecar com um problema de motivacao!!

Problema 1. Prove que o polinomio

(x− a1)(x− a2) . . . (x− an)− 1,

em que a1, a2, . . ., an sao inteiros distintos, nao pode ser escrito como produto de doispolinomios nao constantes com coeficientes inteiros, ou seja, e irredutıvel.

Solucao.

Suponha, por contradicao, que

f(x) = (x− a1)(x− a2) . . . (x− an)− 1 = p(x)q(x),

em que p(x) e q(x) sao polinomios com coeficientes inteiros com grau menor que n. Entao

g(x) = p(x) + q(x)

e um polinomio com coeficientes inteiros com grau menor que n. Entao

p(ai)q(ai) = f(ai) = −1

e ambos p(ai) e q(ai) sao inteiros,

|p(ai)| = |q(ai)| = 1

e

p(ai) + q(ai) = 0.

Page 110: Álgebra nível 3

POT 2012 - Algebra - Nıvel 3 - Aula 12 - Prof. Cıcero Thiago/ Prof. Marcelo

Mendes

Assim, g(x) possui pelo menos n raızes. Mas o grau g < n, entao g(x) ≡ 0. Entao

p(x) = −q(x) e f(x) = −p(x)2,

implicando que o coeficiente lıder de f(x) e um numero negativo, o que e impossıvel, poiso polinomio e monico.

Teorema 1. (Criterio de Einsenstein) Seja P (x) = anxn + an−1x

n−1 + . . . + a1x+ a0 umpolinomio com coeficientes inteiros ai e seja p um numero primo que satisfaz as seguintescondicoes(i) p nao divide an;(ii) p divide a0, a1, . . . , an−1;(iii) p2 nao divide a0.Entao P (x) e irredutıvel sobre Z.

Demonstracao. Suponha, por absurdo, que existem polinomios nao constantes F (x) eG(x) com coeficientes inteiros tais que P (x) = F (x)G(x). Logo,

P (x) = anxn + . . .+ a1x+ a0

F (x) = bsxs + . . . + b1x+ b0

G(x) = ctxt + . . .+ c1x+ c0.

Sem perda de generalidade podemos considerar que s ≤ t, como P (x) = F (x)G(x), aorealizarmos o produto poderemos comparar os coeficientes dos termos semelhantes, obtendoas seguintes igualdades

a0 = b0c0

a1 = b0c1 + b1c0

a2 = b0c2 + b1c1 + b2c0

...

as = b0cs + b1cs−1 + . . .+ bs−1c1 + bsc0

...

at = b0ct + b1ct−1 + . . .+ bsct−s

...

an = bsct.

Por (ii) e (iii) temos que a0 = b0c0 e divisıvel por p e nao e divisıvel por p2, assim, exata-mente um dos numeros b0 e c0 e divisıvel por p. Sem perda de generalidade suponha quep divide b0 e que p nao divide c0.Como p divide a1 = b0c1+b1c0 e b0, entao p divide b1c0, porem como p nao divide c0, entao

2

Page 111: Álgebra nível 3

POT 2012 - Algebra - Nıvel 3 - Aula 12 - Prof. Cıcero Thiago/ Prof. Marcelo

Mendes

p divide b1.Portanto, raciocionando da mesma maneira, podemos demonstrar que p divide b2, b3, . . .,bs−1 e bs. Logo, p divide bsct = an, contrariando (i).

Problema 2. Prove que o polinomio P (x) = x2 + x+ 1 e irredutıvel sobre Z.

Solucao. Nao e difıcil provar que P (x) e irredutıvel sobre Z se, e somente se, Q(x) =P (x+ 1) e irredutıvel sobre Z, Logo

Q(x) = (x+ 1)2 + (x+ 1) + 1

= x2 + 3x+ 3.

O primo p = 3 satisfaz o criterio de Eisenstein, portanto Q(x) e irredutıvel sobre Z e, comisso, P (x) tambem sera.

Problema 3. (TST Romenia) Sejam a, n numeros inteiros, e p um numero primo tal quep > |a|+ 1. Prove que o polinomio f(x) = xn + ax+ p nao pode ser representado como oproduto de dois polinomios com coeficientes inteiros.

Solucao. Seja z uma raiz complexa do polinomio. Vamos provar que |z| > 1. Suponha que|z| ≤ 1, entao zn + az = −p, entao:

p = |zn + az| = |z||zn−1 + a| ≤ |zn−1|+ |a| ≤ 1 + |a|,

contrariando o fato que p > |a|+1. Agora, seja f(x) = g(x)h(x) uma decomposicao de f(x)em polinomios com coeficientes inteiros entao p = g(0)h(0), entao |g(0)| = 1 ou |h(0)| = 1.Suponha que |g(0)| = 1. Se z1, z2, . . . , zk sao raızes de g(x) entao sao tambem raızes def(x), assim:

1 = |g(0)| = |z1z2 . . . zk| = |z1||z2| . . . |zk| > 1,

que e uma contradicao.

Exercıcios Propostos

1. Prove que o polinomioP (x) = x101 + 101x100 + 102

e irredutıvel em Z [x].

2. Prove que para todo numero primo p, o polinomio

P (x) = xp−1 + xp−2 + . . .+ x+ 1

e irredutıvel em Z [x].

3

Page 112: Álgebra nível 3

POT 2012 - Algebra - Nıvel 3 - Aula 12 - Prof. Cıcero Thiago/ Prof. Marcelo

Mendes

3. Prove que para todo inteiro positivo n, o polinomio P (x) = x2n

+ 1 e irredutıvel emZ [x].

4. Prove que para quaisquer inteiros distintos a1, a2, . . ., an o polinomio

P (x) = (x− a1)2(x− a2)

2 . . . (x− an)2 + 1

nao pode ser escrito com um produto de dois polinomios nao constantes com coefici-entes inteiros.

5. Seja p um primo da forma 4k+3, k inteiro. Prove que para qualquer inteiro positivon, o polinomio (x2 + 1)n + p e irredutıvel em Z[x].

6. Seja p um numero primo. Prove que o polinomio

P (x) = xp−1 + 2xp−2 + 3xp−3 + . . .+ (p− 1)x+ p

e irredutıvel sobre Z[x].

7. (IMO) Seja n > 1 um inteiro e f(x) = xn + 5xn−1 +3. Mostre que f(x) e irredutıvelsobre Z.

Bibliografia

1. III Olimpiada Nacional Escolar de Matematica - 2006Jorge Tipe, John Cuya, Claudio Espinoza e Sergio Vera

2. Putnam and BeyondRazvan Gelca e Titu Andreescu

4

Page 113: Álgebra nível 3

Polos Olímpicos de TreinamentoCurso de Álgebra - Nível 3Prof. Cícero Thiago / Prof. Marcelo Mendes

Aula 13

Substituicao trigonometrica.

Um metodo de resolucao para equacoes e inequacoes algebricas bastante eficiente e o dasubstituicao trigonometrica. Vamos ver alguns exemplos.

Problema 1. Sejam a, b, c e d numeros reais tais que a2+ b2 = 1, c2 + d2 = 1 e ac+ bd = 0.Determine ab+ cd.

Solucao. Se a2 + b2 = 1 e c2 + d2 = 1 entao a = sinα, b = cosα, c = sinβ e d = cos β, emque 0 ≤ α ≤ 2π e 0 ≤ β ≤ 2π. Neste caso temos

ac+ bd = sinα · sinβ + cosα · cos β = cos(α− β).

Pelas condicoes do problema ac+ bd = 0 = cos(α− β). Alem disso,

ab+ cd = sinα · cosα+ sin β · cos β =

1

2(sin 2α+ sin 2β) = sin(α+ β) · cos(α− β) = 0.

Problema 2. Resolva a equacao x3 − 3x =√x+ 2.

Solucao. E facil ver que x ≥ −2. Considere os seguintes casos:1. −2 ≤ x ≤ 2. Faca x = 2cos a, 0 ≤ a ≤ π, a equacao resultante e

8 cos3 a− 6 cos a =√

2(cos a+ 1) ⇔

2 cos 3a =

4 cos2a

2⇔

cos 3a = cosa

2.

Entao, 3a− a

2= 2mπ, m ∈ Z ou 3a+

a

2= 2nπ, n ∈ Z. Como 0 ≤ a ≤ π, as solucoes neste

caso sao

x = 2cos 0 = 2, x = 2cos4π

5e x = 2cos

7.

Page 114: Álgebra nível 3

POT 2012 - Algebra - Nıvel 3 - Aula 13 - Prof. Cıcero Thiago/ Prof. Marcelo

Mendes

2. x > 2. Entao x3 − 4x = x(x2 − 4) > 0 e

x2 − x− 2 = (x− 2)(x + 1) > 0 ⇔

x >√x+ 2.

Portanto,x3 − 3x > x >

√x+ 2,

e, com isso, nao temos solucoes.

Problema 3. Sejam a, b e c numeros reais. Prove que

(ab+ bc+ ca− 1)2 ≤ (a2 + 1)(b2 + 1)(c2 + 1).

Solucao. Seja a = tanx, b = tan y e c = tan z com −π

2< x, y, z <

π

2. Entao a2 + 1 =

sec2 x, b2 + 1 = sec2 y e c2 + 1 = sec2 z. Multiplicando os dois lados da desigualdade porcos2 x cos2 y cos2 z encontramos

[(ab+ bc+ ca− 1) cos x cos y cos z]2 ≤ 1.

E facil ver que

(ab+ bc) cos x cos y cos z = sinx sin y cos z + sin y sin z cos x = sin y sin(x+ z)

e

(ca− 1) cos x cos y cos z = sin z sinx cos y − cos x cos y cos z

= − cos y cos(x+ z).

Consequentemente,[(ab+ bc+ ca− 1) cos x cos y cos z]2

= [sin y sin(x+ z)− cos y cos(x+ z)]2

= cos2(x+ y + z) ≤ 1.

Problemas propostos

1. (OBMU) Resolva a equacao em R: x =

2 +√

2−√2 + x.

2. (ITA) Para quais valores do parametro real a existe um numero real x satisfazendo√1− x2 ≥ a− x.

2

Page 115: Álgebra nível 3

POT 2012 - Algebra - Nıvel 3 - Aula 13 - Prof. Cıcero Thiago/ Prof. Marcelo

Mendes

3. Dados quatro numeros reais distintos no intervalo (0, 1), mostre que existem doisdeles, x e y, tais que

0 < x√

1− y2 − y√

1− x2 <1

2.

4. (IME) Seja a uma constante real positiva. Resolva a equacao√a√

a+√a2 − x2 +√

3a√

a−√a2 − x2 = 2

√2x, para x ∈ R e 0 ≤ x ≤ a.

5. Determine todas as solucoes reais da equacao x+√1− x2 =

√2(2x2 − 1).

6. Seja xn uma sequencia satisfazendo

xn+1 =

√3xn − 1

xn +√3, n ≥ 1.

Prove que a sequencia e periodica.

7. A sequencia xn satisfaz√xn+2 + 2 ≤ xn ≤ 2 para todo n ≥ 1. Determine todos os

possıveis valores de x1986.

8. Sejam a, b e c numeros reais tais que ab+ ac+ bc = 1. Prove que

a

1− a2+

b

1− b2+

c

1− c2=

4abc

(1− a2)(1− b2)(1 − c2).

9. Determine o maior valor de

S = (1− x1)(1− y1) + (1− x2)(1 − y2)

se x21 + x22 = y21 + y22 = c2.

Bibliografia

1. Metodos no estandares para la resolucion de ecuaciones y desigualdades.V. P. Suprun

2. Mathematical Olympiad ChallengesTitu Andreescu e Razvan Gelca

3. 103 trigonometry problems - From the training of the USA IMO teamTitu Andreescu e Zuming Feng

3

Page 116: Álgebra nível 3

Polos Olímpicos de TreinamentoCurso de Álgebra - Nível 3Prof. Cícero Thiago / Prof. Marcelo Mendes

Aula 14

Revisao I.

Problema 1. Sejam x, y e z numeros reais tais que1

x+

1

y+

1

z= 1. Prove que

(x− 1)(y − 1)(z − 1) ≥ 8.

Solucao. A desigualdade e equivalente a(

x− 1

x

)(

y − 1

y

)(

z − 1

z

)

≥ 8

xyz⇔

(

1− 1

x

)(

1− 1

y

)(

1− 1

z

)

≥ 8

xyz.

Usando a condicao inicial e MA ≥ MG temos que

1− 1

x=

1

y+

1

z≥ 2

1

y

1

z=

2√yz

.

Analogamente, 1− 1

y≥ 2√

zxe 1− 1

z≥ 2√

xy. Multiplicando as tres desigualdades verifica-

mos a validade da desigualdade inicial. A igualdade ocorre se, e somente se, x = y = z = 3.

Problema 2. Sejam a, b, c ∈ R∗+. Prove a desigualdade

abc

(1 + a)(a+ b)(b+ c)(c + 16)≤ 1

81.

Solucao. Temos que(1 + a)(a+ b)(b+ c)(c + 16)

=(

1 +a

2+

a

2

)

(

a+b

2+

b

2

)

(

b+c

2+

c

2

)

(c+ 8 + 8)

≥ 33

a2

4· 3 3

ab2

4· 3 3

bc2

4· 3 3

64c

4≥ 81abc.

Entao,abc

(1 + a)(a+ b)(b+ c)(c + 16)≤ 1

81.

Page 117: Álgebra nível 3

POT 2012 - Algebra - Nıvel 3 - Aula 14 - Prof. Cıcero Thiago/ Prof. Marcelo

Mendes

Problema 3. (Colombia) Seja f : R → R uma funcao. Prove que se a funcao

F (x) = f(x) +1

2(f(x))2 +

1

3(f(x))3

e periodica, entao a funcao f tambem sera.

Solucao. Note que F (x) = g(f(x)), em que g(x) = x +1

2x2 +

1

3x3 e uma funcao estrita-

mente crescente. Seja T o perıodo de F , ou seja, F (x+ T ) = F (x) para todo x. Como g eestritamente crescente entao ela e tambem injetora. Dessa forma, g(f(x+T )) = g(f(x)) ⇒f(x+ T ) = f(x) para todo x, ou seja, f e periodica.

Exercıcios propostos

1. Sejam a, b, c e d numeros reais positivos. Prove que (a2 + a+ 1)(b2 + b+ 1)(c2 + c+1)(d2 + d+ 1) ≥ 81abcd.

2. Prove que para quaisquer x, y reais positivos entaox

x4 + y2+

y

x2 + y4≤ 1

xy.

3. Sejam x, y e z numeros reais positivos. Prove que

x

x+√

(x+ y)(x+ z)+

y

y +√

(y + x)(y + z)+

z

z +√

(z + x)(z + y)≤ 1.

4. Sejam x, y, z numeros reais positivos tais que x4 + y4 + z4 = 1. Determine o valormınimo de

x3

1− x8+

y3

1− y8+

z3

1− z8.

5. Determine todas as funcoes f : N → R tais que, para quaisquer k, m e n, valef(km) + f(kn)− f(k)f(mn) ≥ 1.

6. (Rioplatense) A cada numero inteiro positivo n associamos um inteiro nao - negativof(n), de modo que sejam satisfeitas as seguintes condicoes:(i) f(ab) = f(a) + f(b),(ii) f(n) = 0, se n e primo maior que 10,(iii) f(1) < f(243) < f(2) < 10.Ache f(1998) sabendo que e menor que 10.

2

Page 118: Álgebra nível 3

POT 2012 - Algebra - Nıvel 3 - Aula 14 - Prof. Cıcero Thiago/ Prof. Marcelo

Mendes

7. (Coreia) Seja f : N× N → N uma funcao tal que f(1, 1) = 2,

f(m+ 1, n) = f(m,n) +m e

f(m,n+ 1) = f(m,n)− n

para todos m, n ∈ N. Ache todos os pares (p, q) tais que f(p, q) = 2001.

8. Seja f : R → R∗ uma funcao tal que f(x+ 2) = f(x− 1)f(x+ 5), para todo x ∈ R.

Prove que f e periodica.

9. (Ira) Seja f : R+ → R

+ uma funcao estritamente decrescente tal que para todox, y ∈ R+,

f(x+ y) + f(f(x) + f(y)) = f(f(x+ f(y)) + f(y + f(x))).

Prove que f(f(x)) = x.

10. Seja n um inteiro positivo ımpar e sejam x1, x2, . . . , xn numeros reais distintos.Encontre todas as funcoes bijetivas

f : {x1, x2, . . . , xn} → {x1, x2, . . . , xn}

tais que |f(x1)− x1| = |f(x2)− x2| = . . . = |f(xn)− xn|.

3

Page 119: Álgebra nível 3

Polos Olímpicos de TreinamentoCurso de Álgebra - Nível 3Prof. Cícero Thiago / Prof. Marcelo Mendes

Aula 15

Funcoes definidas implicitamente III

Problema 1. A funcao f satisfaz a equacao f(x+1)+ f(x−1) =√2f(x) para todo x real.

Prove que esta funcao e periodica.

Solucao. Usando a equacao que foi dada temos que

f(x+ 2) + f(x) =√2f(x+ 1)

=√2[√

2f(x)− f(x− 1)]

= 2f(x)−√2f(x− 1) ⇔

f(x+ 2) = f(x)−√2f(x− 1).

Segue quef(x+ 4) = f(x+ 2)−

√2f(x+ 1)

= f(x)−√2 [f(x+ 1) + f(x− 1)]

= −f(x).

Portanto,

f(x+ 8) = −f(x+ 4) = f(x).

Problema 2. (Ira) Seja f : R∗+ −→ R

∗+ uma funcao estritamente decrescente tal que para

quaisquer x, y ∈ R∗+,

f(x+ y) + f(f(x) + f(y)) = f(f(x+ f(y)) + f(y + f(x))).

Prove que f(f(x)) = x.

Solucao. Fazendo y = x temos

f(2x) + f(2f(x)) = f(2f(x+ f(x))).

Page 120: Álgebra nível 3

POT 2012 - Algebra - Nıvel 3 - Aula 15 - Prof. Cıcero Thiago/ Prof. Marcelo

Mendes

Na ultima igualdade substitua x por f(x) entao

f(2f(x)) + f(2f(f(x))) = f(2f(f(x) + f(f(x)))).

Subtraindo as duas igualdades temos que

f(2f(f(x)))− f(2x) = f(2f(f(x) + f(f(x))))− f(2f(x+ f(x))).

Se f(f(x) > x, o lado esquerdo da ultima igualdade e um numero negativo, entao

f(f(x) + f(f(x)) > f(x+ f(x)) ⇔

f(x) + f(f(x)) < x+ f(x),

o que e uma contradicao. Uma contradicao semelhante ocorre se f(f(x)) < x. Portanto,f(f(x)) = x.

Problema 3. (Tchecoslovaquia) Seja f : R∗+ −→ R

∗+ uma funcao tal que

f(xf(y)) + f(yf(x)) = 2xy,

para quaisquer x, y ∈ R∗+. Prove que f(x) = x para todo x.

Solucao. Fazendo x = y temos que f(xf(x)) = x2 e, em particular para x = 1, f(f(1)) = 1.Assim,

f(1)2 = f(f(1) · f(f(1))) = f(f(1)) = 1 ⇔f(1) = 1.

Fazendo y = 1 na equacao original temos que f(x) + f(f(x)) = 2x. Esta condicao implicaque se f(a) = f(b) entao a = b, ou seja, f e injetora. Agora vamos fazer a substituicao

x = zf(z) e y =1

z, com z > 0, e lembrando que f(zf(z)) = z2 temos:

f

(

zf(z)f

(

1

z

))

+ f

(

1

zf(zf(z))

)

= 2zf(z)1

z⇔

f

(

zf(z)f

(

1

z

))

= f(z).

Como f e injetora segue que f(z) · f(

1

z

)

= 1. Fazendo x = z e y =1

zna equacao original

temos que

f

(

zf

(

1

z

))

+ f

(

f(z)

z

)

= 2.

Como f

(

1

z

)

=1

f(z)entao

f

(

z

f(z)

)

+ f

(

f(z)

z

)

= 2.

2

Page 121: Álgebra nível 3

POT 2012 - Algebra - Nıvel 3 - Aula 15 - Prof. Cıcero Thiago/ Prof. Marcelo

Mendes

Alem disso,

f

(

z

f(z)

)

· f(

f(z)

z

)

= 1.

Portanto,

f

(

z

f(z)

)

= f

(

f(z)

z

)

= 1 = f(1) ⇔

f(z) = z.

Exercıcios propostos

1. (Australia) Seja f : R → R∗ uma funcao tal que f(x + 2) = f(x − 1)f(x + 5) para

todo x ∈ R. Prove que f e periodica.

2. (Australia) Prove que existe uma unica funcao f : R∗ → R satisfazendo f(x) =

xf

(

1

x

)

e f(x) + f(y) = 1 + f(x+ y) para quaisquer x, y ∈ R∗ tais que x+ y 6= 0.

3. (Australia) Determine todas as funcoes reais f : R∗+ → R

∗+ tais que f(1) =

1

2e

f(xy) = f(x)f

(

3

y

)

+ f(y)f

(

3

x

)

.

4. (IMO) Seja f : R → R tal que, para alguma constante positiva a, f satisfaz

f(x+ a) =1

2+

f(x)− f(x)2, ∀x ∈ R.

Prove que f e periodica.

5. (IMO) Determine todas as funcoes f : R → R tais que

f(x2 + f(y)) = f(x)2 + y,

para quaisquer x, y ∈ R.

3

Page 122: Álgebra nível 3

Polos Olímpicos de TreinamentoCurso de Álgebra - Nível 3Prof. Cícero Thiago / Prof. Marcelo Mendes

Aula 16

Sequencias I

1. Progressao Aritmetica

Definicao 1: Uma progressao aritmetica e uma sequencia a1, a2, . . . ou somente (an)(finita ou infinita) satisfazendo a2 − a1 = a3 − a2 = . . . = r; sendo r chamado de razao daprogressao.

Teorema 1. Se (an) e uma progressao aritimetica de razao r, entao

an = a1 + (n− 1)r,

para todo n inteiro e positivo.

Demonstracao. Pela definicao de progressao aritmetica, temos

a2 − a1 = r

a3 − a2 = r

a4 − a3 = r

...

an − an−1 = r.

Somando essas n− 1 igualdades, obtemos an − a1 = (n− 1)r, isto e, an = a1 + (n− 1)r.

Problema 1. Prove que nao existem inteiros positivos a e r tais que os numeros

a, a+ r, a+ 2r, a+ 3r, . . . , a+ nr, . . .

sejam todos quadrados perfeitos.

Page 123: Álgebra nível 3

POT 2012 - Algebra - Nıvel 3 - Aula 16 - Prof. Cıcero Thiago/ Prof. Marcelo

Mendes

Solucao. Suponha que existem a e r como desejado. Para k suficientemente grande temos

a + kr >

(

r − 1

2

)2

e a + kr = q2, para algum q ∈ N. Mas neste caso temos q >r − 1

2,

donde

(q + 1)2 = q2 + 2q + 1 > q2 + 2

(

r − 1

2

)

+ 1 = a+ kr + r = a+ (k + 1)r.

Logo, a+ (k + 1)r nao sera quadrado perfeito. Um absurdo!

Problema 2. Prove que√2,

√3 e

√5 nao podem ser termos de uma mesma progressao

aritmetica.

Problema 3. Prove que os termos de uma P.A. qualquer em que 0 nao participa verficama relacao:

1

a1a2+

1

a2a3+ . . .+

1

an−1an=

n− 1

a1an.

Problema 4. Prove que se a, b e c sao numeros positivos entao a2, b2 e c2 estao em P.A.

se, e somente se,1

b+ c,

1

a+ ce

1

a+ btambem estao em P.A.

Problema 5. Prove que se uma progressao aritmetica de inteiros positivos contem um qua-drado, entao ira conter infinitos quadrados.

Problema 6. O conjunto dos inteiros positivos e particionado em varias progressoes aritmeticas.Prove que pelo menos um dos termos iniciais e divisıvel pela razao de sua progressao.

Problema 7. Prove que os termos de uma P.A. qualquer em que 0 nao participa verficama relacao:

1√a1 +

√a2

+ . . .+1√

an−1 +√an

=n− 1√a1 +

√an

.

Problema 8. (Putnan) Prove que nao existem quatro coeficientes binomiais consecutivos

(

n

k

)

,

(

n

k + 1

)

,

(

n

k + 2

)

,

(

n

k + 3

)

(n, k inteiros positivos e 4 ≤ k + 3 ≤ n) que estao em progressao aritmetica.

Problema 9. (OCM) Os lados de um triangulo medem 3, 7 e 8, respectivamente. Mostreque os angulos deste triangulo, medidos em graus, entao em progressao aritmetica.

2

Page 124: Álgebra nível 3

POT 2012 - Algebra - Nıvel 3 - Aula 16 - Prof. Cıcero Thiago/ Prof. Marcelo

Mendes

Teorema 2. A soma dos n primeiros termos da progressao aritmetica (an) = (a1, a2, . . . ,

an, . . .) e igual a

Sn =(a1 + an)n

2.

Demonstracao.

Sn = a1 + a2 + a3 + . . .+ an−2 + an−1 + an

Sn = an + an−1 + an−2 + . . . + a3 + a2 + a1.

Saı, 2Sn = (a1 + an) + (a2 + an−1) + . . .+ (an + a1).

Observe que, ao passar de um parenteses para o seguinte, a primeira parcela aumenta de r

e a segunda parcela diminui de r, o que nao altera a soma. Portanto, todos os parentesessao iguais ao primeiro, (a1 + an).

Logo, 2Sn = (a1 + an)n e Sn =(a1 + an)n

2.

Problema 10. (OCM) Seja S = 12− 22+32− 42+ . . .+−19982 +19992. Expresse S comoa soma de 1000 numeros ımpares, todos eles termos de uma progressao aritmetica.

Solucao. Usaremos apenas a fatoracao da diferenca de dois quadrados:

a2 − b2 = (a+ b)(a− b).

Assim, teremos:

S = 12 + (32 − 22) + (52 − 42) + . . .+ (19992 − 19982)

= 1 + (3 + 2)(3 − 2) + (5 + 4)(5 − 4) + . . . + (1999 + 1998)(1999 − 1998)

= 1 + 5 + 9 + . . . + 3997.

Esta ultima expressao contem extamente 1000 numeros ımpares em P.A.

Problema 11. (China) Seja Sn a soma dos n primeiros termos de uma progressao aritmetica

(an). Se S15 > 0 e S16 < 0, determine o maior entre os numerosS1

a1,S2

a2, . . .,

S15

a15.

Problema 12. Demonstre que em toda P.A., com numero ımpar de termos, o termo medio eigual a diferenca entre a soma dos termos de ordem ımpar e a soma dos termos de ordem par.

Problema 13. (Espanha) Calcule a soma dos quadrados dos 100 primeiros termos de umaprogressao aritmetica, dado que a soma dos 100 primeiros termos e −1 e a soma dos termosde ordem par (a2, a4, . . . , a100) e 1.

3

Page 125: Álgebra nível 3

POT 2012 - Algebra - Nıvel 3 - Aula 16 - Prof. Cıcero Thiago/ Prof. Marcelo

Mendes

Problema 14. Suponha que a1, a2, . . . , an estao em progressao aritmetica. Ache a formulapara a21 + a22 + . . . + a2n em termos de n, a1 e an.

Problema 15. (ITA) Numa progressao aritmetica com n termos, n > 1, sabemos que o

primeiro termo e igual a1 + n

ne a soma deles vale

1 + 3n

2. Entao o produto da razao desta

progressao pelo ultimo termo e igual a:

(a) 2n (b)2

n(c) 3n (d)

3

n(e) 5n

Problema 16. (ITA) Seja a1, a2, . . . uma progressao aritmetica infinita tal que

n∑

k=1

a3k = n√2 + πn2, ∀n ∈ N

∗.

Determine o primeiro termo e a razao da progressao.

Problema 17. (IME) Determine as possıveis progressoes aritmeticas para as quais o re-sultado da divisao da soma dos seus n primeiro termos pela soma dos seus 2n primeirostermos seja independente do valor de n.

Problema 18. (AIME) Seja a1, a2, a3, . . . uma progressao geometrica com razao 1 tal que

S1 = a1 + a2 + a3 + . . .+ a98,

S2 = a2 + a4 + a6 + . . .+ a98.

(a) Ache uma equacao relacionando S1 e S2.(b) Determine o valor de S2 sabendo que S1 = 137.

2. Progressao Geometrica

Definicao 2: Uma sequencia a1, a2, . . . , an, . . . e uma progressao geometrica se existeum numero q tal que para cada k = 1, 2, 3, . . . ,

ak+1 = qak

Teorema 3. Se (an) e uma progressao geometrica de razao q,entao

an = a1qn−1,

para todo inteiro positivo n.

4

Page 126: Álgebra nível 3

POT 2012 - Algebra - Nıvel 3 - Aula 16 - Prof. Cıcero Thiago/ Prof. Marcelo

Mendes

Demonstracao. Pela definicao de progressao geometrica e admitindo conhecidos o primeirotermo (a1 6= 0), a razao (q 6= 0) e o ındice (n) de um termo desejado, temos:

a2 = a1q

a3 = a2q

...

an = an−1q

Multiplicando essas n− 1 igualdades, temos:

a2a3a4 . . . an = a1a2a3 . . . an−1qn−1

⇒ an = a1qn−1

Teorema 4. A soma dos n primeiros termos de uma progressao geometrica (an) = (a1, a2, . . . , an, . . .)e igual a

Sn =a1q

n − a1

q − 1.

Demonstracao. Temos:

a1 + a1q + a1q2 + . . . + a1q

n−2 + a1qn−1. (1)

Multiplicando ambos os membros por q, obtemos:

qSn = a1q + a1q2 + . . .+ a1q

n−1 + a1qn. (2)

(2)− (1) ⇒ qSn − Sn = a1qn − a1 ⇒ Sn(q − 1) = a1q

n − a1.

Supondo q 6= 1, resulta:

Sn =a1q

n − a1

q − 1.

Teorema 5. Se (a1, a2, . . . , an, . . .) e uma P.G. com razao q tal que −1 < q < 1,entao

S = a1 + a2 + a3 + . . . + an + . . . =a1

1− q.

A demonstracao do teorema 5 ficara como exercıcio.

Problema 19. Determine a razao de uma P.G. de termos nao nulos tal que

an+2 = an+1 + an.

Solucao. Sabemos que an = a1qn−1. Entao

a1qn+1 = a1q

n + a1qn−1 ⇒ q2 − q − 1 = 0 ⇒ q =

1±√5

2.

5

Page 127: Álgebra nível 3

POT 2012 - Algebra - Nıvel 3 - Aula 16 - Prof. Cıcero Thiago/ Prof. Marcelo

Mendes

Problema 20. Prove que podemos eliminar alguns termos de um progressao aritmetica deinteiros positivos de tal maneira, que sempre podemos rearranjar os termos formando umaprogressao geometrica.

Problema 21. (OCM) Determine a soma dos n primeiros termos da sequencia:

1, 1 + 2, 1 + 2 + 22, 1 + 2 + 22 + 23, . . . , 1 + 2 + 22 + 23 + . . .+ 2n−1, . . .

Problema 22. Tres numeros reais nao nulos x, y, z, nessa ordem, estao em P.A. Seusquadrados, na mesma ordem, tambem estao em P.A. Nessas condicoes, prove que x, y, z,

nessa ordem, tambem estao em P.G.

Problema 23. Uma progressao aritmetica e uma progressao geometrica tem, cada uma,200 termos e a1 = b1 = 3 e a2 = b2 = 12. Determine os valores de i para os quais ai e umdivisor de bi.

Problema 24. (OCM) (a) Sabendo - se que os tres lados de um triangulo retangulo, de hi-potenusa a, entao em progressao geometrica. Determine os catetos do triangulo em funcaoapenas de a.(b) Mostre que a altura relativa a hipotenusa tambem faz parte da progressao.

Problema 25. Suponha que a1 = 2 e ak+1 = 3ak + 1 para todo k ≥ 1. Ache uma formulageral para a1 + a2 + . . .+ an.

Problema 26. (AIME) A soma dos infinitos termos de uma progressao geometrica e 2005.Uma nova sequencia obtida elevando ao quadrado cada termo da sequencia original, temsoma dez vezes maior que a soma original. Determine a razao da sequencia original.

Bibliografia

1. Fundamentos de matematica elementar 4Gelson Iezzi e Samuel Hazzan

2. Intermediate AlgebraRichard Rusczyk e Mathew Crawford

6

Page 128: Álgebra nível 3

Polos Olímpicos de TreinamentoCurso de Álgebra - Nível 3Prof. Cícero Thiago / Prof. Marcelo Mendes

Aula 17

Sequencias II

1. Recorrencias lineares

Uma recorrencia linear de ordem k com coeficientes constantes em uma variavel e

fn = cn−1fn−1 + cn−2fn−2 + . . .+ cn−kfn−k + g(n),

em que c1, c2, c3, . . ., cn−k sao constantes e g(n) e uma funcao de n. A recorrencia lineare chamada homogenea se g(n) ≡ 0 e, nao homogenea, caso contrario.

2. Recorrencias lineares de ordem 2 homogeneas

Teorema 1. Seja (fn)n≥1 uma sequencia de numeros reais tal que, para todo k ≥ 1 inteiro,tenhamos

fk+2 + rfk+1 + sfk = 0,

onde r, s sao constantes reais dadas, sendo r 6= 0. Se a equacao x2 + rx+ s = 0, chamadade equacao caracterıstica, tiver raızes reais α e β, entao existem constantes reais A e B,determinadas pelos valores de f1 e f2, tais que:(a) Se α 6= β, entao fn = Aαn−1 +Bβn−1 para todo n ≥ 1.(b) Se α = β, entao fn = Aαn−1 +B(n− 1)αn−1 para todo n ≥ 1.

Problema 1. Determine o termo geral da sequencia de Fibonacci definida por Fn =Fn−1 + Fn−2, n ≥ 2, F1 = F2 = 1.

Solucao.

A equacao caracterıstica associada a equacao em questao (Fn = Fn−1 + Fn−2) e

x2 − x− 1 = 0,

cujas raızes sao α =1 +

√5

2e β =

1−√5

2. As condicoes F1 = F2 = 1 implicam no sistema:

A + B = 1(

1 +√5

2

)

·A +

(

1−√5

2

)

·B = 1,

Page 129: Álgebra nível 3

POT 2012 - Algebra - Nıvel 3 - Aula 17 - Prof. Cıcero Thiago/ Prof. Marcelo

Mendes

cuja solucao e A =1 +

√5

2√5

e B = −1−√5

2√5

. Obtemos, portanto, a conhecida formula para

Fn:

Fn =1√5

(

1 +√5

2

)n

− 1√5

(

1−√5

2

)n

, para n ≥ 1.

Problema 2. Seja (an)n≥1 a sequencia dada por a1 = 1, a2 = 4 e, para todo inteiro positivok, ak+2 = 5ak+1 − 6ak. Calcule an em funcao de n.

Problema 3. (Romenia TST) Considere a sequencia (an)n≥0 definida por a0 = a1 = 1 ean+1 = 14an − an−1, n ≥ 1. Prove que para todo n ≥ 0, 2an − 1 e um quadrado perfeito.

Problema 4. (Ibero) Seja (an) e (bn) duas sequencias de numeros inteiros que verificam asseguinte condicoes:(i) a0 = 0; b0 = 8(ii) an+2 = 2an+1 − an + 2; bn+2 = 2bn+1 − bn(iii) a2n + b2n e um quadrado perfeito para todo n.Determinar pelo menos dois valores do par (a1992, b1992).

3. Recorrencias nao - lineares

Problema 5. A sequencia (xn)n≥1 e tal que x1 = 0 e

xn+1 = 5xn +√

24x2n + 1

para todo n ≥ 1. Prove que todos os termos da sequencia sao inteiros positivos.

Solucao. E facil ver que a sequencia e crescente e todos os termos sao positivos. Temostambem que a recorrencia original e equivalente a

x2n+1 − 10xnxn+1 + x2n − 1 = 0.

Substituindo n por n− 1 temos

x2n − 10xnxn−1 + x2n−1 − 1 = 0.

Entao, para n ≥ 2, os numeros xn+1 e xn−1 sao raızes positivas e distintas da equacao

x2 − 10xxn + x2n − 1 = 0.

Usando as relacoes de Girard temos que

xn+1 + xn−1 = 10xn ⇔xn+1 = 10xn − xn−1, ∀n ≥ 2.

Como x1 = 1 e x2 = 10, segue indutivamente que os termos da sequencia sao inteiros epositivos.

2

Page 130: Álgebra nível 3

POT 2012 - Algebra - Nıvel 3 - Aula 17 - Prof. Cıcero Thiago/ Prof. Marcelo

Mendes

Problema 6. Considere a sequencia (an)n≥1 tal que a1 = a2 = 1, a3 = 199 e

an+1 =1989 + anan−1

an−2, ∀n ≥ 3.

Prove que todos os termos da sequencia sao inteiros e positivos.

Problema 7. (Torneio das cidades) A sequencia xn esta definida pelas seguintes condicoes:

x1 = 19, x2 = 97, xn+2 = xn − 1

xn+1.

Demonstrar que existe um termo desta sequencia que e igual a 0. Determinar o ındice dessetermo.

Problema 8. (China) Seja (an) uma sequencia tal que a1 = 1, a2 = 2 ean+2

an=

a2n+1 + 1

a2n + 1para todo n ≥ 1.(a) Determine an em funcao de n.(b) Prove que 63 < a2008 < 78.

Bibliografia

1. Lecture notes on mathematical olympiad courses - For senior section vol.2Xu Jiagu

2. Mathematical Olympiad TreasuresTitu Andreescu e Bogdan Enescu

3. Topicos de matematica elementar vol.4Antonio Caminha Muniz Neto

4. Introducao a analise combinatoriaJose Plınio O. Santos, Margarida P. Mello e Idani T. C. Murari

3